Ginecologia y Obstetricia ENARM

• • • • •• • • • • •• ••• ••••••••••••••••••••• •• •••••••••••••• •••••••• •••••••••••••••••• • ••• •••••••••• •••

Views 251 Downloads 3 File size 31MB

Report DMCA / Copyright

DOWNLOAD FILE

Recommend stories

Citation preview







• •• • • • •

••

•••

••••••••••••••••••••• •• •••••••••••••• •••••••• •••••••••••••••••• • ••• •••••••••• •••• •• •• ••• •••••••••••••

•••••••••••• •••••• •• ••••••••••••••••••• ••••••••••••••••••••• • •••••••••••••••••• •••• ••••••••••••••••• •••••• •••••••••••••••• ••••••••••••••••••••• •• ••••••••• ••••••••••• ••••••••••••••••• • •••• •• •• ••••••••••• •••••••••••• •••••• ••••• •••••••••••••••• ••••••••••••• ••••••••• •••••••••••••••••••••

•••••••••••••• •••••••• •••• ••••••••••••••••• ••••••••••••• ••••••••

•••••••••••• ••••••••• ••• •••••••••••••••• ••••••• ••••••••••••• ••••••••••••••••••••

• •• • •• •• •• •• •• •• •• •• •• •• •• •• •• •• •• •• ••

•• •• •• •• •• •• •• •• • •• •• •• •• •• •• •• •• •• •• • • ••••••• • • • •••••••

•••• • • ••••••

• • • • • •

1 •









•• • • • ••• • • • • • • • • • • •

• • •

• • • • • • • • •





• • • • • • • • • • • • • • • • • • • • • • • •

••••• ••• • • •• • • • • ••• •

• • • •







• • • • •





•• •



Manual CTO de Medicina y Cirugía

2.a edición

Ginecología y obstetricia



Revisores Andrés Anzá Hernández Fabián H i lario Mendoza

Autora Mar Muñoz Muñiz

Grupo CTO Editorial

NOTA La medicina es una ciencia sometida a un cambio constante. A medida que la investigación y la experiencia clínica amplían nuestros conocimientos, son necesarios cambios en los tratamientos y la farmacoterapia. Los editores de esta obra han contrastado sus resultados con fuentes consideradas de confianza, en un esfuerzo por proporcionar información completa y general, de acuerdo con los criterios aceptados en el momento de la publicación. Sin embargo, debido a la posibilidad de que existan errores humanos o se produzcan cambios en las ciencias médicas, ni los editores ni cualquier otra fuente implicada en la preparación o la publicación de esta obra garantizan que la información contenida en la misma sea exacta y completa en todos los aspectos, ni son responsables de los errores u omisiones ni de los resultados derivados del empleo de dicha información. Por ello, se recomienda a los lectores que contrasten dicha información con otras fuentes. Por ejemplo, y en particular, se aconseja revisar el prospecto informativo que acompaña a cada medicamento que deseen administrar, para asegurarse de que la información contenida en este libro es correcta y de que no se han producido modificaciones en la dosis recomendada o en las contraindicaciones para la administración. Esta recomendación resulta de particular importancia en relación con fármacos nuevos o de uso poco frecuente. Los lectores también deben consultar a su propio laboratorio para conocer los valores normales.

No está permitida la reproducción total o parcial de este libro, su tratamiento informático, la transmisión de ningún otro formato o por cualquier medio, ya sea electrónico, mecánico, por fotocopia, por registro y otros medios, sin el permiso previo de los titulares del copyright. © CTO EDITORIAL, S.L. 2016

Diseño y maquetación: CTO Editorial C/ Francisco Silvela, 106; 28002 Madrid Tfno.: (0034) 91 782 43 30- Fax: (0034) 91 782 43 43 E-mail: [email protected] Página Web: www.grupocto.es ISBN Ginecología y obstetricia: 978-84-16527-25-0 ISBN Obra completa: 978-84-16527-11-3 Depósito legal: M-26880-2015

Ginecología y obstetricia



Manual CTO de Medicina y Cirugía 2.a edición

Grupo CTO •



Editorial

Q) ()

·-e

,_e

ct:S

u

....

+Q) +-

en

..e

o

_

>

.. ct:S

05. Control de la fertilidad

C')

o o u Q) r:::::

·(.!J 01. Ciclo genital femenino 1.1 . 1.2. 1.3. 1.4. 1.5. 1.6. 1.7. 1.8.

Hipotálam o ..... Hipófisis..... Ovario . An d rógenos Estrógenos ..... Progestágenos ..... Endometrio uterino ........ . Modelo fisiológico: integración del ciclo .....

02. Amenorreas .... 2.1 . 2.2. 2.3.

Amenorreas primarias ..... Amenorreas secundarias ..... Diagnóstico de la amenorrea ..... .

1

2

15

5.1 .

Eficacia contra ceptiva

5.2. 5.3. 5.4. 5.5. 5.6.

Métodos naturales ........................................................................................................ 15 Métodos de barrera .. ................................................................................................. 15 Dispositivo intrauterino ............................................................. . ...................... 16 Anticoncepción hormonal ...................................... ................................ ..... 16 Intercepción poscoital ............................................. ....... ....................................... 18

m

m

06. Esterilidad e infertilidad 6.1. 6.2. 6.3.

15

20

Causas de esterilidad 20 Estudio de la pareja infértil 20 Tratamiento .................................. ........................................................ 21 m

m

2

2 3

07. Endometriosis

23

3

7.1.

Epidemiología..........................................................

3

7.2. 7.3 .

Et iopatog en ia m.................. .. Localiza ción ...............................................................................

7.4. 7.5. 7.6. 7.7.

Clínica ............ ........ .................................................. .......................................................... 23 Di agnóstico .......................... . ......................................................... . ........................................ 24 Tratamiento .......................................................... . . ............................................. 24 Endometriosis y cáncer. . 24

5 S 6

23

m

.............................. m

•••••••••••••••••••••••••••••••

23 23

7

08. Infecciones ginecológicas en vagina y vulva. .

03. Síndrome de ovario poliquístico (SOP)

9

3.1. 3.2. 3.3 . 3.4. 3.5. 3.6.

9

Concepto ........ Etiopatog eni a.... Anatomía patológica ..... Clínica ..... Diagnóstico ..... .................................................. Tratamiento ..... . .............................................................................

04. Metrorragias 4.1. 4.2. 4.3 . 4.4.

Clasificación de las hemorragias uterinas ............. Causa de la hemorragia ..... .... . ..... . .. ................................... Di agnóstico . . ..... ...... ... Tratamiento ....

9

8. 1. 8.2. 8.3.

9 9 10 10

12 12 12 12 12

8.4. 8.5. 8.6.

Definición y conceptos ............................................................ ............................ Etiología ...... .. ...................................................... .. .............. Gardnerella vaginalis (vaginosis bacteriana) ............................................... . .............. Candidiasis ................................................................................................................................ Tricomonia sis .................................. . . ....................................................... Infecciones virales .........................................................................................................

09. Infecciones pélvicas 9.1 . 9.2.

26 26 26 27 27 27 28

30

Enfe rm edad inflamatori a pélvica ................................................... 30 Tuberculosis genital .................... .. ............... 3 1

, Ginecología y obstetricia

1 O. Enfermedades de la vulva ...

33

10.1. Trastornos epiteliales no neoplásicos

................. 33

10.2. 10.3.

Liquen escleroso..... Hiperplasia de células escamosas .....

.............. .. 33 ........... ........ 33

10.4.

Neoplasia vulvar intraepitelial .. ................................................... 33

10.5.

Enfermedad de Paget de la vulva ............................................ 34

1

1n d i Ce

15. Patología del cuerpo uterino y endometria 15.1.

48

Mioma ....

. ........................... 48

15.2. Pólipos endometriales . 15.3.

.

Hiperplasia endometrial.. .

49

m

.......................................... ...... 50

16. Cáncer de endometrio . . 11. Cáncer de vulva. ... 11.1 . 11.2. 11.3. 11.4. 11.5. 11 .6.

35

. .. ............................................ Epidemiología .... .... .............. Factores de riesgo ........ .. . .. Clínica . ... Diagnóstico ... . ...................................................................................................... Extensión .... Estadificación ................................................................

11 .7. Pronóstico ..... 11.8. Tratamiento .........

35 35

16.1. Epidemiología 16.2. Factores de riesgo..... 16.3. Tipos histológicos

52

................... ....................................................... 52

.. ...... ............... . ..................................... 52 .

m

16.4. Clínica . 16.5. Estadificación ....

••••••••••••••••••••••••••••••••••••••••••••••••••••••••••••••

35 35 36 36

16.6. Diagnóstico 16.7. Pronóstico .. 16.8. Tratamiento



••••••••••••



•m



52 53

. ..................................... 53 ............................................................................................................... 53 .......................................................................................... 53

.

.

.m.

..

. ......... ...........................................

54

................................. 36 .................................................................... 36

17. Cáncer de ovario 12. Patología del cuello ....

38

56

17.1.

Epidemiología ......................................................................................................................... 56

17.2.

Tumores epiteliales ............................................................... ....................................... 56

12.1.

Biología del epitelio cervical

12.2.

Patología benigna .

.. . ......... ............... 38

17.3. Tumores germinales ................................................ .... .................................. 57 17.4. Tumores de los cordones sexuales-estroma ........ 57

12.3.

Lesiones cervicales intraepiteliales ............... . ..................................................................... ............ 38

17.5. Tumores metastásicos ................................................................................. ...... 58

m

m



38



13. Carcinoma invasor de cuello ...

41

17.6. Tumores del mesénquima sexual mente indiferenciado .................................. ...................................................................................... 58 17.7. Clínica ........................................................................................................................................ ............ 58 17.8. Estadificación ......... 58 m

13.1.

Epidemiología . ....................................................................................................................... 4 1

13.2. Tipos histológicos . ............................................................................. .................... 42 13.3. Clínica ........................................................................................................................................... 42 13.4. 13.5. 13.6. 13.7. 13.8.

Profila xis y diagnóstico precoz del cáncer de cérvi x .................................................................................................. Propagación .............................................................................................................................. Estadificación ...................................................................................................................... Pronóstico .................................................................................................................................. Tratamiento .................................................................................................................................

14. Suelo pélvico

42 42





m

•• • • •• • •••••••••• • • • • • ••••••••••••• •• ••••••

17.1 O. Vías de diseminación .. ..............

.

m

59

17.11 . Tratamiento ............................................................................................................................ 59 17.12. Seguimiento .. ..................... ........................................................................................... 60 17.13. Diagnóstico precoz ................................................................................................. 60 17.14. Patología benigna de ovario .................................................................... 60

43 43

14.1. Prolapso genital .................................................................................................................. 45 14.2. Incontinencia urinaria . 46 •

••••••••••••••••••••••••• • m . .

... ....................................................................................................... 58

43

45 m

17.9. Diagnóstico

18. Patología benigna de la mama.

62

18.1. Trastornos funcionales ........................................................................................ 62 18.2. Trastornos inflamatorios ................................................................................. 62 18.3. Mastopatía fibroquística (MFQ) o displasia mamaria ................................................................................................... 62 18.4. Tumores benignos de la mama ... ·m··· .. 63

,

1n d i C e

1

Ginecología y obstetricia

19. Cáncer de mama ...

66

19.1 . Epidemiología ....................................................................................................................... 66 19.2. Factores de riesgo ..................... ................................................................................. 66 19.3. Diagnóstico 66 m

19.4. 19.5. 19.6. 19.7. 19.8.

m

Clín ica .................................................................................................................................... Vías de diseminación Estadificación ......................................................................................................................... Factores de mal pronóstico .................................................................. .. Tratamiento del carcinoma de mama infiltrante .............. ..... . .. Formas clínicas especiales . . . ... m

m

m

19.9.

m



••



••

20. Menopausia y climaterio .... 20.1. 20.2. 20.3. 20.4.

68

25. Hemorragias del tercer trimestre 25.1. 25.2. 25.3 . 25.4.

Placenta previa . Abruptio placen toe . Rotura de vas a previa. ... . Rotura uterina .. .... .. .

99 99

oo

1

.. ..

. ..

..

.m

101

.. 101

68 68 69

26. Alteraciones de los anejos ovulares

.

103

69 70

26.1. 26.2. 26.3.

Patología del cordón umbilical ... .................. . .......... 103 Alteraciones de la placenta ......................................................................... 103 Patología del líquido amniótico . ........................ ....... 104

73

Clínica ................................................................................................................................................ . Modificaciones endocrinas en la menopausia . . Diagnóstico ................................................................................................................................. Tratamiento .............. ..... . ...... . m

••



.....

21. Fisiología del embarazo ....

73 73 74 74

76

21.1. Fecundación e implantación ........................................................ ......... 76 21 .2. Placenta .................................................................... .............. ...................................................... 76 21.3. Modificaciones gravídicas maternas ............................ . ........ 78

27.1 . Clasificación 27.2. Incidencia

.

. .

106

27.3. Factores etiológicos .................................................................................................... 106 27.4. Patología asociada a la gestación gemelar ................................................................... 107 27.5. Diagnóstico ........................................................................................................................... 107 27.6. Conducta obstétrica 107 m

28. Parto pretérmino 22. Control del embarazo normaL

81

.

............ 109

28.1 . Etiología .............................................................................................................................................. 109 28.2. Diagnóstico . . 109 28.3.

Conducta obstétrica . ..................... .............

28.4. Tocólisis ..............

23. Evaluación gestacionaL

106

······m············································

.....

..................... ...11

o

m .................................................. 110

83

23.1 . Diagnóstico de gestación ........................................ ... .... 23.2. Ecografía obstétrica ................................................................. ...................... 23.3. Métodos de diagnóstico prenatal de cromosomopatías ............................................................. 23.4. Evaluación del bienestar fetal en el tercer trimestre ................................................................................................

24. Hemorragias del primer trimestre

83 83 85

86

90

24.1 . Aborto ..... . .......... ........ ........................................... 90 24.2. Gestación ectópica . ................... 92 24.3 . Enfermedad trofob lástica ..................................................................... 94 m

29 . Gestación cronológicamente prolongada . . . . 29.1. 29.2. 29.3. 29.4. 29.5.

..

112

Etiología .............. ................................................. .. ................................... . 112 Clínica . 112 Diagnóstico ......................... . .............................................................................. 112 Valoración y tratamiento ............................................................................... 11 2 Inducción . 11 2 m

•••••••••••••••••••••••••••••••••••••••••••••• •

• • •• • •••••••••••••••••••••••• • ••••••••••••••••• • • • ••••••••

, Ginecología y obstetricia

30. Elementos de tocología 30.1. 30.2. 30.3. 30.4. 30.5 . 30.6.

.... ... .......... 115

Canal del parto...... ........................... 115 Elementos fetales .. ....................................... 115 Estática fetaL . . .. . . m . 115 Condiciones generales del parto ..... ............... 116 Parto instrumental . .... ........................................................... 117 Parto en presentación pelviana ......................................................... 118

31. Posparto y puerperio ... 31.1. 31 .2. 31 .3. 31.4. 31.5.

. ...... 120

Hemorragia posparto ....... Inversión uterina .... ............................................. 121 Infección posparto y puerperaL.... . ..... 121 Inhibición de la lactancia . . .......... 121 . ..................... 122 Otros problemas del puerperio ....

1n d i C e

1

34. Complicaciones infecciosas ...

. .... 132

. .......................................... 132 34.1. Vacunaciones ... .. .................................................................. 132 34.2. Toxoplasmosis ... . ......................... 132 34.3. Rubéola .. 34.4. Citomegalovirus ..... ........................... 133 34.5. Sífilis . m m 133 34.6. Tuberculosis y embarazo.... . ....................... 133

34.7. 34.8. 34.9. 34.1 O. 34.11 . 34.12.

Varicela .... • •••••••• • • • • m 133 .................................... 134 Hepatitis B y hepatitis C Estreptococo B..... . ..... ............. m 134 Virus del papiloma humano ..... . ..................... 134 VIHyembarazo ..... .......... 134 Herpes simple ..... . .... . 135

35. Fármacos y embarazo . . 35.1.

137

H

Efectos de los fármacos sobre el feto ... .

.. .

m137

32. Estados hipertensivos del embarazo . . 32.1. 32.2.

......

Epidemiología y etiología .... Fisiopatología y manifestaciones clínicas ........................................... 32.3. Clasificación . ....... . 32.4. Definiciones .................................................... ............... 32.5. Tratamiento .... 32.6. Parto .....

33. Diabetes gestaciona 33.1. 33.2. 33.3. 33.4. 33.5. 33.6. 33.7.

H



124

... ................................. 124 ....................... 124 . m 125 m

125

. ................................ 125 .................. 126

128

Diabetes y gestación ....... ....................................128 Efecto diabetogénico del embarazo ............. ........ 128 Morbilidad materna . . ............................................................................... 128 Mortalidad perinatal .. . . m 128 Morbilidad fetal .................................................................................................... 129 Diagnóstico ..... . .................................................................. 129 Control durante la gestación ................................................................ 130

36. Otras patologías de la gestante...

H

139

36.1. 36.2.

Hiperemesis gravídica . . . .. .... m 139 Ictericia recu rrente del embarazo o colestasis intrahepática gestacional .............................. 139 36.3. Hígado graso agudo del embarazo o esteatosis hepática aguda gravídica .... .. ............... 140 36.4. Dermatosis del embarazo . . ................................. 140 36.5. Nefropatía gravídica . . .. .... ...... 140 36.6. Cardiopatías y embarazo m. 140 ................ 141 36.7. Epilepsia y embarazo ... ........................ ................... 141 36.8. Gestante Rh negativo ....

Vídeos.

Bibliografía .

_H

_____________

148

- - - - - - - - - 150

Ginecología y obstetricia

1

Ciclo genital femenino

ORIENTACIÓN

ENARM

Este tema, es fundamental para entender la patología ginecológica y su tratamiento.

El ciclo genital femenino tiene una duración media de 28 días, aunque se considera normal que esté entre 21 y 35 días. Se divide en tres fases: hemorrágica o menstrual, proliferativa o folicular y secretora o lútea.

LH inducido por el incremento de los niveles de estradiol, que provocaría una ovulación espontánea con la cancelación del ciclo. La dopamina, que es el factor inhibidor de la prolactina, inhibe la producción de GnRH.

Análogos de GnRH se utilizan en el tratamiento de la endometrios is, los m iomas, la pubertad precoz y la estimulación ovárica (técnicas de reproducción asistida).

.,.._ 1

_,_fl"

El día que comienza el sangrado menstrual se considera el día 1 del ciclo. Entre los días 1-3, tiene lugar la menstruación o fase hemorrágica. A partir del día 4, comienza la fase proliferativa, que durará hasta la ovulación, el

1.2. Hipófisis

día 14 del ciclo. Por tanto, la fase prol iferativa abarca desde el día 4 al 14. La ovulación ocurre el día 14, y a partir de ahí comienza la fase secretora, que terminará cuando se inicie la fase hemorrágica del sigu iente ciclo,

Cuando la GnRH llega a la hipófisis anterior (adenoh ipófisis) estimula la

aproximadamente del día 14 al 28.

síntesis, almacenamiento y secreción de FSH y LH (hay que recordar que la FSH y LH comparten la subun idad a con TSH y HCG).

1.1. Hipotálamo

FSH

El hipotálamo produce GnRH (hormona reguladora de la secreción de

libera en la primera mitad de la fase proliferativa, y tiene como m isión el

las gonadotropinas FSH y LH). Esta GnRHhipotalámica estimula en la hi-

crecimiento de la cohorte folicular y la selección de l folículo dominante.

La liberación de FSH tiene dos fases: una primera meseta, pequeña, se

pófisis la producción de las gonadotropinas (LH y FSH). La liberación se produce de manera pulsátil, de tal fo rma que los pulsos lentos sobrees-

La segunda fase o segundo pico sucede justo antes de la ovulación. Sus

timulan FSH y los rápidos sobreestimulan LH (como ocurre en el síndro-

acciones son las siguientes:

me del ovario poliquístico). Los ciclos menstruales normales requieren el

Estimula el crecimiento de la capa granu losa en el folículo que ha

mantenimiento de la liberación pulsátil de GnRH dentro de un intervalo

seleccionado.

crítico de frecuencia y de amplitud.

Induce la actividad aromatasa en la granu losa, que convierte losandrógenos en estradiol (por tanto, la FSH estimula la producción de

Esta actividad rítmica pulsátil es una propiedad intrínseca de las neuronas

estrógenos en el folículo ovárico).

productoras de GnRH y el efecto de diversas hormonas y neurotransmi-

Aumenta los receptores de FSH en la granulosa. La FSH es inhibida por

sores de menor importancia. La liberación continua de GnRH desensibi-

la inhibina folicular y los estrógenos. Es deci r, los estrógenos producidos

liza las células por internal ización de sus receptores, inhibiendo la FSH

gracias a la FSH inhiben la propia FSH mediante un feedback negativo.

y la LH, y se provoca un estado de hipoestrogenismo. Esa situación de hipoestrogenismo inducida por los análogos de la GnRH los hace útiles

LH

en el tratamiento de la endometriosis, los miomas uterinos, la pubertad precoz y en la estimulación ovárica.

Su liberación tiene un solo pico, el pico ovulatorio, consecuencia del "efecto gati llo" de los estrógenos. Así pues, la ovulación es el resultado

Los análogos de la GnRH se vienen usando de forma frecuente en los tratamientas de reproducción asistida, con el fin de evitar el pico endógeno de

directo de este pico de LH (sin él, no hay ovulación) Sus acciones son las que se enumeran a continuación:

Manual CTO de Medicina y Ci rugía, 2.8 edición

Estimu la el crecimiento de la teca, q ue produce andrógenos (por

Fase lútea

tanto, la LH estimula la producció n ovárica de andrógenos). Favorece la luteinización del folícu lo tras la ovu lación.

Tras la ovulación, el folículo se co lapsa y se co nvierte en cuerpo lúteo. Es una fase de duración fija : 13-15 días. Hay que reco rdar que al final comienza ya a elevarse algo la FSH.

1.3. Ovario Cuerpo lúteo: es el lugar de producción de progesterona. También sin-

tetiza otras sustancias, como pequeñas cantidades de estrógenos. Es esEl ovario contiene unos 500.000 fol ícu los primordiales en la puber-

timulado por LH y HCG.

tad, de los que sólo 400 llegarán a ovular. A continuación, se estud ian los cambios que se p roducen en el ovario en las diferentes fases de l

Luteólisis y menstruación

ciclo. Los estrógenos inducen a la luteólisis. Para ello, aumentan la concentra-

Fase folicular

ción de prostrag landina F, q ue inhibe la síntesis de progesterona y la capacidad de unión de la LH a su receptor.

La FSH estimula en el ovario el crec im iento de la cohorte de folícu los primordiales seleccionados. La bajada de FSH selecciona el folícu lo dominante -que es aquel con mayor capacidad de respuesta a FSH- y la

1.4. Andrógenos

atresia simultánea del resto por exceso local de andrógenos. En el fo lícu lo eleg ido, se distinguen dos capas importantes (Figura 1): Teca: su desarrollo depende de LH. Produce andrógenos, que son

La LH estimula la teca para que produzca andrógenos. Estos andrógenos

aportados a la granulosa.

son usados por la aromatasa de la granulosa y del cuerpo lúteo para la pro-

Granulosa: su desarrollo está en función de la FSH y del ambiente

ducción de estrógenos. En cambio, los andrógenos en dosis excesivamente

estrogén ico (tiene receptores de FSH). Contiene·aromatasa, que em-

altas ejercen el efecto contrario: inh iben la aromatasa y producen at resia del

plea los and rógenos de la teca para producir estradiol. No obstante,

folículo, disminuyendo así la producción de estrógenos. Dentro de los andró-

si los andrógenos son excesivos (ambiente androgénico), se atresia.

genos naturales, el más importante es la testosterona, aunque su derivado,

Genera inhi bina q ue inh ibe la FSH .

la dihidrotestosterona, es más potente desde el punto de vista biológico. Como andrógeno natural de origen suprarrenal, está la dehidroepiandrosterona y, con origen mixto gonadal y suprarrenal, la androstendiona) (Tabla 1).

Andrógenos

Valores

Testosterona

180-580 pg/ml

Testosterona libre

1,2-2,2%

DHEAS

2,3 JJQ/ml

DHEA

4,2 JJQ/ml

Androstendiona

< 1,5 ng/ml

Tabla 1. Principales andrógenos de la mujer

Andrógeno princ ipal en la mujer: testosterona (origen ová ri co).

Figura 1. Capa granulosa y teca en el folículo de De Graaf

1.5. Estrógenos Ovulación Tiene lugar como consecuencia directa del pico de LH. Aparece el día

La liberación de los estrógenos es bimodal: crecen hasta el pico preovulato-

14 del ciclo (aunque puede variar entre el 11 y el 23). El pico de estrad iol

rio, es decir, entre 24-36 h antes de la ovulación, y tienen otro pico menor en

"dispara" el pico de LH, y este pico de LH provoca, 10-12 horas después,

la fase lútea. Son tráficos para todo el aparato genital. Localmente, inducen

la ovulación .

la síntesis y la expresión a los receptores de FSH. Niveles bajos y moderados inhiben la FSH, y altos tienen "efecto gatillo': disparando la producción de LH.

Ovocito: es ovocito primario en profase de la 1a meiosis hasta la

Proceden de la aromatización de los andrógenos en la granulosa y también se

pubertad. Con la ovulación, se comp leta la 1.• meiosis y pasa a ser

producen en el cuerpo lúteo. El estrógeno sintetizado por el ovario de forma

ovoc ito secundario hasta la fecundación, que estimula la 2• d ivisión

más activa e importante es el estradiol. Los estrógenos estimulan el crecimien-

meiótica.

to y la proliferación de los órganos sexuales femeninos y bloquean la PRL.

01 · Ciclo gen ital femenino

Ginecología y obstetricia El estrógeno principal en la mujer fértil es el estrad iol; en el clim ateri o, la estrona (tamb ién en el síndrome de ovari o poliqu ísti co) .

2.

_...,...,...,."!:""'"'"

1

01

Fase secretora (después de la ovulación): tiene lugar la maduración de las glándu las y el estroma endometrial, debido a la producción de

progesterona y también de estrógenos.

1.8. Modelo fisiológico: integración del ciclo

1.6. Progestágenos La liberación de progestágenos es unimodal : tiene un ún ico pico en la fase secretora que alcanza el nivel máximo ocho días tras el pico de LH. Se fabrican en el cuerpo lúteo. Su misión es la maduración del endometrio (fenó-

El primer día del sangrad o menstrual es el d ía de comienzo del ciclo. En esos momentos, la secreción pu lsáti l de GnRH en el hipotálamo estimula en la hipófisis la producción de FSH, q ue actúa en el ovario

meno que define la fase secretora) y producen mod ificaciones en todo el aparato genital, de forma que lo adecuan a la gestación (de ahí su denomi-

estimulando el crecimiento d e un grupo de folículos.

nación: "pro-gestágenos'?. Prepa ran las ma mas para la lactancia. Deprimen la excitabilidad de las fib ra s miometri ales, puesto que las

La capa g ranu losa d e est os folícu los va a t ransfo rma r los and rógenos en estrad io l por m ed io de la arom atasa. Este estrad io l llega al útero y ocasiona el crecimiento de l endometri o. Además, el estradiol, junto a la inhibina, provocará un descenso de FSH (Figura 2). Esta d isminu-

contracciones uterinas impedirían la gestación. También re lajan el músculo liso digestivo y uretera l. Eleva n el metabolismo y la temperat ura corporal: hasta el día 14, la A partir de la ovulación, la tempetemperatura es menor de 36,9

oc.

ción de FSH va a ocasiona r la selección de un protagon ista entre el grupo de folícu los que estaban creciendo: es el fo lículo dominante, y el resto se atresia . Este fo lículo fue se leccionado porque presentaba

ratura sube por encima de 37 oe, debido a la progesterona. Disminuyen la cantidad de moco cervical y su contenido en ácido

mayor cantidad de aromatización y de receptores para FSH.

siálico, aumenta ndo su viscosidad. Al favorece r que el moco sea escaso y viscoso, dificu lta el paso de nuevos espermat ozoides.

Una vez eleg ido, comienza a produci r estrógenos y es capaz de elevar, él solo, los niveles sisté mi cos de est rógenos. Esta elevación estrogénica va a produc ir un p ico de FSH y, poco después, un p ico de LH; y este p ico de LH da lugar a la ovu lación el d ía 14 de l ciclo. A part ir de

Por ta nto, en la fase prolife rativa o preovulatoria, se segregan fundamenta lmente estrógenos, mientras que en el periodo postovulatorio o secretor, se producen grandes ca ntidades de progesterona y t ambién de estrógenos.

la ovulación, el folícu lo sufre u na transformación gracias a la LH y se convierte en cuerpo lút eo, cuya m isión es estab lecer las cond iciones que favorezcan la gestac ión, para ello, p rod uce progestero na.

1.7. Endometrio uterino

Parte de esta progesterona gene rada por el cuerpo lúteo se transforma en andrógenos y en estrógenos, lo que motiva el pico de estrógenos y de progesterona en la mitad de la fase secretora del ciclo. Si no

A esca la endometria l se observa n dos fases : l . Fase proliferativa (a ntes de la ovu lación): se produce un crecimiento glandula r en el endometrio uterino, provocado por el estímulo

t iene luga r la fecundac ión, se produce la luteól isis y la menstru ación, pero antes de q ue final ice la fase secretora, ya comie nza a aumentar la FSH, que esti mu lará el crec imie nto de un nuevo grupo de fo lícu los en el sigu iente ciclo.

estrogénico.

75

.. ~

,.....____..... ~ 3 7°( ~ . Temperatura corporal ba:aJ

Fase Fa se de desarrollo degenerativa

25 o ~--------------------~~~---FSH

16

8

Folículo 1.0

o -~----

16

&

o Folículo 3.0

Folículo 2°

f

Cuerpo lúteo

Ovulación

· Progesterona

12

8 4

o ~====~==~------------~=--

~~ ~_..,-,~.. _, Día 5 Figura 2. Ciclo menstrual

Día 14

Día 21

Día 28 Día 5

Día 1

Día 5 Fase proliferativa

Día 14 Fase secretora

Día 28 Día 21 Fase isquémica

Manual CTO de Medicina y Cirugía, 2. a edición

Ideas clave J?S "

"

El control del ciclo ovárico por parte del hipotálamo se realiza a través de la secreción pulsátil de GnRH. Los pulsos rápidos estimulan la LH y los lentos la FSH . Sin embargo, la liberación continua, como ocurre con la administración de los análogos de la GnRH, desensibiliza las células, provocando una situación de hipoestrogenismo útil en el tratamiento de la endometriosis, miomas uterinos, pubertad precoz, estimulación ovárica ... La FSH estimula el crecimiento de la capa de la granulosa, induce a actividad aromatasa en dicha capa y convierte los andrógenos en estradiol.

01 · Ciclo genital femenino

"

La LH estimula el crecimiento de la capa de la teca y, por tanto, produce andrógenos. También se encarga de la luteinización del folículo tras inducir la ovulación.

"

Los andrógenos son usados por la aromatasa de la granulosa y del cuerpo lúteo para la producción de estrógenos. Sin embargo, los niveles excesivamente altos producen una inhibición de esta enzima y, por consiguiente, atresia folicular.

"

La progesterona se produce únicamente en el cuerpo lúteo. Sus funciones son: maduración endometrial, disminución de la excitabilidad del músculo liso miometrial, elevación del metabolismo basal y disminución de la cantidad de moco cervical y contenido en ácido siálico.

____G_in_e_c_oly obst et ri eia



Amen arreas

ORIENTACIÓN

ENARM

Se debe atender al apartado del diagnóstico de las amenorreas secundarias.

Pterigium colli

valgus

En la embriogénesis, la ausencia del cromosoma Y permite el desarrollo mülleriano y, por tanto, la formación de gen itales internos femeninos (por eso, un cariotipo 45XO se desarrollará hacia femenino). La ausencia de andrógenos a su vez posibili ta el desarrollo de los genitales externos femeninos. Por eso, un cariotipo XX, pero con exceso de andrógenos, se desarrolla hacia mascu li no. La presencia de andrógenos en la pubertad desarrolla el vello axilar y el pubiano. La causa global más frecuente de amenorrea es la amenorrea fis iológica del embarazo.

2.1. Amenorreas primarias Figura 3. Síndro me de Turn er

Clasificación etiológica

Disgenesia gonadal mixta: el mosa icismo que afecta al cromosoma Y puede asoc iarse a anoma lías d e la diferencia-

Anomalías genitales

ción sexual. Entre los distintos cariotipos posibles, el ca ri otipo 45X0/46XY es el más frecuente . Estos individuos presentan

Disgenesia gonadal: consiste en la formación defectuosa de los

m uy diversos fenotipos, desde recién nacidos con genitales

ovarios, sustit uidos por dos cintil las fibrosas con ausencia de fo lícu-

ambiguos hasta varones férti les normales o fenotipos feme-

los ováricos. Los genitales externos son femen in os, pero infantiles.

ninos normales con gónadas acintadas b ilatera les. La mayoría

Muestra niveles elevados de gonadotropinas, ya que no se prod ucen

tiene ta lla baja, y la tercera parte presenta otros estigmas del

las hormonas que llevan a cabo el feedback negativo. Se presenta

síndrome de Turner.

bajo estas formas:

Síndrome de Turner: los cariotipos son 45XO, 46XX y mosa i-

Síndrome de Rokitansky: en este síndrome, lo fundamental es una

cos que incluyan ambas situaciones. Estos individuos presentan

alteración en la permeabilización de los conductos de Müller. El fe-

talla baja y frecuentes malformaciones extragenitales: pliegue

notipo es femenino norma l. La cromatina sexual es positiva. El cario-

cervical (pterigium col/i), cubitus valgus, alteraciones renales, car-

tipo también es femenino normal: 46XX. Los ovarios son normales. El

díacas (los cariotipos 45XO, coartación aórtica), etc. (Figura 3).

útero es rudimentario y no está cana lizado. Hay agenesia de los 2/ 3

Son causa de abortos, y a veces se detectan en vida embrionaria

superiores de la vagina, por lo que, en la inspección, se aprecia una

por presentar higromas quísti cos, que son tumoraciones linfáti-

vagina corta que termina en fondo de saco ciego. Presenta frecuen-

cas visibles en la ecog rafía desde el primer trimestre.

tes malformaciones renales o urinarias asociadas.

Síndrome de Swyer: es una disgenesia gonada l pura, sin mal-

Himen imperforado: el d iagnóstico se basa en la exploración genital que debe realizarse a toda ni ña prepúber en la primera visita al

formaciones asociadas ni ena nismo. El cariotipo es 46XY pero el cromosoma Y no se expresa correctament e, por lo que func iona

pediatra. El acúmulo menstrual reten ido puede produci r dolor ab-

como un 45XO. Son frecue ntes los cá nceres de ovario: el más

dom inal. Cu ra con la incisión y la evacuac ión del conten ido vaginal (Figura 4).

habitua l es el gonadoblastoma.

Manual CTO de Medicina y Cirugía, 2 .a ed ición

Lesiones hipotálamo-hipofisarias: tumores, traumatismos, hematomas, infartos, granulomas, etcétera, lesionan el eje hipotálamohipofisario e impiden el normal funcionamiento del ciclo menstrual. Pubertad retrasada. Hipogonadismo hipogonadotrópico. Síndromes neurogerminales: Síndrome de Kallman: ocurre una detención en el crecimiento del SNC desde las primeras semanas de vida intrauterina, con defecto de la línea media. Cursa con atrofia del bulbo olfatorio e infantilismo sexual. Se produce amenorrea primaria acompañada de profundas alteraciones del olfato. Las gonadotropinas están descend idas. El cariotipo puede ser femenino o masculino. Figura 4. Amenorrea primaria por alteraciones müllerianas: sinequia congénita de labios menores con himen imperforado

Síndrome de Laurence-Moon -Bield: asocia diabetes, oligofrenia e hipogonadismo. Sínd rome de Alstrom: cursa con retinitis pigmentaria, sordera, nefropatía e hipogonad ismo.

Fem inización t esticular, síndrom e de Morris o pseudohermafro-

Progeria: asocia calvicie prematura, cabe llo grisáceo, cataratas,

ditismo masculino: el cariotipo es mascul ino: 46XY. Los testículos

atrofia muscular y del tejido cutáneo. Lleva a la muerte en fases tempranas de la vida.

están bien conformados, aunque suelen ser intraabdominales, con riesgo de degeneración en un disgerminoma. Los niveles de testosterona son los normales en el hombre, pero hay un déficit de los receptores intranucleares androgénicos, lo que impide la actuación

Síndrome de Prader-Willi: cursa con hipotonía, hipogonadismo, hipomentia

y obesidad (síndrome HHHO). Responden bien

al tratamiento con clomifeno.

de los andrógenos y provoca que dichos individuos tengan fenotipo femenino normal, pero con ausencia de vello axilar y pubiano. Hiperplasia suprarrenal congénita, síndrome adrenogenital o pseudohermafroditismo femenino: se caracteriza por la presencia de genitales externos mascu linizados

2.2. Amenorreas secundarias

y se diagnostica mediante

la demostración de una producción de andrógenos excesiva por la corteza suprarrenal. Puede aparecer durante la vida intrauterina o desarrollarse posnatalmente, dando lugar a la virilización de los genitales externos (este síndrome es, en cierto modo, lo contrario del síndrome

La amenorrea secundaria se define como la falta de menstruación durante al menos tres meses en una mujer que previamente ha tenido la reg la. Entre las causas que la producen, están las siguientes:

de Morris). Este trastorno es el resultado de la deficiencia de una de las

Origen uterino: síndrome de Asherman (sinequias uterinas tras legrados).

mú ltiples enzimas que son necesarias para la síntesis del cortisol suprarrena l. El cariotipo es normal (46XX). La clínica varía según el déficit

Insuficiencia ovárica: también llamado fallo ovárico prematuro (FOP) o menopausia precoz. Consiste en un agotamiento folicu lar

enzimático (HTA e hipoca liemia en el déficit de 17-a-hidroxilasa; viril i-

y síndrome pierde-sa l en el déficit de 21-h idroxilasa). El déficit

antes de los 40 años, lo que provoca un descenso de estrógenos y, por tanto, una elevación de gonadotropinas. Se incluyen en este

más frecuente es el de 21-h idroxi lasa. Agenesia de vagina: es poco frecuente. Se detecta en la exploración.

grupo el síndrome del ovario res istente (tras rad iación o cirugía), en el cual se produce una elevación de las gonadotropinas, ya que el

zación

Amenorrea por anorexia o deportiva

ovario es res istente a ellas, a pesar de existir folículos ováricos no est imulados. La etiología del fallo ovárico prematuro es desconocida

Anorexia nerviosa: el 25% de las mujeres anoréxicas desarrollan

en muchas ocasiones pero se han descrito causas genéticas, autoinmunitarias, etcétera.

amenorrea antes de que haya ocurrido una pérd ida importante de peso. Cursa con gonadotropinas d ism inu idas. La amenorrea se corri-

Tumores ováricos: en tumores grandes se puede ocasionar una destrucción tota l del tej ido ová ri co sano. Por tanto, no se produce

ge con la gana ncia de peso. Amenorrea deportiva: hasta la m itad de las m ujeres que practican ejercicio intenso y competitivo (ballet, gim nasia ... ) pueden presentar

ovu lación

amenorrea. Entre las causas que provoca n esta amenorrea destacan las siguientes: dism inución de peso y del porcentaj e de grasa corporal, aumento de esteroides sexua les, incremento de andrógenos y de prolactina. También aumenta la temperatu ra corporal y hay elevación de hormona del crecimiento, ACTH, ~-endorfinas y ~-l i potropina, de forma que alterarían el patrón de descarga hipota lámica de GnRH.

Causas centrales

y desapa recen las reg las.

Hipogonadismo hipogonadotropo: la más frecuente es la amenorrea hipotalám ica funciona l por ejercicio físico, anorexia nerviosa u otros trastornos psíqu icos. Hiperprolactinemia: todas aquellas causas que la provocan, tanto tumorales (prolactinomas) como no tumorales (traumatismos). Síndrome de Sheehan: amenorrea posparto por infarto hipofisario. Constituye el motivo más frecuente de panh ipopituitarismo en mujeres en edad rep roductiva. Se caracteriza por una incapacidad para la lactancia materna con involución de la glándu la mamaria. Después aparece amenorrea (consecuencia de la anovulación debido al cese de producción hipofisiaria de FSH y LH por necrosis isquémica

como secundarias, probablemente por la li beración de corticotropi-

de la glándula) y la pérdida del vello pubiano y axilar. Otros síntomas son: asten ia, inapetencia, intolerancia al frío, mixedema, pérdida de pigmentación de las aréolas mamarias y de la región genital y pérdi-

na (CRH), que inhibe la secreción de gonadotropinas.

da de peso que puede lleva r a la caquexia.

Amenorrea psíquica: el estrés, el internamiento, el miedo al embarazo o a la sexua lidad pueden producir amenorreas tanto primarias

02 · Amenorreas

Ginecología y obstetricia Tumores hi pofisa ri os secret ores de hormonas prot eicas: como GH (acromegalia), TSH, gonadotropinas, y ACTH y adenomas no secretores que no se man ifiestan clínica mente hasta que no alcanzan gran tamaño (macroadenomas). Craneofa rin gioma: el 60% de los casos presenta amenorrea por la compresión hipofisaria directa de la propia glándula o del sistema vascular que conecta el hipotála mo a la hipófisis. Fármacos: anovulatorios, fenotiacinas, reserpina, digoxina, etcétera. Enfermedades int ercu rrent es: insuficiencia ren al, diabetes.

1

02

da secuencia hormona l, algo fal la desde el punto de vista anatómico en el útero (síndrome de Asherman), en el cérvix o en la vagina (estenosis). Si tiene la regla, su endometri o funciona y la menstruación tiene paso libre hacia el exte ri or. Aho ra se ha acotado el problema : si útero, cérvix y vagina responden a la secuencia hormonal, el problema se encuentra más arriba : el ovario o el eje hi potá lamo-h ipófis is están fallando. Para dife renc iarlo, se determinan gonadotropinas hipofisarias. Si están elevadas (3-4 veces la cifra normal, que es 1O), se sospecha disfunción ovárica, ya que el eje hipotálamo-hipófisis funciona. Si son

Amenorreas psíquicas: anorexia nerviosa, pseudociesi s ("emba razo psicológico") o estrés. De origen supra rrenal o t iroideo: tanto el exceso como el defec-

baja s, el problema no está en el ovario, sino en el eje. Para diferenciar si el fa ll o está en el hipotá lamo o en la hipófi sis, se admini stra GnRH si aumenta n las gonadotropinas, está sana la hipófisis y se sospecha

to de esteroid es o de hormonas tiroideas pueden provocar amenorrea.

alteración hi pota lámica. Si la hipófis is no responde a la GnR H, la causa es hipofisa ri a (Figura S).

2.3. Diagnóstico de la amenorrea

Prueba de embarazo

-----+-

posit iva

-------+-

¡ negativa

¡

Ante una amenorrea, lo primero que hay que descartar es una gestación, y para el lo se realiza, en primer lug ar, un test de embarazo. Si es positivo, se pensará en gestación (o, con menor frecuencia, en un coriocarcinoma de ovario productor de HCG) . Si es negativo, se continuará el estudio. Se hará dete rminaci ón de TSH y PRL: si están alteradas, se ll eva rá a cabo un tratamiento etiológico (la elevación de la TSH en el hipotiroidismo y de la PRL son las alterac ion es frec uentemente imp licadas) . Si so n norma les, se cont inuará el estudio. Se da una pequeña cantidad de progesterona (5-1 O mg/día de acetato de medroxiprogeste rona durante cinco días): si la paciente produce con no rmal idad la 1a fase del ciclo, la proliferativa, pero no llega a ovu lar y, por tanto, no ll ega a produci r progesterona, al administrarle esta progesterona que le faltaba, tendrá la regla, y se concluye que la causa era la anovu lación . Si, a pesar de la progesterona, no tiene la regla, se continuará el estudio. Se administra entonces una combinación de estrógenos y progestágenos, durante tres meses. Si no tiene la regla, a pesar de una adecua-

Ideas clave 1!5 " La causa más frecuente de amenorreas primarias son las disgenes ias gonada les. De estas, la más habitual es el síndrome de Turner, que se caracteriza por presentar talla baja y frecuentes malformaciones (pterigión col/i, cubitus va!gus, renales, cardíacas .. .). Su cariotipo puede ser 45XO o 45X0/46XX. "

El síndrome de Swyer es otra disgenesia gonadal pero, a diferencia del de Turner, no presenta enanismo ni malformaciones asociadas. Su cariotipo es 46XY. Es frecuente la asociación con el gonadoblastoma.

"

El síndrome de Rokitansky es una alteración en el desa rrollo de los conductos de Müller, por lo que las pacientes presentan agenesia uterina y de los dos tercios superiores de vagina. Su cariotipo es 46XX. Son frecuentes las ma lformaciones renales.

Determinación TSH y PRL

~ alterados -

¡

Hiperprolactinemia Hipertiroidismo... (tratamiento etiológico)

no rma les .___li_e_st_ d_e..;p_r_o.;:;. ge _s_te_r_o_n a _ _.

------->-

reg la --~ L - - A-n o_ v_u_la_c_ió_n_ __,

¡ no reg la

¡

no regl a -

Alteración anatómica gen it al

_.... elevadas -

A lteració n ovárica

Est ró genos/ progestágenos -

¡ reg la

¡ M ed ición FSH y LH

¡ baj as

¡ Test de Gn RH

~ sub e

\

FSH

~

no sube FSH -

Alteració n hipotalá mica Alteración hipofisaria

Figura S. Diagnóstico de las amenorreas secundarias

"

La causa más frecuente de amenorrea secundaria es la gestación. Por ello, es el primer paso en el diagnóstico de las mujeres en edad fértil que consu ltan por llevar tres meses sin la regla.

"

En el diagnóstico diferencial de las amenorreas, se realizará en primer lugar el test de supresión con gestágenos. Si la pac iente menstrúa, presenta anovulación . Si no regla, se llevará a cabo una estimulación combinada con estrógenos y gestágenos. Si no sangra, hay que descartar las causas orgánicas tales como el síndrome de Asherman (s inequias uterinas).

"

La valoración de los niveles de gonadotropinas permitirá conocer el origen de las alteraciones del eje ovárico. Si están altas, el problema radica a nivel ovárico (por ejemplo, fallo ovárico precoz, resistencia a gonadotropinas ... ). Sin embargo, si están bajas, el problema es hipotalámico o hipofisario.

Manua l CTO de Med icina y Cirugía, 2.a edic ión

Casos clínicos . Una mujer de 18 años consulta por amenorrea primaria. En la exploración se observa un fenotipo femenino normal pero con ausencia de desarrollo de los caracteres sexuales secundarios. Los genitales externos son femeninos de aspecto infantil y los genitales internos femeninos e hipoplásicos. La talla es normal. El cariotipo es 46,XX. Los niveles plasmáticos de gonadotropinas (FSH y LH) están muy elevados. ¿Cuál de los siguientes diagnósticos corresponde a este cuadro?

tras la administración de progestágeno, pero sí lo hace al administrar un estrógeno junto con un progestágeno. ¿Cuál de los diagnósticos que a continuación se relacionan es el más correcto? 1) Síndrome de ovario poliquístico. 2) Fallo ovárico autoinmune. 3) Tumor hipotalámico o hipofisario. 4) Síndrome de déficit congénito de hormona liberadora de gonadotropinas (GnRH) . RC 3

1) Síndrome de insensibilidad a los andrógenos (feminización tes-

ticular). 2) Disgenesia gonadal tipo síndrome de Turner.

3) Disgenesia gonadal pura . 4) Defecto congénito de hormona liberadora de gonadotropinas

(GnRH). RC3

Una mujer de 21 años presenta una amenorrea secundaria. Los niveles plasmáticos de gonadotropinas (LH y FSH) son inferiores a 10 mUI/ml. Los niveles de prolactina y de hormona tiroestimulante (TSH) son normales. La paciente no menstrúa

Case Study Z::= A 24-year-old female consulting for secondary amenorrhea. Physical examination does not show any relevant information. The patient does not have menstruation after medroxyprogesterone acetate administration, 1O mg per day for 1O days, but has her menstruation after conjugated equine estrogens and progestogens administration. Prolactin and TSH levels are nor-

02 · Amenorreas

Mujer de 19 años que consulta por llevar 6 meses sin regla. Se manifiesta preocupada por su sobrepeso, ha estado a dieta y ha perdido S kg en 8 meses. Actualmente pesa 47 kg (mide 1,65). El test de gestación es negativo. ¿Cuál es la causa más probable de su amenorrea? 1) 2) 3) 4)

Disgenesia gonadal. Hipogonadismo hipogonadotropo. Síndrome de ovario poliquístico. Himen imperfo rado.

RC 2

mal, FSH and LH levels are lower than 1O IU/mL. The most likely diagnosis is: 1) Polycystic ovary syndrome. 2) Premature ovarían fa ilure. 3) Gonadal dysgenesis. 4) Craniopharyngioma.

Correct answer: 4

Ginecnlogía y obste_trlcia__ _ .. •

Síndrome de ovar1o poliquístico

(SOP)

ORIEf~TACION

ENARM

Tema que es importante que se estudie en detalle.

3.1. Concepto

Además, los andrógenos circulantes son convertidos a estrona en la grasa periférica (aumentada por la situación de obesidad en relación con el exceso androgénico). La insulina estimula la actividad aromatasa en las células de la granu losa, convirtiendo los andrógenos de la teca en estrógenos. Pues bien, en las mujeres con SOP hay una insulinorresistencia (clave en la etiopatogenia), que contribuye al aumento de andrógenos al no poder ser estos transformados en estrógenos al nivel de la granulosa. La obesidad agrava el grado de resistencia a la insu lina que pueda

El síndrome de ovario poliquístico (SOP) es una afección muy frecuente, aunque de etiología desconocida, con una incidencia aproximada del

existir, y constituye un factor importante de mantenimiento de la anovu lación crónica. Es conveniente recordar el papel preponderan-

3-7% de las mujeres en edad reproductiva. Es un cuad ro clínico caracterizado por: Clínica: aparecen con diferente frecuencia síntomas como: anovulación/esterilidad, obesidad, hirsutismo/androgenización.

te de la hiperinsulinemia en el denominado síndrome metabólico ca racterizado por: resistencia a la insulina, obesidad, HTA. dislipemia,

Alteraciones hormonales: está aumentada la LH, con niveles de FSH bajos o inferiores a lo normal, por lo que la relac ión LH/FSH es mayor. Hay un incremento leve de andrógenos, aumento de la estro-

hipertrigliceridemia, diabetes tipo 11, anomalías de la coagulación y del metabolismo esteroideo. Estas alteraciones implican un riesgo muy elevado de enfermedad cardiovascular que pueda resultar en enfermedad coronaria .

na y descenso de estrad iol. Anatomía: ovarios grandes, nacarados, polimicroquísticos, con hiperplasia de la teca interna.

3.2. Etiopatogenia La causa primaria se encuentra en discusión . Clásicamente, se ha descrito lo siguiente: Hay una elevación de LH (probablemente por pulsos demasiado rápidos en la secreción hipotalámica de GnRH).

3.3. Anatomía patológica Macroscópicamente, los ovarios pueden estar agrandados, tienen la superficie lisa y son de color grisáceo. Microscópicamente, hay engrosamiento y fibrosi s de la albugínea (cápsu la que rodea al ovario). La granulosa está poco desarrollada. La hiperplasia de la teca interna es lo más característico. Hay aumento de la zona medu lar ovárica.

Esta LH aumentada estimula en exceso la teca, ocasionando hiperplasia tecal. La hiperplasia tecal genera una sobreproducción de andrógenos ováricos (hay que recordar que la principal misión de la teca es la producción androgénica).

3.4. Clínica

También se produce una hiperproducción de andrógenos suprarrenales. Existe alteración en la regulación enzimática de P450cl7. Esta

No hay ningún signo ni síntoma constante ni patognomónica. Las pacientes suelen acudir por esteri lidad, trastornos menstruales o hirsutismo (Figura 6):

enzima es responsable de la actividad de otras enzimas ováricas y suprarrena les implicadas en la síntesis androgénica por lo que aumentan los andrógenos. Este aumento puede provocar obesidad, hirsutismo y anovu lación (recuérdese que un ambiente androgénico excesivo provoca atresia folicular) .

Esterilidad: es el síntoma más frecuente (presente en el 73% de los casos). Se debe a la fa lta de ovulación. Trastornos menstruales: la menstruación suele comenzar en la pubertad de forma normal y, varios años después, se inicia el trastorno en forma de oligomenorrea y de baches amenorreicos.

Manual CTO de Medicina y Cirugía , 2.a edición

Hirsutismo, acantosis nigricans, obesidad (típicamente androide con un cociente cintura/cadera > 0,85) y acné, acompañado a veces de alopecia. Varían en intensidad y en frecuencia. El grado de hirsutismo puede cuantificarse mediante la escala de Ferriman y Gallway. Resistencia a la insulina: que afecta a casi la m itad de las pacientes con SOP. El 40% de las mujeres con diabetes tipo 11 durante la edad rep roductiva tienen.

y, en aquellas que presentan un elevado riesgo de desarrollar diabetes mellitus tipo 2 (fuerte historia familiar de diabetes, IMC superior 30 kg/m 2, antecedentes diabetes gestacional), repetirla anualmente. También es posible evaluar la resistencia a la insulina mediante el cálculo del HOMA (insulinorresistencia si los valores se encuentran por encima de 3.90) o QUICKI (valores norma les por encima de 0.33). Síndrome metabólico: se realiza usando los siguientes criterios: incremento de la presión sanguínea (mayor o igual a 130/85 mmHg), incremento de la circunferencia de cadera (mayor o igual90 cm), elevación de la g lucosa en ayunas (mayor o igual 100 mg/dl), disminución de los niveles de HDL lipoproteínas (menor o igual a 50 mg/dl) y elevación de los niveles de triglicéridos (mayor o igual a 150 mg/ dl). Laparoscopia: permite apreciar el aspecto del ovario y tomar biopsias para estudio anatomopatológico. Anatomía patológica: aunque proporciona el diagnóstico de certeza, es poco habitual realizarla. Se debe realizar el diagnóstico diferencial con el hiperandrogenismo de causa suprarrenal por déficit enzimáticos congénitos de tipo

Trastornos me nstrua les

parcial, como la 21-h idroxilasa, y con tumoraciones secretantes de esteroides androgén icos. Y también con variaciones en los niveles de esteroides por alteraciones en la concentración de proteína transportadora de esteroides sexuales (SHBG). Para rea lizar el diagnóstico de SOP es recomendable utilizar los criterios internacionales establecidos por diferentes sociedades científicas (ESHRE/ASRM). Para ello es preciso q ue cump lan dos de las siguientes t res ca racterísticas después de la exclusión de trastornos relacionados: Oligovulación y/o anovulación.

Figura 6. Clín ica del SOP

Datos clín icos o bioquímicos de hiperandrogenismo. En el SOP hay anovulac ió n, po r lo que el ri esgo de padecer cá ncer de ova ri o es menor.

Ovarios poliquísticos, defin idos según ecografía transvag inal. Con que la imagen ecográfica sea en un solo ovario es suficiente.

Recuerda 3.6. Tratamiento

3.5. Diagnóstico Depende de la forma de presentación de este síndrome. Pérdida ponderal: constituye la primera opción terapéutica en paClínica: la sintomatología es muy variable, si bien las manifestaciones clínicas típicas son: anovulación, hiperandrogenismo e infertilidad.

cientes obesas. La pérdida de peso puede reducir el hiperinsulinismo e hiperandrogenismo, reducir el riesgo de diabetes tipo 2 y enferme-

Laboratorio: el aumento de LH y la disminución de FSH provocan una relación LH/FSH >2.5. También se produce un incremento de andró-

dad cardiovascu lar, regu larizar la menstruación y mejorar las posibi-

genos ováricos. Están elevadas la testosterona libre, la DHEA y la androstendiona. Disminuye la SHBG. Las concentraciones de SHBG están controladas por un equilibrio de influencias hormonales en su síntesis hepática. La testosterona es inhibidora m ientras que los estrógenos y la tiroxina son estimu ladores. Hay aumento de la estrona. La progesterona está ausente en la segunda mitad del ciclo, por lo que no se eleva la

lidades de gestación. Oligomenorrea y/o amenorrea: los anticonceptivos orales consiguen regularizar las reglas en estas pacientes, reducir el riesgo de adenocarcinoma de endometrio y frenar el exceso de síntesis de andrógenos. En aquellas pacientes que no deseen tomar anticonceptivos, deberá prescribirse la toma de gestagénos para inducir una hemorragia por deprivación por lo menos cada 3-4 meses.

temperatura en este periodo (la curva de temperatura es monofásica). Ecografía: la ecografía transvaginal debe cumplir los siguientes criterios para sospechar SOP: presencia de 12 o más fo lículos con

Hi rsutismo: para su tratamiento sintomático, se administrarán anticonceptivos orales, que disminuyen la producción de esteroides suprarrena les y ováricos, reduciendo el hirsutismo hasta en 2/3 de

diámetros de 2-9 mm y/o un volumen ovárico mayor de 1O mi. La combinación de los criterios ecográficos y hormonales (elevación de

las pacientes. Si fuera preciso, se asociará un antiandrógeno (espironolactona, acetato de ciproterona, flutamida, cimetidina, finasterida,

LH, disminución de FSH y elevación de andrógenos) perm ite diagnosticar SOP con alta sensibil idad (98%) y especificidad (93%). Resistencia a la insulina: la insulinoresistencia no se considera un

dexametasona, prednisona ... ). Ante cualquier tratamiento antiandrogénico en mujeres fértiles debe de asegurarse la anticoncepción efectiva para evitar el riesgo de feminización o teratogenia sobre el

cr iterio diagnóstico para SOP, pero su identificación permite conocer aquellas pacientes que se beneficiarían con un enfoque hacia la reducción de la resistenc ia a la insulina . Se evalúa med iante la sobre-

feto. Los antiand rógenos de efecto prolongado deben suspenderse al menos 2 meses antes de buscar gestación. lnsulinorresistencia: en pacientes que presenten alteraciones en el metabolismo hid rocarbonado está demostrada la utilidad de anti-

carga ora l de g lucosa con 75 gramos. Es recomendab le realizar una prueba oral de to lerancia a la g lucosa a todas las mujeres co n SOP

03 · Síndrome de ovario poliquístico (SOP)

diabéticos ora les sensibilizadores a la insuli na (metformina). La met-

Ginecología y obstetricia fo rmina no sólo mejora la sensibilidad a la insuli na, sino también el hiperand rog enismo, disminuye la concentración de LH y aumenta la SHBG. Un aspecto novedoso lo con stituye la utilización de la metfor-

1

03

Ca bergol ina (u otros análogos de la dopami na como bromocript ina, lisurida, etcétera): si la PRL está alta. Destrucción ovárica parcial con elec t roca ute rización o con láser por vía lapa roscópica (drilling): hace q ue dism i-

mina durante el emba ra zo que pa rece mejorar las tasas de aborto y de diabetes gestac iona l, sin efectos teratogén icos. Est erilidad: el tratamiento se hace de la siguiente manera: Inducción de la ovu lación:

nuya la síntesis de andróg enos, estabi lizándose la relación LH/FS H (an ti guamente se rea lizaba resecció n cuneifor me del ova ri o) .

Citrato de clomifeno: es el t ratamiento más usado. Induce la

ovu lación, incl uso, a veces, ovulaciones múlti ples. Se considera acepta ble su em pleo dura nte un máximo de doce meses. Gonadotropi nas: la FSH q ue se aporta refu erza el déficit de FSH endógena. Entrañ a ri esgo de síndrome de hiperestim ulación ovárica y de emba razo múltiple.

La te ndenc ia actua l para el tra tam iento de la este ri lidad en el SOP es usa r, como prime ra o pción, clo m ifeno y metformi na . Si fa ll a, se em p lea n go nadotropi nas co m o segun da o pció n y, si nu eva mente se fracasa, des trucció n parcial del ovario po r vía laparoscó pica .

está motivada por la anovulación crónica que presentan estas pacientes.

Ideas clave Jé?S "

El síndrome de ovario poliquíst ico es una alteración muy fre cuente, de etiología desconocida, que se caracteriza por una elevación de LH con niveles de FSH normales/ bajos, por lo que el cociente LH/ FSH está aumentado (> 2,5) .

"

Un elevado porcentaje de mujeres con ovario poliquístico sufren alteraciones metabólicas (síndrome X), por lo que es muy frecuente la asociación de: resistencia a la insulina, obesidad, hipertensión, dislipemia, hipertrigliceridem ia, diabetes tipo 11, anomalías de la coagulación y del metabolismo esteroide. Todas estas alteraciones implican un elevado riesgo cardiovascular.

"

En el d iagnóstico, es preciso la combinación de un criterio clínico y uno bioquím ico (cocient e LH/FSH elevado, aumento de andrógenos ováricos, elevación de testosterona y DHEA) junto con la confirmación ecográfica (1 O o más folícu los de peq ueño tamaño subcorticales, con aumento del estroma), si bien el diagnóstico de certeza es la comprobación de hipertecosis en una biopsi a ovárica.

"

El tratamiento en las mujeres que no deseen gestación será con anticonceptivos hormonales, solos o asociados a fármacos antiandrogénicos. Si buscan gestación, se deberá inducir a la ovu lación con citrato de clom ifeno o gonadotropinas. Si se fracasa, se intentará una destrucción ovárica parcial con electrocauterización por vía laparoscópica (drilling ovárico) para, posteriormente, volver a intentar inducir la ovulación.

Los síntomas típicos son obesidad, trastornos menstruales (oligomenorrea, opsomenorrea), hirsutismo, resistencia insulínica y esterilidad. Esta última es el síntoma aislado más frecuente y

"

Casos clínicos Paciente de 32 años, obesa (talla 160 cm, peso 102 kg), con antecedentes de un aborto espontáneo hace tres años y diagnosticada de síndrome de ovario poliquístico. Actualmente lleva dos años de búsqueda de embarazo sin éxito. ¿Cuál sería su primera recomendación para alcanzar dicho objetivo?

1) 2) 3) 4)

Fecundación in vitro. Inseminación artifici al intraconyuga l (I AC). Metformina. Pérdida de peso.

Ante una paciente de 28 años con un índice de masa corporal (IMC) > 30, baches amenorreicos, acné, hirsutismo y esterilidad de dos años de evolución, cabría pensar en:

1) 2) 3) 4)

Hipoti ro id ismo. Fallo ovárico precoz. Sínd ro me de ova ri os pol iq uísticos. Amenorrea hipogonadotropa.

RC: 3

RC: 4

Case Study · · A 25-year-old female was admitted to hospital for evaluation of amenorrhea. Menarche occurred at age 11 and she had regular spontaneous menses at 34-day intervals lasting for four days. During the previous six months, she had had three menstrual bleeds. She had signs of clinical hyperandrogenism represen ted by the presence of hirsutism (Ferriman-Gallwey score of 9) and persistence of acne. Physical examination revealed mode rate obesity (149 cm, 55 kg). Serum gonadotropin levels were:

FSH 5 mU I/mL and LH 12.5 mUI/mL. Serum testosterone and androtendione levels were increased. lndicate the first step to undertake regarding this patient:

1) 2) 3) 4)

Weight loss. Clomiphene cit rate. Com bined oral contra ceptive. Spi ro nolacto ne.

Correct answer: 1

1

Ginecología y obstetricia_

Metrorragias

ORIENTACIÓN

ENARM

Este tema es poco importante. Hay que centrarse fundamentalmente en las Ideas Clave .

sibles procesos orgánicos que pueden estar asociados a los episodios de sangrado. Para el lo se dispone de: Exploración ginecológica: ayuda a descartar patología en genitales externos, vag ina, cérvix o tumorac iones uterinas causantes del sangrado, así como la presencia de traumatismos o cue rpos extraños. Ecografía: es la primera prueba diagnóstica en pacientes con hemorragia para identificar alteraciones estructura les.

4.1. Clasificación de las hemorragias uterinas

Histeroscopia: perm ite visua lizar la cavidad endometrial al m ismo t iempo que obtener muestras de biopsias. Estaría indicada en mujeres peri menopáusicas o postmenopáusicas con metrorrag ia para descartar el cáncer de endometrio o las lesiones prema lignas (h iperplasias) y

Las hemorragias uterinas se clasifican en los siguientes t ipos:

en mujeres con un aumento del grosor endometrial por ecografía (por

No cíclicas: cuando el sangrado es independ iente de la reg la. Se

encima de S mm en mujeres postmenopáusicas y de 12 mm en mu-

llama metrorragia.

jeres perimenopáusicas). Es mejor realizarla una vez que la paciente ha

Cíclicas: si el sangrado es cíc lico, pueden ser:

dejado de sangrar, ya que el sangrado dificulta la visualización de la

Hipermenorrea o menorragia: pérdidas en una cantidad

cavidad endometrial. Si no se dispone de la histeroscopia para la toma

superior a 180 mi o de duración de más de siete días, o ambas

de biopsias se podrá optar por el legrado fraccionado.

que ocurren con interva los regulares.

No se debe t ratar hasta haber llegado a un diagnóstico mediante explo-

Polimenorrea: la menstruación es más frecuente (intervalos en la

ración, cito logía, co lposcopia, ecografía, histeroscopia, laparoscopia, etc.

regla de menos de 21 días), pero normal en ca ntidad y du ración .

4.2. Causa de la hemorragia

Mujer perim enopá usica o postmenopáusica co n metrorrag ia: descartar adenoca rc inoma de endometr io co n histeroscopi a con toma de b iopsias.

Las causas de la hemorragia son las siguientes: Orgánicas: tumores malignos, tumores benignos (miomas, pólipos), eritroplasia, traumatismos, endometriosis, coagu lopatías, conges-

4.4. Tratamiento

t ión venosa secundaria a insuficiencia cardíaca, HTA asociada a arteriosclerosis de los vasos uterinos, cirrosis, etcétera . Disfuncionales: no hay lesión orgánica, sino alteración en la regulación endocrina del ciclo. En la mayoría de los casos se encuentra un

El tratam iento debe persegu ir los siguientes objetivos: Cohibir la hemorragia.

endometrio proliferativo simple o hiperplásico. Son más frecuentes tras

Evitar recidivas.

la menarquia y en la perimenopa usia debidas a ciclos anovu latorios.

Tratar la anemia ferropénica.

4.3. Diagnóstico

Cohibir la hemorragia No hormonal: Antifibrinolíticos: disminuyen la hemorragia hasta en un 50%. Ac-

Se debe estar alerta ante toda hemorragia gen ital, ya que es el principa l

túan reduciendo los niveles de plasminógenos por lo que reduce la

síntoma, y habitualmente el más precoz, de la mayoría de los tumores

fibrinólisis. El más utilizado es el ácido tranexám ico. Los efectos se-

genitales. Ante un cuadro de metrorrag ia es importante descartar los po-

cundarios asociados a su utilización son náuseas, vóm itos y diarreas.

Ginecología y obstetricia

04

1

AINE: inhiben la prostaglandinasintetasa y, por tanto, los niveles endometriales de prostagland inas vasod ilatadoras. Son muy uti lizados el ácido mefenám ico, el ibuprofeno y el naproxeno. La reducc ión del sangrado menstrual, de manera global, alcanza un 30%. Los efectos secunda rios más important es son los gastrointestinales, siendo su incidencia baja y no grave. Se debe n usar con precaución en muj eres con antecedentes de ulcus, hemorragia digestiva o en pacientes con insuficiencia renal. Hormonal: las pautas e indicaciones son com plejas y variadas. En genera l, se emplearán : Estrógenos: se usan en hemorragias agudas y en endometrios atróficos, ya que al estimula r la prolife ración endometrial frenan el sangrado. Son de elección los estrógenos equ inos conjugados intravenosos aunq ue también se pueden usar preparados ora les. Estrógenos más gestágenos: se emplean en hemorragias moderadas (etinilestradiol más acetato de noretisterona). Progest ágenos: se administran diariamente del 5° al 26° día del ciclo, ayudando a controlar la pérdida hemática a corto plazo, siendo especialmente útiles en casos de endometrios hiperplásicos proli ferativos.

Vasos uterinos

Li gamento redondo

DIU-Levonorgestrel: reduce el sang rado en más del 95% de los casos tratados con benefi cio máximo a los 6 meses. Especia lmente útil en muj eres que no deseen gestación. Danazol : es un esteroide sintético que inhibe la esteroidogénesis en el cuerpo lúteo, bloquea ndo el pico de FSH y LH, por lo que produce un aumento de and rógenos y descenso de estrógenos. Quirúrgico: Legra do endometrial: es un método eficaz para contener la hemorragia de forma rápida y además proporciona material para estudio histológ ico, aunque requie re hospitalización y anestesia, por lo que no debe ser utilizado como medida terapéutica de primera elección. Está ind icado en mujeres con anemia intensa de causa ginecológ ica. Ablación-resección endometrial: son técnicas que buscan la destrucción (ablación) o extirpación (resección) del endometrio, incluyendo 1-3 mm del miometrio subyacente para destruir la

Figura 7. Histerectomía abdominal

Evitar recidivas Una vez contenido el episodio agudo, se debe regularizar el ciclo para evitar recidivas: Si el endometrio es proliferativo, no hay ovu lación y es una mujer joven que desea descendencia, se induce a la ovulación (clomifeno, gonadotropinas, análogos de GnRH).

capa basal del endometrio y así reducir la cantidad de sangrado o producir amenorrea. Son proced imientos efectivos con tasas de satisfacción elevadas, siendo una alternativa útil cuando fracasa el tratamiento médico en mujeres mayores de 40 años, con tamaño uterino menor de 11 cm, con enfermedades sistémicas que incrementen el riesgo anestésico y sin deseos de embarazo posterior.

En mujeres que no desean descendencia, se pautan anticonceptivos ora les, durante un periodo que oscila entre los 3 y 6 meses. En mujeres posmenopáusicas co n metrorrag ia recidivan te, se indica histerect om ía.

Histerectomía: es el proced imiento más radica l y defin itivo y está ind icado cuando el tratam iento médico fa lla en mujeres que no desean más descendencia, o en aq uellas en las que el

Tratar la anemia ferropénica

examen anatomopatológico demuestre una hiperplasia endometrial atípica (Figura 7).

Se debe evaluar la situación hematológica con hemog rama y estudio de coagu lación, y si existe anemia, tratarla.

d iagnóstico de elección es la biopsia dirigida por histeroscopia, y si no se dispone de la histeroscop ia, se realizará un legrado fra cci o nado.

Ideas clave ES "

"

Las metrorrag ias pu eden t ener una cau sa orgánica (tumores malignos o ben ignos, endometriosis, coa gulopatías. ..) o di sfuncion al (a lteraciones en la regulación hormonal del ciclo ovárico, sobre todo debidas a ciclos anovulatorios). Es fundam ental realiza r un estudio histológ ico, ya que puede t rata rse del primer síntoma de un tumor genital. Por ello, el

"

En el t ratam iento, es pos ible uti lizar est rógenos solos o combinados con gestágenos, progestáge nos en seg unda fase del ciclo, danazol, antifibrino líticos y AIN E. Otra alternativa de tratamiento es la cirugía, pud iendo rea lizar legra do uterino, histeroscopia quirúrgica, ab lación endometrial o histerectomía.

Manual CTO de Medicina y Cirugía, 2.8 ed ición

Case Study A 45-year-old woman (gravida 2, para 2) presented distension of the lower abdomen. Past medical history is unremarkable, with no history of abdominal or pelvic surgery or episodes of genital bleeding. Uterine myoma had been diagnosed age 40, but no treatment was performed, as she remained asymptomatic. Interna! examination showed an unmovable uterine tumor and marked deviation of the uterine cervix. Transvaginal ultrasonograhy revealed a 25 x 35 cm uterine tumor. Doppler

04 · Metrorragias

ultrasound showed numerous vessels within the tumor. Laboratory analysis revealed a decreased hemoglobin level of 8.2 g/dL. Which do you think is the most correct treatment for this patient? 1) 2) 3) 4)

Medroxyprogesterone acetate. Myomectomy. Uterine artery embolization. Abdominal hysterectomy.

Correct answer: 4

Control de la fertilidad

Tema muy significativo, así que no hay que dejar de estudiarlo, sobre todo lo referente a los anticonceptivos hormonales y al dispositivo intrauterino.

5.2. Métodos naturales Los métodos natu rales son los siguientes: Ogino: se ha de tener en cuenta que la ovulación ocurre el día 14, y que el óvu lo puede ser fecundado sólo durante 24-36 horas, se ca lcula el periodo fértil o de inseguridad en func ión de la duración del

5.1. Eficacia contraceptiva

ciclo más largo y del más corto. Lactancia materna: du rante la lactancia, los niveles elevados de prolactina suprimen en un grado va riable el eje hipotálamo-hipofi-

Se acepta que la eficacia de los distintos métodos, de mayor a menor

sario, pero los niveles de PRL varían considerablemente y es imprevisible la duración de la amenorrea. Coito interrumpido: es un método poco seguro. Sus inconvenien-

eficacia, es la siguiente: l.

La esterilización quirúrgica masculi na (vasectom ía) y femenina (bloqueo tubárico) son los métodos má s eficaces, están en el mismo

2.

nivel que los modernos anticonceptivos hormonales. El dispositivo intrauterino (DI U) es el siguiente en eficacia.

3.

Le siguen el diafragma y el preservativo, con un nivel similar entre sí.

4. S.

Algo menos eficaz es la esponja. Menos eficaces se muestran los métodos naturales, como los del ritmo, la temperatura, etcétera, y por debajo de el los, el coito inte-

tes son: existe capacidad fecundante del espermatozoide en vulva; previa a la eyacu lació n se produce fuga espermática; puede generar frustración así como hipertrofia prostática, síndrome de congestión pelviana, frigidez e insatisfacción sexual. Temperatura: el periodo de "segu ridad " empieza la noche del tercer día de hipertermia confirmada y finaliza con la llegada de la menstruación (hay que recordar que en la ovulación se produce un audebido a la acción mento de la temperatura por encima de 37

oc

hipertérmica de la progesterona).

rrumpido (Figura 8).

5.3. Métodos de barrera

B

A

Los métodos de barrera son los siguientes: Preservativo masculino: el número de fallos de este método desciende considerablemente si se le asocian espermicidas. Es el método anticonceptivo de elección en el varón joven. Menos usado que él es el preservativo femen ino. Diafragma: es indispensable el empleo conjunto del mismo con una crema espermicida. Está indicado en casos de intolerancia a la píldora y en aquellas mujeres en las que la colocación de un DIU no es aconsejable. No debe usa rse en caso de anomalías morfológicas y tampoco en el pos parto in mediato (deben pasar entre tres y cinco meses). Espermicidas: el objetivo de los esperm icidas es doble: el bloqueo mecán ico del cue llo y la destru cción de los espermatozoides. La máxima protección se obtiene aplicando conjuntamente el espermicida con un anticonceptivo de barrera tipo preservativo. Ofrecen

Figura 8. Métodos anticonceptivos: (A) AO, (B) DIU, (C) preservativo, y (D) Diafragma

una protección relativa frente a ETS. Esponjas vaginales: se trata de discos cilíndricos que poseen espermicida. Absorben el semen y destruyen los espermatozoides.

Manual CTO de Medicina y Cirugía, 2.a edición

S.4. Dispositivo intrauterino

Embarazo ectópico y DIU

Mecanismos de acción

El DIU previene mejor el embarazo normal que el ectópico, por lo que la frecuencia relativa de este último aumenta. Además el DIU favorece la enfermedad inflamatoria pélvica (EIP), que es un facto r de riesgo para

El dispositivo intrauterino (DIU) posee varios mecan ismos de acción: Todos ellos provocan una reacción inflamatoria local a un cuerpo extraño que produce un aumento de la permeabilidad capilar, edema endometrial e incremento de los macrófagos, ejerciendo un efecto espermicida y antii mplantatorio sin descartarse actuación alguna sobre la motilidad t ubári ca . Los DIU liberadores de cobre ejercen una acción gameticida (sobre todo espermicida) lo que dificu lta la fertilización. El cobre potencia la acción espermicida y, por tanto, eleva la eficacia anticonceptiva. Su efecto es dosis depend iente: a mayor carga de cobre, más alta eficacia

el embarazo ectópico. La frecuencia de gestación ectópica crece con el tiempo de uso del DIU.

EIPy DIU El factor determinante de desarrol lar EIP en las mujeres portadoras de DIU está directamente relacionado con las ETS. El riesgo atribuible al DIU se encuentra en relación con el proceso de inserción y aparece, sobre todo, en los tres primeros meses tras su colocación. Ante la sospecha de EIP, se deben hacer cultivos, iniciar el tratam iento empírico con antibióticos y extraer el DI U.

anticonceptiva durante más tiempo. Los DIU liberadores de levonorgestrel (LNG) ejercen además un efecto sobre el endometrio, lo que lo hace hostil a la migración de los espermatozoides, por lo que dificulta la fertilización. Esta acción se ve reforzada por la producción de un moco cervica l de características gestagénicas, barrera impenetrable para los espermatozoides. Se puede insertar a partir de las seis semanas tras el parto, ya que no afecta a la cantidad ni a la calidad de la leche materna y no ocasiona

S.S. Anticoncepción hormonal Los anticonceptivos hormonales combinados son fármacos compuestos por un estrógeno y un progestágeno, ambos sintéticos, que pueden administrarse por distintas vías (oral, parenteral, transdérmica, vaginal) y cuyo mecanismo

efectos negativos sobre el recién nacido. En el 20% de las mujeres, la regla desapa rece por completo, lo que no entraña ningún riesgo para la mujer. Este DI U, a diferencia de los otros, permite un aumento

de acción es la inhibición de la ovulación, lo que les confiere una alta eficacia anticonceptiva, reversible tras la suspensión de su administración.

de los niveles de hemoglobina y ferritina, reduce el riesgo de enfermedad inflamatoria pélvica y disminuye el riesgo de emba razo ectó-

El estrógeno presente en todos los preparados comerc ializados es el etini lestrad iol. La dosis es va riabl e osci lando entre 50 y 15 1-1g diarios.

pico respecto a otros dispositivos. Tambi én red uce la incidencia de miomas y mejora la dismenorrea.

Contraindicaciones No se recomienda insertar o continuar el uso del DIU en las sigu ientes situaciones: Embarazo confi rm ado o sospechado.

Se util izan mú ltiples progestágenos en los diferentes preparados dispon ibles. Todos ellos se caracterizan por presentar una potente actividad antigonadotrófica (que les confiere la alta eficacia anticonceptiva), progestagénica y antiestrogénica. La diferencia entre ellos radica en la capacidad para interaccionar o no con receptores de and rógenos, glucocorticoides o mineralcorticoides, por lo que puede así presentar actividad g lucocorticoidea, and rogénica, antiandrogénica o antim ineralcorticoidea .

Hemorragia genital sin filiar. Infecciones pélvicas agudas, recientes o recurrentes. Sangrado uterino anormal o tratamiento con anticoagu lantes. En este caso no está contraindicado DIU-LNG.

Se dispone de los siguientes progestágenos: Acetato de ciproterona: presenta potente actividad antiandrogénica por lo que resulta de gran ayuda en el tratamiento del acné, del hirsutismo

Distorsiones graves de la cavidad uterina o cervical, congénitas o ad-

y de la hi pertricosis. Posee también una leve actividad glucocorticoidea

quiridas.

Levonorgestrel: pertenece a la segunda generación de gestágenos y presenta una leve actividad androgénica por lo que modifica el perfillipídico, aumenta ndo el nivel de triglicéridos y el de LDL, dismi-

Neoplasia genita l. Endometritis posparto, aborto infectado. Enfermedad de Wilson (contra indicado DIU-Cu) .

Momento de la colocación

nuyendo el de HDL. Gestodeno, desogestrel, etonogestrel: son los llamados gestágenos de tercera generación. Tienen menor actividad androgénica lo que determina una alteración más pequeña del perfillipídico; sin embargo, pre-

Durante la menstruación. Tras la primera regla después de un aborto precoz. Tras la segunda menstruación después de un parto, o al menos seis semanas posparto, o tras un aborto tardío.

Embarazo y DIU Ante un embarazo en una mujer portadora de DI U, es prioritario establecer si se trata de una gestación intrauterina o ectópica. Si se confirma que el embarazo es intrauterino, si el DIU no se extrae, existe un SOo/o de riesgo de aborto si bien no está descrito un aumento de malformaciones fetales ni de partos pretérmino.

05 · Control de la fertilidad

sentan un mayor riesgo tromboembólico venoso en combinación con el etinilestradiol que la combinación de levonorgestrel con etinilestradiol. Norgestimato: se comporta como los gestágenos de tercera generación en cuanto al metabolismo lipídico y como el levonorgestrel en lo referente al riesgo tromboembólico. Drospirenona: presenta una marcada actividad antiandrogénica, si bien es menos potente que el acetato de ciproterona. Posee act ividad antimineralcorticoidea por lo que evita en mayor o menor medida los efectos colatera les debidos a la retención hídrica. Acetato de clormadinona: con una estructura muy similar a la progesterona, t iene una elevada acción antiandrogénica que sólo es superada por el acetato de ciproterona.

Ginecología y obstetricia Mecanismo de acción Disminuyen la GnRH, ya que producen un feedback negativo que in-

1

05

Mejoran la dismenorrea y el dolor periovulatorio. En algunas mujeres pueden resultar eficaces para el síndrome premenstrual al aliviar algunos de sus síntomas.

hibe la liberación hipot alámica de GnRH . Impiden el pico ovulatorio de LH, ya que anulan la secreción pulsátil

Embarazo ectópico: su aparición es excepcional g ra cias a la eleva-

de GnRH, responsable del pico preovulatorio de LH. En el ovario inhiben la ovulación, puesto que no se ha producido el

da eficacia anticonceptiva que presentan.

pico preovu latorio de LH.

Enfermedad inflamatoria pélvica: su incidencia es más baja en las mujeres usuarias y además presentan cuadros de menor gra-

Cambian la capacitación espermática, espesan el moco cervical y al-

vedad.

teran el medio vagina L

Enfermedad benigna de la mama: disminuye la aparición de en-

Varían la contracción uterina, dificultando el transporte de los esper-

fe rmedad fibroquística y de fibroadenomas.

matozoides.

Quistes ováricos: se reduce el riesgo de aparición de quistes fun-

Alteran la motilidad y funcionalidad de la trompa, dificultando la fe-

cionales.

cundación.

Acné, hirsut ismo y seborrea: cualquier preparado mejora esta

Modifican la estructura endometrial e impiden la implantación.

sintomatología, ya que inducen un incremento en la síntesis de la proteína transportadora de hormonas sexuales (SHBG), por lo que

Clasificación

desciende la testosterona libre activa. Además los preparados que contienen acetato de ciproterona, acetato de clormadinona, dros-

La clasificación de los anticonceptivos hormonales se realiza en función

pirenona o desogestrel son más beneficiosos por su efecto antian-

de diferentes aspectos:

drogénico.

Según la dosis administrada a lo largo del ciclo:

Osteoporosis: existe un efecto protector sobre la densidad mineral

Monofásicos: llevan una dosis constante de estrógenos y ges-

ósea que aumenta con una mayor duración de uso.

tágenos a lo largo del ciclo. Son los más empleados.

Dism inuyen el riesgo de cáncer de endometrio.

Bifásicos: todos los comprimidos contienen ambos esteroides;

Reducen la incidencia del carcinoma epitelial de ovario, incluso en mujeres con historia familiar o con m utaciones genéticas, al inhi-

sin embargo, durante los pri meros días, la dosis de gestágenos es menor. Trifásicos: la dosificación de estrógenos y gestágenos se hace en tres niveles diferentes, según los días del ciclo. Según la forma de administración:

bir la ovulación.

Efectos adversos menores Los efectos adversos menores son los siguientes:

Orales.

Náuseas y vómitos, secundarios al contenido de estrógenos del pre-

Parenterales.

parado. Tienden a desaparecer con el uso continuado. Aumento de peso que se atribuye generalmente a la retención de

Sistemas de liberación continuada: Anillo anticonceptivo vaginal: libera 15 ~g de etilinilestra-

líquido, por lo que los gestágenos con actividad antimineralcorticoi-

diol y 120 ~g de etonogestrel; se coloca en la vagina la primera semana posmenstruación y se retira una semana cada

dea pueden reducir su incidencia.

21 días.

Cefaleas. Masta lgia que se debe al efecto estrogénico sobre áreas de hiperplasia focal o por la interrupción brusca del gestágeno en la semana de

Implantes subdérmicos: son barritas con etonogestrel que se insertan bajo anestesia local en la cara interna del

descanso.

antebrazo o el brazo. Duran entre tres y cinco años.

Cloasma, hiperpigmentación cutánea parcheada, de desarrollo lento

Parches: la absorción es transdérmica y el recambio es semanal, liberando diariamente 20 ~g de etilini lestradiol y

y en relación con el tratamiento y la exposición solar prolongada.

150

~g

de norelgestromina.

Depresión. Disminución de la libido. Infecciones urinarias dependientes de la dosis de estrógeno. Sangrados intermenstruales (spotting): se presentan con mayor frecuencia en los primeros ciclos y son más habituales cuanto más baja

La eficac ia del antico nceptivo o ral depende del gestágeno

es la dosis de estrógeno. Amenorrea como consecuencia de una escasa proliferación endometrial por predominio del componente gestagénico sobre el estrogénico.

Efectos beneficiosos

Amenorrea pospíldora.

Sus efectos beneficiosos son los sigu ientes:

Adenomas hepáticos: tumoraciones quísticas hemorrágicas, habitualmente asintomáticas, que pueden estallar y producir shock hipovolém ico. Es posible su involución tras la supresión de la anticoncep-

Ciclo menstrual: Regu lan el ciclo menstrual, por lo que resultan útiles en el ma-

ción hormonal (Figura 9).

nejo de las hemorragias disfunciona les.

Colel itiasis: los estrógenos poseen propiedades litogénicas, incrementando la secreción biliar de colesterol y su índice de sa-

Disminuyen el sangrado menstrual, siendo beneficiosos en el tratamiento de menorragias e hipermenorreas funcionales, disminuyendo la incidencia de anemia ferropénica de origen ginecológico.

turac ión. Colestasis: puede ocurrir en los primeros meses de uso y se manifiesta por prurito e hiperbilirrubinemia conjugada.

Manual CTO de Medicina y Cirugía, 2.a edición

Hipertensión arterial. Diabetes mellitus no controlada. Migraña focal. In suficiencia rena l. Alteraciones de la coagulación. Antes del inicio del tratamiento con anticoncepción hormonal, es imprescindible realizar una historia clínica y un examen físico, que debe incluir una exploración mamaria y una citología, así como la medida de la presión arterial y del peso. La s pacientes deberán tener una revisión ginecológica anua l que comprenda: exploración mamaria, citolog ía y colposcopia, control de la presión arteria l, y pruebas de laboratorio que incluirán: glucemia basa l, colesterol total y las fracciones HDL y LDL, triglicéridos y antitrombina (Figura 10) . Figura 9. Adenoma hepático en paciente usua ria de anticoncepción hormonal

Efectos adversos mayores Trom boembolismo venoso: existe una elevación del riesgo relativo en las mujeres usuarias, sobre todo durante el primer año de empleo. Este riesgo, que es menor que el observado en las mujeres gestantes, se debe a que la anticoncepción induce una elevación de los factores 1, 11, VI l, IX, X y del plasminógeno. Los estrógenos a altas dosis aumentan la actividad fibrinolítica. Incrementan el riesgo posoperatorio, por lo que se aconseja suspender su toma un mes antes de una intervención. Hi pertensión arteria l: es poco frecuente pero puede aparecer hasta en un 5% de las usuarias, sobre todo, en los primeros seis meses de

USO.

Infarto ag udo de m ioca rd io: su incidencia se relaciona con otros factores de riesgo como: hipertensión arterial, hiperlipoproteinemia, diabetes y antecedentes familiares. Cáncer de mama: se ha descrito un ligero aumento del riesgo relativo en mujeres jóvenes que iniciaron la toma de manera precoz y prolongada. Los cánceres diagnosticados tienen mejor pronóstico por ser más localizados en comparación con los diagnosticados en no usuarias. Cá ncer de cérvix: se ha descrito un riesgo aumentado de carcinoma in situ e invasivo en usuarias a largo plazo con infección persistente por HPV al actuar de facilitadores en la carcinogénesis.

Contraindicaciones Las contraindicaciones para la administración de los anticonceptivos hormona les son las siguientes:

Figura 1O. Exploración mamaria y ginecológica previa a la toma de anticoncepción hormonal

Lactancia en los primeros seis meses postparto. Embarazo o sospecha de embarazo. Tener o haber ten ido enfermedad tromboembólica, incluyendo

5.6. Intercepción poscoital

trombosis venosa profunda, embolia pulmonar, accidente cerebrovascular, insuficiencia cardíaca, cardiopatía isquémica y cardiopatía reumática .

La probabilidad de embarazo tras mantener relaciones sexua les sin pro-

Tener o haber tenido cáncer de mama o de cérvix.

tección o con rotura de preservativo varía entre un 20-40%, si el coito

Tener o haber tenido tumores hepáticos benignos o mal ignos.

ocurre a mitad del ciclo, y es de un 5% en cualquier otro momento del

Enfermedad hepática aguda o crónica activa.

mismo. Los métodos que se pueden utilizar como anticoncepción de

Durante la administración crónica de rifampicina o anticonvulsivantes.

emergencia son los siguientes: Levonorgestrel: 1,S mg en dosis única (de elección) o fracciona-

Hemorragia uterina anormal de etiología no determinada.

do en dos dosis (0,75 mg cada una con un intervalo máximo de 12 horas) . Tiene máxima eficacia cuanto más precoz se realice el trata-

Fumadoras mayores de 35 años de edad.

miento, evitando entre el 85-95% de los embarazos, si se administra

Se debe tener precaución a la hora de administrarlos si:

05 · Contro l de la fertilidad

Ginecología y obstetricia

1

05

en las primeras 72 horas. La incidencia de efectos secundarios es escasa (el más frecuente: náuseas). Método Yuzpe: es un combinado de estrógenos y progestágenos en altas dosis, vía oral; administrar un com primido cada 12 horas, dos tomas, dentro de las primeras 72 horas después del coito.

DIU poscoital: se usa cuando han transcurrido entre tres y cinco días poscoito. Es muy eficaz. Mifepristona (RU-486): es un fármaco antiprogestágeno, empleado en interrupciones voluntarias del embarazo (IVE) y en intercepción poscoital. Tiene una efi cacia cercana al 100%.

Ideas clave z;

teración de la contractilidad uterina, modificación endometrial y modificación de la capacitación espermática .

"

El dispositivo intrauterino (DIU) produce una reacción inflamatoria que interfiere en el transporte espermático e impide la implantación. Su eficacia es potenciada por el cobre (tiene tam bién acción espermicida) o por ellevonorgestrel (produce atrofia de las glándulas del endometrio y modifica el moco cervical, dificultando la penetración espermática) .

"

El DIU está contraindicado en casos de embarazo, infección pélvica, reciente o recurrente, tumor maligno cervical o uterino y en sangrados uterinos anormales o tratamientos anticoagulantes. Sin embargo, en este último caso, sí se puede utilizar el DIU-Ievonorgestrel, ya que al actuar localmente, reduce el crecimiento del endometrio, lo que motiva una reducción del sangrado tanto en cantidad como en duración. También, en comparación con los otros DI U, reduce el riesgo de enfermedad inflamatoria pélvica y de embarazo ectópico.

"

Los anticonceptivos hormonales presentan varios mecanismos de acción : inhibición de la GnRH, impiden el pico ovulatorio de LH, alteración de la motilidad y funcionalidad de la trompa, al-

Casos clínicos Una paciente de 19 años, nuligesta, acude a la consulta por irritabilidad, inestabilidad emocional, cefaleas, sensibilidad y distensión mamaria. La sintomatología se inicia una semana antes de la menstruación y desaparece tras presentarse la regla. Una vez comprobada la relación con el ciclo menstrual mediante un diario de síntomas, decidimos pautar tratamiento pues la paciente refiere que la sintomatología interfiere con su trabajo y sus relaciones sociales. ¿Qué aconsejamos como primera elección terapeútica?

Case Study . A 33-year-old housewife was admitted to the hospital. She smoked 40 cigarettes per day and had used oral contraception for at least five years. Five days befare admission she complained of headaches, upper abdominal pain, and vomiting. On examination she was obese, had right hypochondrial tenderness and blood pressure of 220/140 mmHg. There were no abnormal laboratory findings. On day 3 the patient was anuric

"

Aparte del efecto anticonceptivo, presentan efectos ginecológicos beneficiosos tales como: disminución de la patología ovárica y mamaria benigna, bajada de la incidencia de carcinoma epitelial de ovario y de adenocarcinoma de endometrio y disminución de la dismenorrea.

"

La anticoncepción hormonal es un factor de riesgo para la enfermedad tromboembólica y el infarto de miocardio, al mediar una elevación de los factores 1, 11, Vil, IX, X y plasminógeno, aunque también existe un aumento de la actividad fibrinolítica. Por ello, están contraindicados en mujeres con riesgo cardiovascular (fumadoras > 35 años, no fumadoras > 40 años, antecedentes de trombosis profunda o embolismo pulmonar, alteraciones de la coagulación, diabetes con afectación vascular, cardiopatías graves ... ). Otras contraindicaciones para su utilización son: pacientes con afectación hepática importante (adenoma hepático, hepatopatías activas), porfiria aguda intermitente, antecedentes de colestasis intrahepática, discrasia sanguínea, sangrado genital anormal no filiado, etc.

1) Aumento de in gesta de cafeína y azúcares refinados en segun-

da fase del ciclo menstrual. 2) Progesterona 200-300 mg/día del día 14 al 23 del ciclo.

3) Evitar ejercicio físico y tomar suplemento de vitamina B12 durante sintomatología a una dosis de 150 mg/día. 4) Alprazolam 0,25 mg/8 horas, desde el día 20 del ciclo hasta el 2. 0 día de la regla, disminuyendo después a una toma al día. RC:4

with absent femoral pulses. Blood pressure: 140/90 and both legs were cold and pulseless. The most likely complication of oral contraception is: 1) 2) 3) 4)

Hepatocellular adenoma. Massive arterial thrombosis. Cholestasis. Arterial hypertension.

Correct answer: 2

...---Gine_c_a1o_g ía y obstetticia_

Esterilidad e infertilidad

ORIENTACION

ENARM

La parte que más atención requiere es el apartado de Tratamiento.

Alteraciones sistémicas: son procesos orgánicos que afectan a la neuroh ipófisis o a las glándulas suprarrenales, enferm edades sistémicas graves, alteraciones tiroideas, obesidad, adelgazamiento extremo, abuso de drogas y tóxicos.

Factor masculino La esterilidad se define como la incapacidad de uno o de ambos miembros de la pareja para la concepción natural en un plazo razonab le. Pre-

Se produce un aumento muy significativo de la esterilidad mascu lina en las

senta una prevalencia de entre ell O y el 20%.

últimas décadas. Las causas de la misma varían en las diferentes poblaciones estudiadas. Entre las más frecuentes, se encuentran las sigu ientes: varicocele, esterilidad masculina de causa idiopática, insuficiencia testicular, criptorqui-

Esterilidad primaria: incapacidad de conseguir una gestación espontánea desde el inicio de las relaciones sexua les sin que, al menos durante un año, se hayan utilizado métodos anticonceptivos. Esterilidad secundaria: incapacidad pa ra consegu ir una gestación espontánea tras la consecución previa de un embarazo. Infertilidad: incapacidad de lograr un recién nacido viable tras, al menos, haber log rado dos embarazos consecutivos. Subfertilidad: incapacidad de consegu ir una gestación espontánea en un periodo de tiempo superior al de la media de la población.

dia, azoospermia, orquiepid idimitis, alteraciones hipotalámicas, etcétera.

Factor mixto O de origen desconocido Supone entre ell 0-20% de los casos, por causas idiopáticas, o por reacción inmunológica al semen que im pide que penetre en el moco cervical.

6.2. Estudio de la pareja infértil 6.1. Causas de esterilidad Se considera que el 20% de las ellas son de causa mixta, el 40% de causa

El objetivo del estudio de la pareja estéril es realizar aquellas pruebas diagnósticas que objetiven alteraciones cuya corrección se asocie con tasas superiores de gestación, evitando las pruebas innecesarias que demoren

mascu lina y el 40% de causa femenina.

el diagnóstico y encarezcan el proceso. Entre las pruebas que deben realiza rse obligatoriamente en todo estud io de esterilidad se encuentran:

Factor femenino Las causas dentro del factor femenino son las sig uientes:

Alteraciones tubáricas: representan la causa más frecuente dentro del factor femenino (40%). Genera lmente son debidas a secuelas de procesos infecciosos o inAamato rios como: EIP, infecciones postaborto, infecciones puerperales, hidrosálpinx.

Alteraciones ováricas: pueden estar debidas a alteraciones orgán icas (agenesia, endometriosis, tumores ... ) o funcionales (anovulación, insuficiencia de cuerpo lúteo). Alteraciones anatómicas: las que se prese nta n con mayor frecuencia son ad herencias uterinas, lesiones endometriales orgánicas como pól ipos y miomas submucosos. Alteraciones de la migración espermática: procesos inAamatorios vaginales, endometritis, malformaciones, alteraciones del moco cervical, etcétera .

Anamnesis, exploración y analítica: historia clínica general, ginecológica y sexual de la pa reja . Se toman muestras para citología y se hace un estudio analítico: hemograma y velocidad de sedimentación, bioquímica, orina, g rupo sang uíneo, Rh y serologías frente a rubéola, toxoplasmosis, sífi lis, hepatitis B, C y VI H.

Ecografía transvaginal: inform a acerca del útero, los anejos y las posibles alteraciones morfológicas, así como de la patología endometrial, la endometriosis, los ovarios poliqu ísticos, etcétera. Perfil hormonal: se incluirá la determinación de FSH, LH y estradiol en la fase folicular precoz (2°-4. 0 día ciclo), prolactina y TSH. Existen tres parámetros diagnósticos que presentan una marcada correspondencia con la posibilidad de consegu ir un embarazo:

Confirmación de la ovulación: la técnica de elección es la determinación de progesterona plasmática en mitad de la fase lútea. También se podría valorar med iante la curva de temperatura corporal,

Ginecología y obstetricia fundamentada en el pequeño incremento de temperatura por el efecto de la progesterona. Aunque es un método barato resulta engorroso, poco fiable e incómodo para la paciente.

1

06

6.3. Tratamiento

Seminograma: se estudia el número, la movilidad, la morfología de

Como las causas de infertilidad son múltiples, se debe hacer un trata-

los espermatozoid es y las características fís icas y bioquímicas del se-

miento individualizado para cada caso.

men. El test de capacitación espermática o REM es útil para identifica r el número real de espermatozoides con mejor movilidad, una vez desechados el plasma seminal, los espermatozoides inmóvi les y las células in maduras y detritus que pud iera haber en la muestra.

Fármacos empleados en reproducción asistida

Confirmación de la permeabilidad tubárica: la histerosalpingografía (HSG) es el método de elección. Consiste en la visualización fluoroscó-

Cit rato de clomifeno: fue el primer fármaco utilizado para inducir la

pica y radiográfica de la cavidad uterina y trompas, tras la inyección de contraste opaco, 2-5 días después de la mestruación. Permite valorar

ovulación. Es un antiestrógeno que actúa bloqueando los receptores hipotalámicos e hipofisarios de estrógenos, lo que, a su vez, estimu la

una obstrucción tubárica y ocasionalmente puede resultar terapéutica y repermeabilizar una obstrucción tras su realización (Figura 11 ). Es una

la secreción hipofisaria de FSH. Se emp lea para la inducción de la ovulación en pacientes con ol igoanovulación y niveles de estrógenos normales (ovario poliquístico).

prueba en ocasiones dolorosa y existe posibilidad de reagudización de un absceso tubo-ovárico o enfermedad inflamatorio pélvica antigua (1-3%).

Gonadot ropinas: obt enidas por purificación a partir de la orina de mujeres posmenopáusicas (hMG) o por recombinación genética (rFSH, rLH ), se utilizan para la estimu lación ovárica en técnicas de reproducción asistida, ya que reclutan folículos y estimulan su crecimiento al mismo tiempo que facilitan su maduración. Gonadot ropina corión ica humana (hCG): se administra para desencadenar la ovulación, que se produce unas 34-36 horas después. Análogos de la GnRH : evitan el pico prematuro de LH con la luteinización posterior del folículo que, con frecuencia, ocurre durante la estimulación con gonadotropinas. Existen dos tipos: los agonistas, que administrados de forma continua provocan una liberación inicial de gonadotropinas (efecto flare-up), que va seguido de un bloqueo reversible de la liberación hipofisaria de FSH y LH, y los antagonistas, que producen un bloqueo reversible del receptor de GnRH mediante una unión competitiva del mismo, sin activarlo.

Figura 11. Histerosalpingografía que muestra obstrucción tubárica bilateral

El control del crecimiento fol icu lar se hace mediante determinaciones seriadas de estrad iol y ecografías.

La siguiente seri e de pruebas no se realizará de manera rutinaria, sino en func ión de la sospecha diagnóstica:

Laparoscopia: es complementa ria a la HSG. Es muy útil en el diagnóstico y tratamiento de adherencias y de endometriosis. Test poscoital: se lleva a cabo para descartar incompatibilidad del moco con los esperm atozoides. Sólo se usa en parejas jóvenes y con historia corta de esteri lidad.

Histeroscopia: perm ite visua liza r la cavidad uterina y, en ocasiones, tratar la alteración: en casos de pólipos, m iomas, sinequ ias, tabiques uterinos, etcétera. Está indicada en pacientes con abortos de repetición o con partos pretérmino, también en pacientes diagnostica-

Técnicas de reproducción asistida Las técnicas de reproducción asistida son las siguientes:

Inseminación artificial conyugal: es la técnica más simple y más usada. Se necesita que las trompas sean permeables y que en el recuento de espermatozoides móviles (REM) se recuperen más de tres millones de espermatozoides móviles progresivos poscapacitación. Consiste en inducir la ovulación e introducir con una cánula el semen capacitado del cónyuge dentro del útero. Las indicaciones

das de alteraciones en la cavidad uterina por histerosalpingografía o

serían: oligoastenoteratozoospermi a, incapacidad para depositar semen en la vagina, disfunciones ovulatorias, factores cervicales, facto-

cuando no se encuentra otra causa de esterilidad. Biopsia de endometrio: debe realizarse premenstrualmente. Aun-

res uterinos y esterilidad de origen desconocido. Inseminación artificial con semen de donante: es sim ilar a la an-

que antiguamente era obligada, hoy su uso está disminuyendo.

terior, pero el semen que se usa procede de un banco de semen. Útil

Determinación de anticuerpos antiespermáticos.

en casos de azoospermia y de mujeres sin pareja.

Cariotipo: se realiza en el caso de sospecha de anomalía cromosómica de los progenitores (azoospermia, abortos de repetición, hijos

Fecundación in vitro (FIV): tras inducir a la ovulación, se recogen ovocitos mediante punción folicular vía vagina l con control ecográfico (raramente con control endoscópico). Se ponen en contacto los

previos con aneuploidías, etcétera). No constituye una prueba imprescindible en el estudio básico de esteril idad.

Detección de mutaciones de la fibrosis quística: es obligatoria en varones que presentan azoospermia obstructiva y se demuestra age-

ovocitos extra íd os con los espermatozoides y, una vez conseguida la fecundación, se transfieren no más de tres embriones.

nesia de los conductos deferentes. En va rones con recuentos espermáticos muy bajos se deben determinar también las microdeleccio-

Los embriones no transferidos son criopreservados y transferidos en ciclos posteriores. Puede aumentar la incidencia de gestación ectópica o de abortos, pero no aumenta el riesgo de malformaciones

nes del cromosoma Y o la delección del brazo corto del cromosoma Y.

congén itas. Está indicada en caso de:

Manual CTO de Medicina y Cirugía, 2. a edición Pa tolog ía tubárica bilatera l, como obstrucción de las trompas. Insuficiente número de espermatozoides pa ra rea lizar inseminación intrauterin a. En caso de fracaso de insemi nación int rauterina tras cuatro o seis intent os.

La FIV no aumenta el ri esgo de malforma ciones congénitas.

La microinyección esp ermática (ICS I): consiste en inyecta r un único espermatozoide dentro del ovocito. Es una va ri ante de la FIV que está indicada en casos de oligospermi a severa, incluso extrayendo los esperm atozoides directamente del

La apa ri ción del SHO está re lac ionada con la aplicac ión de hCG que ejerce su acción a través de med iadores que induce n un incremento en la perm ea bilidad vascular (e l más estudi ado de todos es el VEGF [Vas-

cular Endothelial Growth Factor]). El SHO es autolim it ado, con una regresión espontá nea en ausencia de gest ación . Su evo lución depende de los nive les sérico s de hCG, pud iendo d isting uir entre un SHO temprano (tres y siete días des pu és de la ad ministración), q ue se relaciona con la respuesta ová ri ca y depende de la ad ministración de hCG exógena, y ot ro ta rd ío que aparece ent re 12 y 17 d ías después de la adm inistración de hCG y se relaciona con la gestación (que pro longa o agrava el síndrome inicial) y depend e de la hCG endógena prod ucida por el sincit iotrofoblast o. Los primeros síntomas q ue pueden aparecer en los últimos días de la estimulación ová rica so n ascit is y crecim iento ová ri co. Los síntomas se

testículo. También lo está en caso de fallo de FIV o de mala ca lidad de los ovocitos. Esta técnica permite realiza r un diagnóstico genético

hace n más frecuentes e intensos a las 48 horas de la ad mi nistración de hCG y consist en en ná useas, vó mitos, disten sió n y dolor abdomi nal que pueden progresar rápidamente y llegar a ser graves, provocando

preimplantatorio en los embriones, seleccionando embriones cromosóm icamente sa nos o no afectados de enfe rmedades genéticas (por

fracaso renal, tromboem bolismos, SDRA y poner en ri esgo la vida de la paciente.

ejemplo, fib ros is quística, hemofilia, atrofia muscula r espinal, etcétera).

Síndrome de hiperestimulación ovárica {SHO) El síndrome de hiperestimulación ovárica (SHO) es una complicación que se produce durante la fase lútea del ciclo y que consiste en una respuesta anor-

Como ya se ha comentado, la evolución del cua dro depende de la presencia o no de gestación. En ausencia de em barazo, la mejoría comienza a los dos o tres días de iniciarse y desapa rece con la menstru ación.

complicación iatrogénica ocasionada por el uso de hormonas para la estimulación de la ovulación y desencadenada tras la administración de hCG. Se estima que tiene una incidencia del 10%. Los factores de riesgo son los

El tratam iento debe ser individualizado y multidiscipli nar. Las medidas reco mendables son: reposo relativo, hidratación adecuada, analgesia y antieméticos, vigilancia de los signos de agrava miento y monitorización analítica. En los casos de SHO grave puede ser necesario un abordaje qu irúrgico. Ant e todo se ha de evita r la gest ación, ca ncelando la inseminación artifi cial o criopreservar em briones para t ra nsferencia en ciclos

siguientes: edad joven, historia previa de SHO, SOPe historia de alergia.

posteriores.

malmente elevada de los ovarios a la estimul ación hormonal que persiste y se prolonga. Esta respuesta se presenta casi de manera excl usiva como una

Ideas clave Ji!5 "

"

En un 40% de los casos, el factor implicado en la esterilidad es el masculino; en otro 40%, es el factor femenino (la mayoría de los casos por alteració n tubá rica) y en un 20%, es un factor mixto o de origen desconocido. En el estudio de esterilidad es obligatorio la anamnesis, una exploración y una ana lítica, una ecografía transvagina l (evaluación de morfología AGI y un diagnóstico de posibles patologías como endometriosis o síndrome de ovarios poliquísticos), una ana lítica hormonal (en primera fase y en segunda fase), un seminograma (número, movil idad y morfología de espermatozoides, así como test de capacitación espermática) y una histerosa lpingografía (morfología de cavidad uterina y permeabilidad tubárica).

Case Study .. A 29-year-old woman was referred to hospital two days after embryo transfer beca use of increased abdominal girth and dyspnea of three days duration. The patient' s weight was 80 kg and height was 162 cm. The stimulation was performed using a long protocol with follitropin alfa. Full blood count showed raised hemoglobin (16.1 g/ dl) and hematocrit (47%), as well as thrombocytosis (459,000/!JL); her coagulation screen was normal. Abdominal ultrasound revealed moderate volume ascites, bilateral pleural

06 · Esterilidad e infertilidad

"

Para la rea lización de inseminación artifi cial es necesario que las trompas sean permeables. Es útil en casos de problemas ovulatorios, cervica les o vagina les, así como en casos de impotencia mascu li na o pobre calidad espermática.

"

La fecundación in vitro está indicada en casos de patología tubárica bilateral, número insuficiente de espermatozo ides para realizar inseminación artificial o tras el fracaso de varios intentos de insem inación.

"

La microinyección espermática es una variante de fecundación in vitro que consiste en la inyección de un único espermatozoide dentro del ovocito. Se utiliza en casos de oligoespermia grave, en el fracaso de FIV o si se quiere realizar diagnóstico genético preimp lantatorio.

effusions and an enlarged, multi-cystic right ovary (15 x 8 cm) . The most likely diagnosis and treatment is: 1) 2) 3) 4)

Mucinous cystadenoma: exploratory laparotomy. Mucinous cystad enoma: Gn RH anta gonist. Normal ovari an stimulation: conservative treatm ent. Hyperstimulation syndrome: conservative treatment with rest and anti-coagulation t herapy with low molecular weight hepa ri n.

Correct answer: 4

Ginecología y obstetricia



Endometriosis

ORIENTACIÓN

ENARM

Este tema relevante en forma de caso clínico.

A más menstruación, más riesgo de endometrio sis: menarqui a precoz, cic los cortos, menstru ación abundante

La endometriosis consiste en la presencia y prol iferac ión de tejido endometrial fuera de la cavidad uterina. Se puede encontrar endometriosis en

7 .3. Localización

cualqu ier tejido de la anatom ía. La adenomiosis (endometriosis miometrial) debe considerarse apa rte, ya que es muy frecuente y asintomática. La loca lización más frecuente de la endometriosis es en el ovario. Habitualmente en relación co n la descamació n del tejido endometria l a este

7 .1. Epidemiología

nivel, como respuesta a las mod ificaciones hormonales del ciclo, se forman quistes que se llenan de sangre (se trata de tejido endometrial que responde hormonal mente por lo que, al igual que durante la menstrua-

Aunque la preva lencia varía mucho según los autores, afecta aproxima-

ción, se produce la descamación del endometrio eutópico, se descama

damente al 1Oo/o de las mujeres. Los factores de riesgo son los siguientes:

también el endometrio ectópico de los quistes) y, ese conten ido marrón

Edad fértil.

oscuro, parecido al chocolate, hace que se les llame "qu istes de chocola-

Ciclos cortos (menores de 27 días) con sangrado menstrual abun-

te': Le siguen en frecuencia de localización: ligamentos uterosacros, fosa

dante (mayor de siete días) favorecerían el reflujo de sangre a la ca-

ovárica peritonea l, peritoneo del fondo de saco de Douglas y tabique rec-

vidad peritoneal.

tovag inal.

El tabaco pa rece tener un efecto protector sobre la endometriosis porque dism inuye el estrad iol. No hay una re lación clara con la raza.

7 .2. Etiopatogenia

7 .4. Clínica La clínica de la endometriosis es la sigu iente:

Dolor (95%): es el síntoma más característico y frecuente en la endometriosis. Su intensidad es muy variable. Suele estar localizado La etiopatogenia continúa siendo desconocida. Las teorías más acepta-

en la pelvis y su forma más característica de presentación es como

das sobre la patogénesis son las sigu ientes:

dismenorrea progresiva que no cede a la toma de anticonceptivos

Teoría del desarrollo in situ: la endometriosis puede originarse en

orales. También puede apa recer dolor con las relaciones sexuales

el sitio en el que se diagnostica. Esto puede ocurrir a partir de restos

(dispareunia).

mü llerianos o del cond ucto de Wolff, o también puede ser debido a

Alteraciones menstruales (65%): como ya se ha visto, la menar-

metaplasia del tejido peritoneal u ovárico.

quia se adelanta, los ciclos son cortos y con sangrado abundante y

Teoría de la inducción: se basa en la diferenciación de células mesenquimales, activada por sustancias liberadas por endometrio que degenera, tras llegar a la cavidad abdomina l.

la regla es dolorosa. Infertilidad (41 %): es un motivo frecuente de esteril idad. Las causas son varias y poco conocidas: ciclos anovulatorios, elevación excesiva

Teoría del trasplante o del implante: está basada en el trasplante y

de LH con retraso en el aumento de progesterona, alteraciones in-

posterior implante del tejido endometrial. Se produciría por el trans-

munitarias, obstrucción tubárica, etcétera.

porte de células endometriales viables durante la menstruación de

Otros. Abdomina les: distensión abdominal, rectorragias. Urinarias:

forma ret rógrada a través de las trompas de Falopio hacia la cavidad

disuria, etcétera. Puede producir una elevación moderada de Ca-125

abdominal, el implante de estas células en el peritoneo y, a continua-

sérico. También es posible la aparición de neumotórax espontáneo

ción, el desarrollo de la endometriosis.

durante la menstruación (catamenia l), cefa leas, etcétera.

Manual CTO de Medicina y Cirugía, 2.a edición

Además, en el mismo acto lapa roscópico, se puede llevar a cabo un traEndometriosis es IGUAL a dolor (dismenorrea, dispareunia .. .).

tamiento quirúrgico. La endomet riosis se clasifica en cuatro estadios, de acuerdo con una puntuación que va lora la presencia de lesiones y de adherencias en el perito-

7 .S. Diagnóstico

neo, el fo ndo de saco posterior, los ovarios y las trompas, siendo el est adio 1, mínimo; el ll, leve; elll l, moderado, y el IV, grave.

El diagnóstico se sospec ha por la clíni ca. En el tacto bimanual encon-

7 .6. Tratamiento

tramos como datos compatibl es con endometriosis el dolor pélvico, con útero fijo, ligamentos uterosacros do lorosos y ovarios aumentados de tamaño. La ecografía-Doppler pos ibi lita eva luar las características de la pelvis, pero, actualmente, el diagnóstico de certeza de la en-

El tratamiento de la endometriosis se puede llevar a cabo de la siguiente fo rm a:

dometriosis es por laparoscopia, que p resenta las siguientes ventajas:

Laparoscopia: es el tratamiento de elección de la endometriosis.

permite visualizar directamente las lesiones, que t ienen aspecto de

Permite resolver gran parte de las lesiones. Debe ser un tratam ien-

quemadura de pólvora, es posible va lorar el resto de l aparato genital, realizar la estad ifi cación y tomar muestras para estudi o anatomopato-

to conservador, cuyo objeti vo es la restauración de la anatomía. Se toman biopsias y citología de líquido peritoneal, se rea liza quistecto-

lóg ico (Figuras 12 y 13).

mía, se extirpan o se destruyen los implantes peritonea les, se rea liza lavado peritonea l final y, en ocasiones, se usan sustancias antiad herentes pa ra los anejos. Los medios de sección y fulguración son los más efi caces y los menos lesivos. Cirugía radical: est á indicada cuando la enfermedad no ha podido ser controlada con tratamiento qu irúrgico conservador ni con tratamiento médico. Por ejemplo, dolor resistente al trata miento e invalidante, afectación de otros órganos: intestino, vías urinarias, etcétera. En este caso, se practicará hist erectom ía t otal con doble anexectomía. Tratamiento médico: se basa en la idea de que la endometriosis precisa un ambiente hormonal de estrógenos y también de progestágenos (por ello, no apa rece antes de la menarquia, desaparece tras la menopa usia y au menta al comienzo del embarazo). Los fá rm acos más usados so n: Anticonceptivos orales: son bien tolerados. Producen inhibición cent ral y decidual ización local. DIU-Ievonorgestrel: reduce la recurren cia de los periodos do-

Figura 12. Ecografía transvaginal de quiste endometriósico ovárico

lorosos en las mujeres que han sido sometidas a cirugía por la endometriosis. Análogos de la GnRH: crean un ambiente hipoestrogénico que deja quiescent es las lesiones endometriósicas, pero su uso pro longado favorece la atrofi a genitourinaria y la disminución de la densidad mineral ósea. Danazol : provoca un ambiente hipoestrog én ico que favorece la atrofia endometri al pero produce efectos adversos androgénicos que lim itan su util idad clínica. Gestágenos: pueden ocasionar aumento de peso y hemorragia interm enstrual, sobre todo cuando se adm ini stra n a dosis altas necesa ria s para inducir atrofia endometrial.

7.7. Endometriosis y cáncer La endometriosis es una enfermedad benigna. En los raros casos de en-

Figura 13. Diagnóstico de endometriosis por laparoscopia

07 · Endometriosis

dometriosis atípica, sí hay un potencia l precanceroso. Los tipos histológicos que se asocian a est a con mayor frecuencia son el ca rcinoma endometrioide de ova ri o y el ca rcinoma de célu las claras.

Ginecología y obstetricia

Ideas clave "

" La localización más frecuente es en el ovario, en el que aparecen quistes de contenido hemático antiguo, lo que le da el aspecto de "q uistes de chocolate': Tras el ovario, se suelen afectar el ligamento ancho y los uterosacros, si bien puede aparecer a cualquier nivel. "

"

El diagnóstico de sospecha se hará por la clínica y por la ecografía, en la que es habitual la visualización de los quistes endometriósicos, si bien el diagnóstico definitivo es a través de laparoscopia, que además permitirá tratar a la paciente.

"

El tratamiento, aunque es quirúrgico, debe ser lo más conservador posible, ya que suele tratarse de mujeres jóvenes y, en muchos casos, con problemas de esterilidad asociados. Por ello, estará indicado la realización de quistectomía, liberación de adherencias y destrucción de los implantes peritoneales. La cirugía radical se utilizará en aquellos casos que no se pueden controlar con el tratamiento quirúrgico conservador y/o médico (análogos GnRH, anticonceptivos hormonales, gestágenos, danazol. .. ).

El dolor es el síntoma más frecuente y característico. Generalmente se manifiesta en forma de dismenorrea progresiva que no cede con la toma de antiinflamatorios ni con anticoncepti-

Casos clínicos Paciente de 30 años, nuligesta, con última regla hace 20 días, que acude a la consulta por presentar esterilidad y dismenorrea importante. En las exploraciones que se realizan, destaca tumoración anexial en la ecografía y elevación moderada de Ca-125 sérico. ¿Cuál es el diagnóstico de presunción? 1) 2) 3) 4)

Embarazo ectópico. Cáncer de ovario. Mioma uterino. Endometriosis.

01

vos hormonales. Otros síntomas que pueden aparecer son: alteraciones menstruales, rectorragia, disuria, infertilidad . ..

~

La etiología de la endometriosis aún no está clara, si bien existen tres teorías aceptadas: desarrollo in situ, inducción, trasplante o implante.

1

Una paciente de 32 años, con un cuadro clínico de sangrado menstrual irregular, dismenorrea, dispareunia, y una esterilidad de 3 años de evolución, presenta una analítica hormonal normal y un estudio ecográfico transvaginal que informa de un útero normal y sendas formaciones quística ováricas bilaterales de 4 cm con signos ecográficos de sospecha. ¿Cuál sería la orientación diagnóstica? 1) 2) 3) 4)

Síndrome adherencia! pélvico. Hemorragia uterina disfuncional. Síndrome del ovario poliquístico. Endometriosis.

RC: 4 RC: 4

Case Study A 41-year-old woman with no significant past medical history was presented to our hospital because of six months of recurrent abdominal pain, vomiting and diarrhea. She also presents dysmenorrhea, dyspareunia and polymenorrhea and menorrhagia. Physical examination revealed a palpable 3 cm x 4 cm mass in the right lower quadrant abdomen. Laboratory test revealed slightly elevated CA 19-9 and CA 125 levels. Transvaginal ultra so-

nography showed a forced retrotlexed uterus with a retrocervical nodule depending on the right ovary. No other abnormalities were found. The most likely diagnosis is: 1) 2) 3) 4)

Polycystic ovary syndrome. Endometriosis. Bilateral ovarian dermoid cyst. Adherential syndrome.

Correct answer: 2

Ginecología y obstetricia



Infecciones ginecológicas en vagina y vulva

ORIENTACIÓN

ENARM

por la flora local. Aunque en un principio es posible hacer un tratamien-

Este tema es poco importante. Hay que estudiar sobre todo la candidiasis, la tricomoniasis y la vaginosis bacteriana.

to méd ico con fluorquinol o na s o cefa losporinas más metronidazol, con frecu encia requ iere tratam iento qu irúrg ico, que puede co nsistir en marsupialización de la g lándula: se sutura la pared del qu ist e a la pared vestibu lar, o bien se puede realizar incisión y drenaje simple de la g lándu la. Se practicará la exéresis glandular tota l en caso de recidivas.

8.1. Definición y conceptos

8.2. Etiología

Las molestias vulvovag inales son un m otivo de consu lta muy frecuente por la elevada incidencia de las infecciones ginecológ icas en vagina y en vu lva.

Más del 90% de estas infecciones están causadas por uno de estos t res microorganismos (Tabla 2):

La vagina es un conducto carente de glándu las propias que está tapizado

Gardnerel/a vaginalis (40-50%): el más frecuente, aunque muchas

por epitelio plano poliestratificado. El flujo vagina l normal es blanco, no ho-

veces pasa inadvertido, por lo que se diagnostica en menor número

mogéneo e inodoro. Proviene del drenaje en la vagina de diferentes glándu-

de casos que la cand idiasis.

las (glánd ulas vestibulares, g lándulas de Bartholino, del cérvix, del endome-

Candida (20-25%): la más sintomática, de ahí que sea la que más se diagnostica. Trichomona vaginalis (15-20%): es conside rada una enfermedad de trasmisión sexua l (ETS).

trio y del endosálpinx), del líqu ido trasudado de la pared vaginal y de células vagi nales descamadas. Las hormonas sexuales (estrógenos y progesterona) influyen en las características de la secreción y de la flora vagina les.

Dóderlein es el responsable de acid ificar el medio puesto que produce

El 1Oo/o restante incluyen : vag initis por cue rpo extraño, vag initis atrófica, Chlamydias, gonococos, micoplasmas, herpes gen ital, parásitos (oxiuros

ácido láctico haciendo que el pH sea de 4 en la mujer en edad fértil. En

en niñas), vag initi s alérgicas, etcétera. Si existe ulceración vu lvar, se debe

la deficiencia de estrógenos (por ejemplo, en la menopausia), el pH suele

pensar en: sífili s, herpes, chancro ide, linfogranuloma venéreo, granuloma

La vagina en co ndiciones norma les tiene un med io ácido. El bacilo de

ser superior a 6. La flora vag inal norma l está constituida por microorga-

ingu inal, sobreinfección de condiloma, molluscum contagiosum, sarna,

nismos aerobios y anaerobios. En ella es posible encontrar microorganis-

traumatismo, neoplasia, eritema multiforme o síndrome de Beh 4,5

Positivo

en edad fértil, en la púber y en la posmenopáusica, el pH asciende, permitiendo la génesis de las infecciones va-

Negativo

ginales graves.

Ami nas volátiles con KOH (1 Oo/o)

Ocasional

Esporas

Polimorfonucleares tricomonas

"clue-cells" cocobacilos

Se considera aparte la barthol initis,

Microscopia con suero salino

cuya causa más frecuente es la obs-

Tratamiento

Metronidazol oral

Metronidazol o clindamicina

trucción del canal excretor de la g lán-

Derivados imidazólicos tópicos

dula, una infección que se produce

Tabla 2. Diferencias entre las vu lvovaginitis

Ginecología y obstetri cia

8.3. Gardnerella vagina lis ( vaginosis bacteriana)

1

08

encuentra en gran número, orig inan la vaginitis sintomática. La cand idiasis se produce por la proliferación de patógenos oportunistas ante cambios en el medio vagina l, en la flora bacteriana local o en la resistencia del huésped. Se consideran factores pred isponentes: niveles altos de estrógenos (emba-

Es necesario recordar que es la infecc ión vagina l más predom inante en

razo, anticonceptivos orales), diabetes, uso de co rticoides o antibióticos de

mujeres en edad reproductiva de países desarrollados. Resu lta de la sus-

amplio espectro. Las pacientes con VIH t ienen ca ndidia sis vaginales más

titución de la flora vag inal saprofita no rm al por altas concentraciones de

graves y recurrentes.

Gardenere/la vaginalis, Micoplasma hominis y bacterias anaerobias.

Diagnóstico Clínica El pH vagina l no suele variar (entre 4 y 4,5). Ap roximadamente un S0-70% de las muj eres co n una vag inosis están

Frotis en fresco: presencia de m ice lios o de esporas.

asintomáticas. El resto refiere un au mento del fluj o, de color blanco-gri-

Cultivo vag inal: es el m étodo m ás sensible y específico (medio Sa-

sáceo, y, en ocasiones, maloliente. No se asocian signos inflamatorios de

boureud).

edema o eritema vulvar. No se trata de una infección de transm isión sexual. Son factores de ri esgo la gestación previa y el uso de DIU.

Tratamiento

Diagnóstico

Lo s co mpuestos azólicos son el tratamiento de elección : Vía vaginal: clot ri mazol (de elección en la em barazada), miconazol,

El pH vaginal es superi or a 4,5.

ketoconazol, fentico nazol y sertaco nazol.

Pru eba de am inas: positiva . Consiste en añadi r en un porta con una

Vía oral: el itraconazo l, fluconazo l y ketoconazol.

muestra de fluj o vag inal una got a de hidróxido potásico al 10%. Inmediatamente se produce la liberación de am inas que co nlleva a la

La vía más usada es la vag inal. La vía ora l tiene la ventaj a de llegar al re-

aparició n de olor fétido.

servo ri o intestinal, si bien está contraindicada en el embarazo. Se reserva

Frotis con tinc ión de Gram : presencia de las clue-cells (fijación de co-

para las rec idivas o los casos con afectación digestiva asociada. El trata-

cobacilos en la superficie de las cé lulas vag inales).

miento de la pa reja se rea lizará si tiene síntomas o en casos de candidiasis recidivante (cuatro episod ios anuales o más). En estos casos está

Tratamiento

recomendado hacer un tratam iento supresor de seis meses de du raci ón.

La importancia de la vaginosis bacteriana se debe a las complicaciones con

Parece útil la ingesta de yogu res que contengan fermentos lácticos vivos

las que se relaciona. En el emba razo puede estar relacionada con el parto

(Lactobacillus acidophilus).

pretérmino, la rotura de membranas y la endometritis pos parto o postaborto. En mujeres no gesta ntes es posible que produzca un incremento en el riesgo de padecer una enfermedad inflamatoria pélvica y otra s infecciones,

8.5. Tricomoniasis

principalmente de transm isión sexual, inclu ido el VIH. Estará indicado trata r a las mujeres sintomáticas, a las gestantes y a las muj eres asintomáticas si va n a ser sometidas a intervenc iones ginecológicas (inserción DIU, biopsia

Clínica

endometrial, leg rad o, histeroscopi a, etcétera). No es necesario tratar a la pa reja de manera rut inaria pero sí en casos de recidiva.

Se trata de una infección por transmisión sexual no víri ca con elevada preva lencia. Est á producida por un protozoo anaerobio y no es infrecuen-

Existen varias pautas: Metronidazol vía oral siete días. Se cons idera el tratamiento de elección.

te que se asocie a otros m icroorga nismos co m o el gonococo y/o ch/amydias. Cursa co n un prurito vulva r intenso, escozor y un aumento del

Metron idazol ora l monodosis.

fluj o vagina l. La leucorrea típica es de co lor amarillo-grisáceo, espumosa

Clindam icina en gel siet e d ías.

y de mal o lor. Es frecuen te la presencia de disuria . En la exploración gi-

Clindamicina ora l siete días.

necológica es posible encontrar una vagina ligeram en te enrojecida y un punteado rojo en vag ina y cérv ix (pequeñas hemorragias subepitelia les).

8.4. Candidiasis

Diagnóstico El pH vaginal suele ser superior a 4,5 .

Clínica

Frotis en fresco: se visua liza el protozoo que tiene forma de pera con cilios

En el 80-90% de los casos, están producidas por Candida a/bicans y el síntoma

Cultivo vaginal: es el método definitivo para el diagnóstico, ya que es

principal es el picor y el escozor en la vulva y/o en la vag ina. La leucorrea tiene

positivo en el 95% de los casos.

en un extremo y, en muchas ocasiones, puede observarse su movilidad.

unas características muy típicas, como el color blanco y la co nsistencia pastosa (parecida al yogur), que hace que se ad hiera a las pa redes vaginales y a la vulva.

Tratamiento

Puede asocia r molestias urinarias o co n las relacio nes sexua les. No es infre-

El tratam iento de elecció n es el metro nidazol. Pueden utilizarse pautas

cuente que aparezca un enrojecim iento de la vulva. La Candida es un hon-

cortas por su fácil cumplimiento aunque ta mbién es posible que sean

go saprofito aislado frecuenteme nte en el t racto genital inferior pero si se

pautas de siete d ías, si bien no suele aplicarse como t ratam iento inicial

Manual CTO de Medicina y Cirugía, 2. a edición

y se reserva para aquel los casos en que el tratamiento en dosis única no

Infección subclínica: lesiones co lposcópicas acetoblancas (leuco-

ha resu ltado eficaz. Al tratarse de una infección de transmisión sexual, es

plasias) y displasia.

importante el tratamiento de la pareja sistemáticamente para evitar rec i-

Infección clínica: las verru gas o los condilomas gen itales son las

divas. Debe evitarse el consumo de alco hol por su efecto anta bus.

lesiones más frecuentes del aparato genital externo. Consisten en lesiones exofíti cas vu lvares y perianales pru riginosas de consistencia blanda y de co loració n grisácea o rosa da . Los genotipos más fre-

8 .6. Infecciones virales

cuentemente implica dos son HPV-6 y HPV-11.

Diagnóstico Infección por papilomavirus (HPV) A simple vista se observan las típicas lesiones cerebriformes (condiloSe trata de un papoviridae, que es un ADN-virus, del cual se reconocen

mas). En la citología se encuentran co ilocitos (célula epite lial con núcleo

más de 150 tipos vi ral es. El contagio por HPV se produce por microtrau-

engrosado, hi percromático y halo o vacuolización perinuclear). Con la

matismos a través del coito con una persona infectada, por el canal del

colposcopia se aprecian zonas de leucoplasia tras la aplicación de ácido

parto, o por medio de fómites, con un periodo de incubación de entre dos

y tres meses de promedio (osci lando de uno a ocho meses).

acético, que se deben biopsiar. Es posible rea lizar el estud io virológ ico en cortes tisu lares (PCR o hibridación in situ) o sobre extensiones cito lógicas (PCR, hibridación in si tu o ca ptu ra de híbri dos).

Constituye la infección de transmis ión sexua l (ITS) más co mún, siendo la prevalencia de un 10,4% de media en mujeres asintomáticas, con cifras

Tratamiento

más altas (22%) en mujeres jóvenes (20 años); en varones la preva lencia es menor, con va lores máximos en la cua rta década de la vida. Nueve

El tratamiento de los co nd ilomas depende de su loca lizació n y de su ex-

meses después de la respuesta inmun ita ria puede producirse bien una

t ensión, siendo su objetivo casi siempre estético, ya que la infección per-

remisión, o bien una infección persistente. Se estima que sólo un 1% de

manecerá latente.

los portadores de HPV tiene infección clínica. Los factores favorecedores

Tratamiento médico:

de d icha infecc ión son la promiscuidad sexual, la inm unosupresión (tanto

Ácido tricloroacético a concentraciones del 85% en apl icación

por quimioterapia como porVIH), los corticoides, la gestación y el tabaco.

semanal. Remisiones del 30%. Es el tratamiento médico de elección en embarazadas.

Los distintos tipos vira les se dividen en dos grupos: de bajo riesgo onco-

Podofi lotoxina al 0,5%: tres aplicaciones semanales durante seis

lógico (los más frecuentes son los tipos 6 y 11 ), y de alto ri esgo oncológi -

semanas. Remisiones del 70%.

co (los más habitua les son los tipos 16

y 18, que aparecen en el 70% de

lmiquimod al 5%: tres aplicacio nes semana les un máximo de 16

los carc inomas de cérvix) (Tabla 3). Se ha demostrado que en el 99,7%

semanas. Remisiones del60-70%.

de los tumores de cérvix hay ADN de HPV (de ta l modo que la infección

5-fluoracilo.

por HPV es ca usa necesaria, pero no suficiente para el cáncer de cérvix). En las lesiones invaso ras, el ADN viral está habitualmente integrado en los

Tratamiento quirúrgico: extirpación quirúrgica, electrodiaterm ia,

cromosomas de las cé lu las neoplásicas, a diferencia de las lesiones intrae-

crioterapia, láser de C0 2.

pitelia les de bajo grado (cambios por HPV

y displasia leve), en las que a

menudo se encuentra de forma episómica extracromosómica.

Profilaxis Actua lmente se comercializan dos vacunas (bivalente

Tipos de HPV

Grupo

y tetravalente) para

prevenir la infección por HPV, compuestas por partículas similares al virus (VLP, virus-like-particle), producidas mediante clonación del gen principal de

Tipos de alto riesgo oncológico (HPV-AR)

16, 18, 31' 33, 35, 39, 45, 51' 52, 56, 58, 59, 68, 73, 82

la cápside viral (L 1) de los distintos tipos de HPV; por tanto, estas vacunas no

Tipos de bajo riesgo oncológico (HPV-BR)

6, 11,40,42,43,44,54,61,70, 72,81

son infecciosas ni oncogénicas, dado que no poseen ADN. La vacuna bivalente confiere protección frente a los tipos 16 y 18, por consigu iente, está

Tabla 3. Clasificación epidemiológica de los tipos de HPV

destinada a reducir las displasias y los cánceres de cérvix provocados por estos tipos, no protegiendo frente a cond ilomas. La vacuna tetrava lente confie-

La detección de HPV-AR es útil en los siguientes casos: Prevención secundaria en combinación con la citología o como prueba

re protección frente a los t ipos 6, 11 , 16 y 18, por lo que protege frente a condi lomas, además de frente a displasias y cánceres de cérvix por dichos t ipos.

única para la detección de lesiones precancerosas del cáncer de cuello. Selección de las mujeres con citologías que presentan alteraciones meno-

La adm inistración de ambas vacunas es intramuscu lar. La vacuna se pue-

res que requieren reevaluación para diagnóstico y eventual tratamiento.

de adm inistrar a mujeres con cito log ía anormal o PCR-HPV positiva o

Prueba de seguimiento en la mujeres tratadas por lesión intraepite-

cond ilomas, a sabiendas que la efectividad no está demostrada (y sólo

lial escamosa (SIL) de alto grado, para así poder predecir las posibi li-

protegería frente a los otros tipos de HPV). La vacunación en varones está

dades de curación o de recidiva.

todavía en estudio, sugiriendo que supond ría una menor trasmisión a mujeres

y una d ism inución del carc inoma anogen ital.

Clínica Las diferencias entre ambas vacuna s, además de que una de ellas contenEs posible encontrar va rios tipos de infecció n

y de lesiones:

y la inmunogenicidad y co ntinúan siendo motivo de estud io para determinar la

ga dos t ipos más, re siden en el adyuvante utilizado

Infección latente: ADN viral posit ivo sin ningún t ipo de lesión cito-

que provocan

histológ ica.

necesidad de dosis de recuerdo cada cierto número de años. Las muje-

08 · Infecciones ginecológicas en vagina y vulva

Ginecología y obstetricia res vacunadas con la vacuna bivalente presentan una protección cruzada contra los tipos 45 y 31.

1

08

Las vacunas han demostrado ser seg uras, in munógenas y muy efi caces pa ra la prevención de la infecc ión, del CI N 11-111 y del carcinoma in situ producidos por los ti pos vira les incluidos en la vacuna; tam bién serían efi-

tan largo de esta patología. No obstante, no puede detenerse el cribado de lesiones precancerosas al introducir esta vacuna, dado que no protege frente a todos los t ipos de HPV aunque sí podría cambiar las estrategias de cribado del cá ncer de cérvix, siendo la prueba clave la detección de HPV, en lugar de la citología cervicovagi nal hasta ahora, quedando relegada esta últ ima a pacientes con HPV+.

caces para la prevención de verru gas genitales y otras lesiones gen ita les externas como el Va lN y el VIN.

Infección por herpes simple {VHS)

La reducción del impacto del cá ncer de cé rvix en la población vacunada no se ve rá hasta dentro de 25-30 años, debido al periodo de incubación

Véase Enfermedades de transmisión sexual en la sección de Enfermedades infecciosas.

"

El diagnóstico se realizará mediante el cultivo, si bien un dato que permite diferenciar la infección por Candida de otras vulvovaginitis es el hecho de que el pH vaginal no se modifica(< 4,5).

La causa más frecuente de infecciones vulvovaginales es la Gardnerella vaginalis. Sin embargo, la Candida es la que más síntomas produce.

"

La infección por Gardnerella produce un flujo maloliente grisáceo, pero sin embargo es poco habitual que produzca prurito. El diagnóstico de la vaginosis bacteriana se puede hacer mediante la prueba de las aminas, que será positiva, y por la visualización de las "c/ue-cel/s '~

El t ratamiento de elección consiste en pautar derivados imidazólicos por vía vaginal (clotrimazol), aconsejándose el tratamiento por vía oral en casos recurrentes. La pareja se tratará si presenta síntomas.

"

La infección por tricomoniasis suele ser asintomática. La leucorrea es abundante, maloliente y espumosa . En ocasiones es posible que se visualicen hemorragias puntiformes en el cérvi x.

"

El diagnóstico se realiza por la visualización del parásito en fresco.

"

El tratam iento de elección es metronidazol vía oral.

"

Los facto res favorecedores de la infección por papilomavirus (HPV) son la promiscuidad sexual, la inmunosupresión, los corticoides, la gestación y el tabaquismo. Hoy se conoce que esta infección es un cla ro factor de riesgo de aparición de lesiones premalignas y carcinoma de cérvi x.

Ideas clave J?S "

"

"

"

En el tratamiento, se puede utilizar metronidazol como fá rmaco de elección o clindamicina . Es obligado el tratamiento de todas las gestantes por el riesgo de parto pretérmino. No es necesario el tratamiento de la pareja de forma rutinaria, aunque sí está recomendado en casos de recidiva . La infección por Candida produce mucho pru rito asociado a leucorrea blancoamarillenta grumosa y viscosa. Son factores de riesgo: embarazo, anticoncepción hormonal, diabetes, corticoi des o antibióticos de amplio espectro.

Casos clínicos Una mujer de 38 años consulta por una secreción vaginal anormal con mal olor, como a "pescado podrido'; que se hace más intenso después del coito. No tiene prurito genital. En la exploración se observa una abundante secreción blanco-grisácea, que no se adhiere a las pareces vaginales. Al mezclar una muestra de la secreción con una gota de hidróxido potásico al1 Oo/o, se apre-

Case Study A 28-year-old woman presents symptoms of vulva! prutitis and profuse vaginal discharge for severa! days. She was not taking regular medication. On examination, the vulva looked healthy but the vagina was erythematous and white plaques were noted. The cervix appeared normal and bimanual pelvic examination was unremarkable. The most likely treatment for this patient is:

cia claramente el mal olor referido. ¿Cuál de los siguientes es el tratamiento de elección? 1) 2) 3) 4)

Am oxicil ina con ácido clavul ánico por vía oral. Clotrimazol por vía intravag inal. Doxiciclina por vía ora l. Cl inda micina por vía intravagi nal.

RC:4

1) 2) 3) 4)

Vaginal cotrimazole. Oral fluconazol e. Vag ina l metronidazole. Ora l clindamycin.

Correct answer: 1

Ginecología y obstetricia

1

Infecciones pélvicas

llHií:NTACIÓN

ENARM

Tema poco importante, así que hay que centrarse sobre todo en la enfermedad inflamatoria pélvica.

Clínica El sínto ma m ás frecuente es dolo r abdominal bajo, que habit ualmente es bilatera l y asociado a leucorrea. Sínto mas co ncurrentes suelen ind icar infección de otras estructuras anatómicas. As í, por ej em plo, puede aparecer d isuria como consecuencia de una uretrit is.

9.1. Enfermedad inflamatoria pélvica

Otros síntomas sistém icos, co m o fi ebre y prese ncia de náuseas, con vómitos o sin ellos, señalan inflamación perito nea l y una afección clínica más grave.

La enfermedad inflamato ri a pélvica (EIP) es una infección de útero, trompas y ova rios debida a una infección bacteriana ascendente desde el t rac-

Diagnóstico

to genita l inferior: los gérmenes pe netran por la vag ina y ascienden hacia la pelvis.

El d iag nósti co de EI P es funda m enta lment e c líni co: o ri enta la presencia de do lo r abd o m inal baj o, fi eb re y leucorrea . Para el citado diag nós-

Etiología

ti co se req uiere la presencia d e t odos lo s cr it erios m ayores y al menos

La EIP está causada por gérm enes q ue se transm iten a través de las rela-

q ue este d iag nóstico clíni co es poco sens ib le (65%) y poco específico

ciones sexua les. La ca usa m ás frecuent e es: Chlamydia trachoma tis segui-

(65%).

uno de los crite ri os m enores, q ue se d escriben en la Tabla 4, au n-

do de Neisseria gonorrhoeae. En algu nos casos de portadoras de DIU co n EIP, se aisla Actynomices israelii. Criterios mayores

Estos gérmenes prod ucen inflamación del endocérvix (endocervicit is).

Criterios menores

Historia o presencia de dolor en abdomen inferior

Temperatura mayor de 38 °C

Dolor a la movi lización cerv ica l en la exploración

Leucocitosis mayor de 10.500

un papel en la co nta m inació n de las t rompas de fa lopio y del perito neo. Hasta en un 40% de pacientes co n sa lping itis aguda se ha ident ifi cado

Dolor anexial en la exploración abdom inal

VSG elevada

Ecog rafía no sugestiva de otra patología

Gram de exudado intrace rvica l demostrand o d iplococos intracelulares sugestivos de gonococo, cultivo positivo para N. gonorrhoeae o cultivo positivo u observació n al examen d irecto IFD de Chlamydia trachomatis

Con la d isem inación ascendente, estos pat ógenos provoca n infl amación en todo el tracto gen ita l superior (endometrit is). También puede te ner

una infecció n polimicro biana.

Factores favorecedores Los factores favorecedores de la EIP son los sig uientes:

ETS: co nst ituye el mayor factor de ri esgo. DI U: las porta doras de DIU ti enen una incidencia entre dos y nueve veces mayor de EIP: los hilos del DIU favo recen el ascenso de los gérmenes. No obstan te, cuando se aplica correctamente el DIU en

Tabla 4. Criterios mayores y menores de EIP

mujeres con un sólo compañero sexua l y sin antecedentes de ETS, apenas aumenta la incidencia de EIP

La lapa roscopia es el método d iag nóstico más seguro, aunque no es el

AO: d ism inuyen la incidencia de EIP, ya q ue proporcionan mayor vis-

de elección, por ser m ás caro y com pl ejo. Está ind ica do en pacientes en

cosidad al moco ce rvica l y d ificulta n así la ent rada de gérmenes.

q uienes hay d uda sobre el d iag nóstico o en las que fa lla el t ratam iento

Edad joven (15-39 años) .

méd ico.

Ginecología y obstetricia Tratamiento médico El tratamiento ideal es preveni r la transm isión de ETS. El tratamiento de elección cuando aparece EIP es médico. El objetivo del tratamiento es aliviar el dolor y conserva r la fu nción de las trompas: sólo el diagnóstico y el manej o precoz reduc irán el número de mujeres que sufren las secuelas. Se toman muestras pa ra cu ltivo y se com ienza t ratam iento antibiótico em pírico. Tratamiento ambulatorio en EIP leve-moderada: se recom ienda alguna de las siguient es altern ativas: Ceftriaxona i.m. dosis única y doxiciclina v.o. durante 14 días. Cefotaxima i.m. dosis única y doxiciclina v.o. durante 14 días. Puede agregarse metronid azol v.o. durante 14 días a cualquiera de los ante riores reg imenes para cobertura de anaerobi os.

1

09

9.2. Tuberculosis genital La tubercu losis del tracto genita l es una enferm edad que puede producir EI P crón ica y esterilidad. El bacilo que la ocasiona con mayor frec uencia es Mycobacterium tuberculosis. La infección p ri maria se produce genera lmente en el pulmón, aunq ue puede cu rsa r con rad iografía de tórax normal. Las bact erias se disem inan por vía hematógena hasta el aparato genita l, por lo que es la t rompa habit ualment e la más afectada, ya que lo hace en primer luga r. La tuberculosis genit al apa rece en un 10% de mujeres con t ubercul osis pul monar. A partir de las trompas, se extiende la infecc ión al rest o del aparat o genital, por propagación directa . En la mitad de las pacientes, la enfe rm edad es completa mente asintomática, presentando únicamente esteri lidad o amenorrea.

Tratamiento hospitalario: se pueden admi nistrar algu na de los siguientes protocolos: Ceftriaxo na i.m. y doxiciclina v.o. durante 14 días. Clinda mici na y gentamicina i.v. Los criterios para la hospital ización son: fa llo del tratamiento ambulatorio por fa lta de mejoría tras tres días de tratamiento, incapacidad para el seguimiento o la tolerancia de los antibióticos orales, embarazo, enfermedad grave con fiebre, absceso tubo-ovárico y urgencia quirúrgica que no pueda ser excluida.

Tratamiento quirúrgico

Otros síntomas son los sig uientes: metrorragias, dolor pélvico (35%) o masas anexiales bilaterales. La sa lpingit is t uberculosa debe sospecharse ante una paciente q ue no responde a la antibiot erapia convencional. La oclusión de la cavidad uteri na como sec uela de una endometri tis tuberculosa se denom ina síndrome de Netter. Aproximadame nte el 10% de las m uje res t ienen una tubercu losi s del t racto urinari o concom ita nte a la afectac ió n ge nita l. El d iag nóst ico más adecuado de la t u bercu los is gen ita l es el mi crolegrado uterino (cánu la de Corn ier) El hall azgo clás ico consiste en células gigantes, g ranulomas y necrosis caseosa . El examen d irecto de las trompas

El tratamiento quirúrgico ha disminuido de forma importante. Queda restringido a las infecciones graves que ponen en peligro la vida de la paciente, a los abscesos tubo-ováricos rotos, al drenaje de un absceso en el Douglas o a los

ofrece una imagen de "bolsa de tabaco" con los extremos di sta les

abscesos persistentes. Debe hacerse una cirugía lo más conservadora posible.

El tratam iento debe ser méd ico, con al menos dos antibióticos para evita r res istencias. La pa uta habitual es ison iacida y rifamp icina, durante seis meses, añadiendo etambutol d urante los dos primeros meses (o pirazi-

Secuelas

eve rt idos.

nam ida). El dolor pélvico crónico es la secuela más frecuente (SO%), seguida de la esterilid ad de ori gen tu bá ri co (25%). La tasa de infertil idad aumenta proporcionalmente al número de episodios de EIP, al ig ual que se prod uce un incremento de hasta seis veces en el riesgo de gest ación ectópica. En

La ci rugía se reserva para mujeres con masas pélvicas persistentes, con orga nismos res ist entes y aquellas en las que los cultivos se ma ntienen positivos. Debe hacerse diag nósti co diferencial con neoplasias de cérvix

el 25% de las pacientes, la EIP recidiva.

y de ova ri o, dadas las masas abiga rrad as que aparecen.

serva pa ra los casos en los que haya duda o en los que fracasa el tratamiento médico.

Ideas clave ES "

Los gérmenes que producen con mayor frecuencia la enfermedad inflamatoria pélvica son Chlamydia y Neisseria gonorrhoeae. En las mujeres usuarias de DI U, también es muy frecuente el Actynomices israe/ii.

"

Las enfermedades de transmisión sexual y la presencia de un DIU son los principales factores, además de la edad joven para las infecciones pélvicas. Por el contrario, los anticonceptivos hormonales disminuyen la incidencia de EIP, ya que las modificaciones que producen en el moco cervical dificultan el paso de bacterias por vía ascendente.

"

El diagnóstico es fundamentalmente clínico, por la presencia de dolor abdominal bajo, fiebre y leucorrea. La laparoscopia se re-

"

El tratamiento de elección es médico. Se utilizan diferentes pautas de antibioterapia, tanto por vía oral como por vía intravenosa. Una de las más empleadas es ceftria xona, junto a doxiciclina. La ci rug ía se reserva fundamentalmente para la resolución de los abscesos tuboováricos o pélvicos.

"

La tuberculosis genital es una enfermedad rara en la que, desde un foco pulmonar, se produce una diseminación hematógena que afecta a las trompas, y desde ahí al resto del aparato genital, produciendo esterilidad. En raras ocasiones, pueden aparecer otros síntomas como metrorragias, dolor pélvico, masas anexiales . .. El tratamiento es médico, con pautas similares a las utilizadas en la tuberculosis pulmonar.

Manual CTO de Medicina y Cirugía, 2.a edición

Case Study A 41-year-old woman, gravida 4, para 3, was admitted to hospital due to lower abdominal pain and a history of high fever for four days. Physical examination revealed tenderness in the lower abdomen. Laboratory test demonstrated marked leukocytosis 23,800/mm 3 and elevated (-reactive protein concentration. CA 125 and CA 19-9 were within the normal range. The patient had been an IUD user for six years. She did not have any history

09 · Infecciones pélvicas

of sexually transmitted disease. lndicate the incorrect statement about the di sea se you believe the patient has: 1) 2) 3) 4)

Diagnostic certainty is given by laparoscopy. Treatment is empi rical in this disease. The most frequent complication is sterility. One of the combination s of antibiotics is ceftria xone and doxycycline.

Correct answer: 3

_llinecología y obstetricia

1

Enfermedades de la vulva

ORIENTACIÓN

ENARM

Este es un tema poco relevante para el conjunto de la asignatura.

Tratamiento El tratamiento se lleva a cabo con corticoides tópicos, como la hidrocortisona.

1 0.1. Trastornos epiteliales no neoplásicos

1 0.4. Neoplasia vulvar intraepitelial

Los trastornos epiteliales no neoplásicos son lesiones de la piel y de la muco-

duración de las células epiteliales asociada a hipercromatosis y pleomor-

sa vulvar, cuyo síntoma más frecuente es el prurito crónico. De ellos, se van a

fismo nuclear con acúmulos celulares y mitosis anormales. Se clasifica en

estudiar los siguientes: liquen escleroso e hiperplasia de células escamosas.

los siguientes tipos:

La neoplasia vulvar intraepitelia l (VI N) se caracteriza por la pérdida de ma-

1 0.2. Liquen escleroso

VIN tipo comú n (relacionado con el HPV): de aparición generalmente en mujeres jóvenes. Suelen presentar lesiones multifocales y polimorfas, frecuentemente sobreelevadas y papilomatosis. Se localiza en áreas mucosas, sin vello, en el terc io inferior de la vulva. VIN tipo diferenciado (no relac ionado con HPV): es mucho menos

El liquen escleroso es la dermopatía más frecuente del área vulvoperineal. La

frecuente y aparece habitua lmente en m ujeres mayores. Las lesiones

etiología es desconocida. Se produce un adelgazamiento de la epidermis,

suelen ser ún icas, de color blanco o rojizo y están situadas en áreas

con pérdida de anejos y engrosamiento de la dermis con infiltración linfoci-

con ve llo. Se asocia habitualmente con la hiperplasia escamosa y

taria. Aparecen pápulas blancas que conAuyen, muy pruriginosas. La piel está

con el liquen escleroso.

tensa, frágil y se descama con faci lidad. Puede mostrar una retracción importante de los tejidos. Es típico de mujeres en la posmenopausia por el déficit

Diagnóstico

estrogénico (aunque también aparece a otras edades). No es premaligno. La VIN no tiene una imagen patognomónica, por lo que el diagnóstico de

Tratamiento

elección es la biopsia .

No existe ningún tratamiento absolutamente eficaz. El t ratamiento mé-

Tratamiento

dico permite calmar los síntomas y evitar la evolución atrófica. Entre ellos se disponen los siguientes: Corticoides locales (de primera elección). Testosterona en pomada al 2%.

El objetivo del tratamiento es ali viar la sintomatología y prevenir la progresión a una lesión invasora. La elección del mismo debe individualizarse en función del tipo de lesión, de la localización de esta y de la edad de la paciente. Se puede optar por:

1 0.3. Hiperplasia de células escamosas

Conducta expectante: está descrita la regresión espontánea, pero es imprescindible asegurar un correcto seguimiento. Trata mientos escisionales: se ha considerado el tratamiento de elección, ya que presenta la ventaja de obtener material para estudio

También llamada liquen simple crónico, consiste en zonas de piel engrosadas, blancas, que se acompañan de hiperqueratosis e infiltrado inAama-

de seguridad de S mm en lesiones un ifoca les o mult ifocales aisladas. En las lesiones múltiples y extensas, estará indicada la vulvectomía

torio crónico. Tiene un bajo potencial premaligno.

simple.

histológico. Se puede rea lizar exéresis loca l amplia con un margen

Manual CTO de Medicina y Cirugía, 2.a edición

Tratamientos destructivos (vaporización con láser): consiguen excelentes curaciones con mínima cicatrización y mejores resultados cosméticos, pero con el inconveniente de carecer de muestra para estudio histológico. Se pueden utilizar en combinación con los tratamientos escisionales. Tratamientos médicos: se ha empleado 5-fluoracilo, interferón o cidofovir con el fin de evitar el tratamiento quirúrgico pero con resu ltados muy pobres.

1 0.5. Enfermedad de Paget de la vulva La enferm edad de Paget de la vulva es muy poco frecuente. Aparece sobre los 60-70 años. Se trata de manchas o placas múltiples, eritematosas

Ideas clave 2S "

"

El liquen escleroso es la patología más frecuente a nivel de la vulva. Se trata de un adelgazamiento de la epidermis con engrosamiento de la dermis por un infiltrado linfocitario. Afecta sobre todo a mujeres mayores por el déficit estrogénico. No se trata de una lesión premaligna. Su tratamiento consiste en la administración de pomada de testosterona, progesterona o corticoides.

o blanquecinas, bien delimitadas, con bordes irregu lares y aspecto eccematoso, que asientan preferentemente en ambos labios mayores, periné y región perianal. La ca racterística hi stológ ica más importante es la prese ncia de célu las de Paget (células de citoplasma amplio y basófilo PAS positivo con núcleo redondeado y nucléolo prominente). Se puede asociar con adenocarcinomas de l aparato genital o extrage nital, por lo que se recomienda estud io exhaustivo para descartar dicha asociación . El tratamiento de elecc ión de la enfermedad de Paget es la escisión quirúrg ica co n márgenes amplios latera les y profundos. Recurre con frecuencia (15%).

"

La neop lasia vu lvar intraepitelial es una lesión pre invasiva que presenta atipias. Se clasifica en tres grados, en función del estrato más o menos profundo en el que se encuentren las atipias celulares. Se diagnostica mediante biopsia y su tratamiento consiste en escisión local amplia con márgenes libres.

"

La enfermedad de Paget de la vulva es otra lesión premaligna que se caracteriza por la presencia de placas eritematosas-blanquecinas en labios mayores. Su diag nóstico es también med iante biopsia y el tratamiento es la escisión loca l amplia con márgenes li bres.

El liquen simple crónico (hiperplasia de células escamosas) se caracteriza por zonas de hiperqueratosis con infiltrado inflamatorio. Se trata con corticoides tópicos y no tiene potencial maligno.

Case Study , A 59-year-old female patient consulted for a fluctuating sore and itchy zone in her vulva, present for several months. Physical examination revealed a poorly demarcated erythemato-squamous patch on the right labia major, with a diameter of 1-2 cm. The lesion was preliminarily diagnosed as genital seborrheic dermatitis and probatorily treated with a crea m containing lithium succinate and zinc sulfate. As the lesion did not improve, and areas of superficial maceration and erosion developed a biopsy was consi-

10 · Enfermedades de la vulva

dered. Skin biopsy revealed the presence of large cells with abundant pale cytoplasm and pleomorphic nuclei in all layers of the epidermis, spreading in the epithelium of the follicular structures, without invasion of the dermis. The most likely diagnosis is: 1) 2) 3) 4)

Vulvar lichen sclerosus. Vulvar mela noma. Vulvar Paget disease. Vulvar squamous cell carcinoma

Correct answer: 3

_Gine_coJngia_y_ ob stet ri eia



Cáncer de vulva

ORIENTACIÓN

ENARM

Es un tema poco importante. Sobre todo, es necesario recordar las Ideas Clave

11.3. Clínica El síntoma principal es el prurito vu lvar (>80%) de larga evolución; por el lo, ante todo prurito vulvar en ancianas, se debe descartar el cáncer de vulva.

El cáncer de vulva es una neop lasia re lativa m ente rara, la cua rta en

Con menor frecuencia debutan como masa, hichazón, ulceración o sangrado. La localización más frecuente es en los labios mayores (Figura 14)

frecuencia de l tracto gen ital femenino. La variedad más frecuente de

(> 50%). Suelen presentarse como lesiones unifocales de tamaño variable.

cánce r de vulva es el epidermoide (tambié n llamado carc inoma escamoso).

11.1. Epidemiología La edad de aparición más frecuente es de los 65 a los 75 años. Se ha planteado que existen dos t ipos epidem iológicos de cá ncer de vulva: el tipo 1 aparece en mujeres jóvenes, es menos habit ual la forma invasora, suele estar menos ava nzado y es de mejor pronóstico; el tipo 11, el más frecuente, afecta a m ujeres mayores, son usua les las formas avanzadas y tiene peor pronóstico.

Tipol

Tipoll

Edad

Jóvenes(< 55 años)

Mayores (7• década)

Asoc iación VIN/HPV

Frecuente

Infrecuente

Subtipos histológicos más frecuentes

Condilomatoso y basaloide

Escamoso bien diferenciado

Frecuencia de afectación vaginal y cervical

Mayor

Menor

Asoc iación a liquen escleroso e hiperplasia de células escamosas

Infrecuente

Frecuente

Figura 14. Cáncer de vulva

Tabla 5. Grupos de incidencia de cáncer de vulva

11 .2. Factores de riesgo

11.4. Diagnóstico

Los factores de riesgo son los siguientes: tabaco, inmunosupresores, cán-

El diagnóstico de elección es la biopsia. La visua lización tras la aplicación de ácido acético o tras la tinc ión con azul de tolu idina (test de Collins)

cer de cérvix, VI N, virus del papiloma humano y distrofia vulvar con atipias.

puede llegar a seleccionar la zona a biopsiar.

Manual CTO de Medicina y Cirugía, 2.a edición

11.5. Extensión

11.8. Tratamiento

El cá ncer de vulva se extiend e por continuidad y por vía linfát ica . La extensión por vía linfática es la forma má s importante de propaga ció n

La ciru g ía debe ser el tratam iento de primera elección, siempre que sea fa ct ible, ga ranti zando unos m árg enes d e segu ridad suficientes y q ue no req uiera una amplia mutilación. En la actual idad, la base del trata-

de la enfermedad y constituye la causa más frecuente de fracaso terapéutico. Las metástasis en ó rg anos distantes son muy raras y tardías.

11.6. Estadificación

m iento qu irúrgico es la esc isión de la les ió n con criterio de rad ica lidad, es decir, con un margen macroscópico entre 15 y 20 mm y escisión en profundidad hasta la fasc ia del diafragma urogenital, por lo que se evitan así cirugías más ag resivas, no exentas de una elevada tasa de comp licaciones.

En la Tabla 6, aparece la estadificación del cáncer de vulva .

Las áreas ganglionares inguinales constituyen el primer esca lón de drenaje linfático de la vulva y, por t anto, son de estud io obligado dent ro del

••• Estadio 1

Valores lA lB

Lesiones ,.;; 2 cm, confinadas a la vulva o periné, con invasión estroma < 1 mm, sin metástasis ganglionares Lesiones> 2 cm o con invasión estroma > 1 mm, confinadas a la vulva o periné, sin metástasis ganglionares

Estadio 11

Tumor de cualquier tamaño con afectación de 1/3 inferior uretra, 1/3 inferior vagina o ano, si n metástasis ganglionares

Estadio 111

Tumor de cualquier tamaño con afectación de 1/3 inferior uretra, 1/3 inferior vagina o ano, con metástasis gang lionares lilA

(1) 1 metástasis gang li onar(~ 5 mm) (2) 1-2 metástasis ganglionares (< 5 mm)

IIIB

(1) 2 o más metástasis ga n g l ionares(~ 5 mm) (2) 3 o más metástasis ga nglionares(< 5 mm)

II IC Estadio IV

nectomía inguinal (exéresis de todo el paquete gangl ionar ing ui nofemoral superficial y profundo). El tratam iento radioterápico del cáncer de vu lva se realiza, básicamente, med iante irradiación externa del área donde asentaba el tumor y/o las cadenas ganglionares. Sus indicaciones serían: tumo res superiores a 4 cm, márgenes quirúrgicos afectos o próximos(< 8 mm), invasión linfovascular y/o adenopatías hi stológ ica mente positivas. La qu imi oterap ia en el tratami ento del cánce r de vu lva ti ene un pape l muy limitado. Actua lmente se utiliza en los estadios avanzados (1 11-IV) .

Con ganglios positivos con extensión extracapsular

Tumor que invade 2/3 superiores vag ina o 2/3 superiores uretra o estructuras distantes IVA

trata mi ento qui rúrg ico. Para ello se podrá rea lizar biopsia selectiva de ga ng lio centinela en casos seleccionados (tumores 20 mm y afectación del tercio inferior del útero.

Tto: histerectom ía total + dobl e anexectomía + linfadenectomía pélvica y paraaó rtica Bajo ri esgo: no adyuva nci a Riesgo intermed io: radioterapia Alto ri esgo: radiotera pia y quimi oterapia

1 ~--------------------~ 1 Tto: cirugía estándar si es posible

Radi oterapia y quimioterapi a adyuva nte

M#ffl!MPM

La hormonoterapia en el cáncer de endometrio se emplea en la enfermedad avanzada y metastásica y en el tratamiento de las recidivas no rescata bies con otros tratamientos. Los fármacos más frecuentemente

M§ffl!fflj!M

utilizados son los gestágenos a altas dosis, como el acetato de medroxiprogesterona o el acetato de megestrol. La quimioterapia en el cáncer de endometrio se utiliza en estadios de alto riesgo de rec idiva (IBG3, IIG3, lilA y IIIB) y en enfermedad metastásica. Las pacientes con cáncer d e endometrio se dividen en tres grupos según el riesgo de recurrencia de acuerdo a la estadificación y los factores pronósticos con fines de tratamiento y seguim iento en tres g rupos, según se indica en la Tabla 12.

Tto: hi sterectomía radica l +doble anexecto mía + linfa denecto mía pé lvica y paraaó rtica . Radioterap ia adyuva nte

Tto: radi ot erapi a, qui mioterapia y/o hormonoterapia

Figura 24. Estad ios del adenocarcinoma endometri al Bajo riesgo Tipo 1, Estadio lA Gl-2

Alto riesgo

Riesgo intermedio Tipo 1, Estadio lA G3

Tipo 1, Estad io lB G3

Tipo 1, Estadio lB G1-G2

Tipo 1, Estadio 11 G3

Tipo 1, Estadio 11 Gl -G2

CualquierTipo 11

Tabla 12. Grupos y su riesgo en el cáncer de endometrio El tratam iento det allado por estad ios es el sigu iente (Figura 24): Est adio 1 con bajo riesgo: tratamiento quirúrg ico estánda r. No re qu ieren tratamiento adyuvante. En este grupo, en ausencia de invasión linfovascular se puede omitir la rea lización de linfadenectom ía dada la infrecuente afectación gangl iona r. Est adio 1con ri esgo interm ed io: se recom ienda radioterapia pélvica adyuvante tras el tratamiento qu irúrg ico. Estad io 1co n alt o ri esgo: se recomienda radioterap ia y quimioterapia adyuvante tras el tratamiento quirúrg ico. Est adio 11: histerectomía radica l, salpingo-ooforectomía bilatera l, linfadenectomía pélvica y para-aórtica seguida de radioterapia adyuvante (Figura 25). Estadio 111: si es posible, tratamiento quirúrg ico y como alternativa radioterapia externa y braquiterapia asociada a quimioterapia adyuvante. Est adio IV: rad ioterapia, quimioterapia y/o hormonoterapia.

16 · Cáncer de endometrio

Figura 25. Histerectomía rad ica l de Wert heim-Meigspor un adenocarcinoma de endometrio de estadio 11. En la pieza, se observa el cérvix con el ma ng uito vag inal en la parte más próxima al observador

Ginecología y obstetricia

Ideas clave

z

"

El cáncer de endometrio afecta sobre todo a mujeres mayores, por encima de los 50 años.

"

Los factores de riesgo implicados en su etiopatogenia son: menarqu ia precoz, menopausia tardía y ciclos anovulatorios; obesidad; diabetes; administración de estrógenos sin gestágenos y tamoxifeno. Se pueden considerar factores protectores la multiparidad, la toma de anticonceptivos hormonales y el tabaco.

"

El diagnóstico del carcinoma de endometrio se basa en la biopsia endometrial. El método ideal para obtenerla es la histeroscopia, ya que permite visualizar la cavidad uterina y dirigir la

Casos clínicos · El estudio histopatológico de la pieza quirúrgica de histerectomía muestra, tras la estadificación completa y como única focalidad neoplásica, un adenocarcinoma endometrioide, limitado al endometrio, con aproximadamente un 30% de patrón de crecimiento sólido. Indíquese el estadio y grado:

1

16

biopsia. Si no se dispusiera de la histeroscopia, se realizará una biopsia endometrial ambulatoria o legrado fraccionado. "

El estadio 1 se trata de tumores limitados al útero. El tratamiento consiste en cirugía (histerectomía total+doble anexectomía+linfadenectomía pélvica-paraaórtica) y radioterapia en el grupo de riesgo intermedio y radioterapia y quimioterapia en el grupo de alto riesgo de recidiva.

"

El estadio 11 agrupa a tumores que afectan al cérvi x uterino en los que se realizará histerectomía radical, doble anexectomía y linfadenectomía pélvica y paraaórtica seguida de radioterapia y quimioterapia.

"

En los estad ios 111 y IV, si es posible, se llevará a cabo tratamiento quirúrgico, seguido de radioterapia y quimioterapia.

estadio 1con grado de diferenciación histológico 1. ¿Cuál sería la actitud correcta? 1) Administrar radioteraparia pélvica (4.500-5.000 cGy dosis total). 2) Administrar quimioterapia con progestágenos. 3) Realizar controles periódicos sin tratamientoad icional. 4) Completar cirugía con linfadenectomía pélvica.

RC: 3 1) la G1. 2) la G2 . 3) lb G1 . 4) le G2.

Paciente de 58 años que ha sido diagnosticada de adenocarcinoma de endometrio. En el informe anatomopatológico, se refiere una invasión mayor del 50% del miometrio y afectación del estroma endocervical por el tumor. El grado de diferenciación es moderado. ¿Cuál es el estadio del tumor?

RC: 2 Paciente de 74 años que tras hemorragia uterina posmenopáusica es diagnosticada de adenocarcinoma de endometrio. Se realizó tratamienteo quirúrgico consistente en histerectomía abdominal total con salpingo-ooforectomía bilateral más toma para citología peritoneal. Tras estudio anatomopatológico se determina que la paciente se encuentra en un

Case Study



A 71-year-old female, with BMI 44, presented with her first episode of postmenopausal vaginal bleeding. She underwent an endometrial biopsy which revealed grade 3 endometrioid endometrial carcinoma. MRI revealed a uterus measuring 19 x 13 x 12 cm with complete loss of usual myometrial architecture with increased T2 signa! at the infiltrative lesion. The mass appeared to involve the entire myometrium and appeared to extend along the right fallopian tu be. No definite extension to parametria was

1) lbG2. 2) IBG3. 3) !la G2. 4) llbG2.

RC: 4

observed on MRI. A 3 x 2.5 cm left externa! iliac lymph node was enlarged a long with a 1.9 x 2.4 cm para-aortic node, which both appeared suggesting metastasis. lndicate the correct classification of this tumor: 1) lB.

2) 11. 3) lilA. 4) !liB. Correct answer: 3



Ginecología y obstetricia

Cáncer de ovario

Multiparidad

Es un tema importante. Un aspecto muy significativo son los diferentes tipos histológicos de cáncer de ovario, así que no hay que dejar de estudiarlo.

Contraceptivos orales Histerectomía Esterilización Ooforectomía Edad Factores de riesgo

17 .1. Epidemiología

Historia famil iar Mutaciones del BRCA 1 y BRCA2

Tabla 13. Factores de riesgo en el cáncer de ovario A pesar de ocupar el tercer o cua rto luga r en frecuencia, el cáncer de ovario es la primera causa de muerte por cáncer ginecológico (excluyendo el

17 .2. Tumores epiteliales

cáncer de mama). Lo padecen una de cada 70 mujeres. El pronóstico de este cáncer depende del estadio; así, la supervivencia a los cinco años de aque llas pacientes en estadios precoces es del 90%, mientras que, en estadios avanzados, desciende hasta un 18%. Por

Los tumores epiteliales suponen el 75% de todos los casos de tumores de ovario (incluyendo benignos y malignos). Engloban a un conjunto de tumores derivados del epitelio de revestimiento del ovario, y es posible

desgracia, al ser en general poco sintomático, el diagnóstico precoz es casi imposib le, por lo que en el momento del m ismo suelen presentar

encontrar las siguientes variantes histológ icas: Serosos (60-80%) (cistoadenoma [Figura 26], cistoadenocarcinoma):

estadios avanzados.

son los más frecuentes. Contienen quistes llenos de líquido seroso. Con

Casi el90% de los cánceres de ovario son de origen epitelial y su incidencia aumenta con la edad, teniendo su máxima frecuenc ia entre los 65 y

frecuencia son bi laterales. Los cuerpos de psamoma son pequeñas calcificaciones que aparecen en la mayoría de los tumores bien diferenciados, por lo que supone un signo de buen pronóstico. Incluso existe

los 80 años. Los factores de riesgo en el cáncer de ovario están poco claros.

una variante llamada psamocarcinoma, compuesta por estos cuerpos de psamoma que, a pesar de ser invasor, se comporta como borderline.

Para el cáncer epitelial de ovario, se proponen los sigu ientes (Ta-

Los serosos malignizan tres veces más que los mucinosos.

bla 13): Teoría de la ovulación incesante: pa rece que el m icrotrauma producido du rante la rotura de la cápsu la ovárica que ocurre en cada ovu lación sería un estímulo para la aparición del cáncer de ovario. Nuligestas: tienen mayor incidencia de cáncer de ovario, ya que la gestación es un periodo de reposo ovulatorio. Están protegidas las mujeres con SOP y las que toman anticonceptivos orales (ninguna de ellas ovula, por lo que se evita el estímulo ovárico). Parece que la ligadura tubárica y la histerectomía tamb ién serían factores p rotectores debido a una disminución de la ovulación por defecto en la irrigación vascula r. Genético: es más frecuente en pac ientes con historia fami liar de cáncer de ovario. Parece que el30o/o de los cánceres de ovario expresan niveles elevados de determ inadas mutac iones genéticas.

Figura 26. Cistoadenoma seroso ovárico

Ginecología y obstetricia Los tumores que presentan cuerpos de psamoma son: cistoadenoma seroso pap il ar de ovario, carcinoma papilar de tiro ides y meningioma.

1

11

cuencia es el derivado del neuroectodermo (lo más habitual neural, cartílago, hueso) (Figura 28).

Mucinosos (25%) (cist oadenoma, cistoadenocarcinoma): la mayoría son benignos (cistoadenomas). Presentan qu istes multilocu lados, que pueden llegar a ser muy grandes, llenos de mucina. A veces se asocian a un pseudomixoma peritoneal. Se trata de una afección asociada con neoplasia mucinosa ová rica, formado por un tumor ovárico con gran ascitis mucinosa, implantes peritoneales quísticos y adherencias. Puede deberse también a un tumor mucinoso primario extraovárico (apendicula r) con diseminación secundaria a ovario y peritoneo (Figura 27).

Figura 28. Quiste dermoide Disgerminoma: deriva directamente de la célula germinal. Es el tumor maligno más frecuente de este grupo y el maligno más habitual en pacientes menores de 30 años. Es radiosensible, si bien el pilar fundamenta l del tratamiento es la cirugía, aunque podemos utilizar la radioterapia en casos de recid iva tumoral. También es posible sustituir la radioterapia por qu imioterapia (BEP: bleomicina, etopósido y cisplatino). Tumor del seno endodérmico: es altamente maligno. Produce u-fetoproteína. Coriocarcinoma: es infrecuente. Genera HCG. Carcinoma embrionario: también es productor de u -fetoproteína. Figura 27. Tumor ovárico mucinoso

Gonadoblastoma: el 90o/o de los casos derivan de gónadas disgenéticas, comprobándose cromosoma Y, como el síndrome de Swyer.

Endometrioides (20%) (endometrioma, carcinoma): la mayoría son ma lignos. En el 1Oo/o de los casos se asocian a endometriosis ovárica: son los quistes de chocolate, con comportamiento benigno. En el 30o/o, se vincu lan a adenoca rcinoma primario de endometrio. De células claras (S%): son los tumores ma lignos más frecuentes en

17 .4. Tumores de los cordones sexuales-estroma

casos de endometriosis. Se consideran una va riante del carci noma endometrioide, y en ambos se contra indica la terapia hormona l sus-

Suponen el So/o de todos los t umores del ovario. Se trata de tumores

titutiva, ya que dicho t ratamiento puede fa vorecer su proliferación

que reprod ucen estructuras propias del folícu lo ovárico o del testículo y,

por responder al estímu lo estrogénico.

como ta les, son capaces de producir sus propias hormonas:

Tumor de Brenner (< 1 %): la mayor parte son benignos. Se carac-

De la granulosa: la mayoría son uni laterales y malignos aunque de

terizan porque el componente epitelia l consiste en nidos de células

bajo grado. Microscópicamente está formado por células que se-

transicionales similares a las que revisten la vejiga (urotel io).

mejan a la granulosa de los folículos maduros, se disponen en forma de roseta con cavidad central rellena de materia l PAS positivo constituyendo los llamados cuerpos de Caii-Exner. Producen estró-

17 .3. Tumores germinales

genos, por lo que pueden dar clínica de pseudopubertad precoz, amenorrea-metrorragia, hiperplasia endometrial. Esto fac ilita que se detecten pronto: el 90o/o en estadio l. Su máxima incidencia ocurre

Es el segundo grupo en frecuencia (15-25%) de los tumores ováricos. Suelen aparecer en mujeres jóvenes, siendo los más frecuentes en este grupo de edad. Teratoma: se trata de la tumoración más frecuente de este grupo. La gran mayoría son benignos. Deriva de célu las germinales. Está

en la postmenopausia . De la teca-fibroma: generalmente son benignos. El SOo/o de los tecomas producen estrógenos-a ndrógenos. El 40o/o de los fibromas ocasionan ascitis, y en el 1o/o se encuentran un síndrome de Meigs: ascitis, hidrotórax y tumor de ovario.

constituido exclus ivamente por tejidos bien diferenciados y es maligno sólo excepciona lmente. Con frecuencia presenta tejido de las

Androblastoma: reproduce elementos testiculares. Pueden ser de-

tres hojas embrionarias, si bien predom ina el ectodérmico: glándulas

andrógenos y, por ello: acné, hipertrofia de clítoris, atrofia sexual se -

sebáceas, sudorípa ras, pelo. Puede producir u-fetoproteína El teratoma maligno es infrecuente. El tejido que reproduce con más fre-

cu ndaria, oligomenorrea, etcétera. Suelen ser benignos. Constituyen la primera causa de virilización de origen ovárico (Tabla 14).

rivados de células de Sertoli, de Leydig o mixtos. El SOo/o produce

Manual CTO de Medicina y Cirugía, 2. 8 edición

..• Epiteliales (derivan del epitelio celómico)

•• Endometrioide

·-.

.

. : . . .

Germinales (derivan del ovocito)

. .. .. ..

El más frecuente de todos. Bilateral. Unilocu lado

El síntoma inicial más frecuente es el aumento del perímetro abdomi-

Un ilatera l. Multiloculado. Con moco. Pseudomixoma peritoneal

de asc it is, seg uido de l dolor abdom ina l variab le, y, en tercer lugar, la

Relacionado con endometriosis

encuentra en situación avanzada.

Relacionado con exposición dietilestilbestrol y endometriosis

En tumores con gran crec im iento y/o en estad ios avanzados, aparece un síndrome constitucional (astenia, caquexia, etcétera). Puede mos-

El más frecuente de este grupo. Tejido derivado del ectodermo

trar clín ica p recoz en los casos en los que se p roduzcan complicacio-

Tejido neural



El mal igno más frecuente del grupo. Radiosens ible (pero el t ratamiento inicialmente es quirúrgico)

·-

Produce alta- a-fetoproteína

Ca. embrionario

Produce alta- a-fetoproteína

Coriocarcinoma

Produce P-HCG. Excepcional que sea primario del ovario

.

De la granulosa

.

...

Estroma les

nes tales como tors ión, rotu ra, infección, etcétera. Los sigu ientes signos hacen sospechar ma lignidad: Ascit is. Palpación de tumoración pélvica. Poca movilidad por adherencias. fndice de crecimiento rápido. Edad no reproductiva .

endodérmico

Gonadoblastoma

metrorragia. Cua ndo se establece el diagnóstico, el 75% de los casos se

Mejor pronóstico (es benigno)

. ,.,.,,Tnm" inmaduro

. •

na l, prod ucido por el p rop io crecim iento del tumo r o por la presencia

Nódu los en fondo de saco de Douglas. Bilateralidad (la m ayoría de los benig nos son unilatera les frente a los ma lignos, q ue suelen ser b ilatera les).

Asociado al síndrome de Swyer Cuerpos de Caii-Exner. Produce estrógenos Benignos. Produce andrógenos y estrógenos. A veces síndrome de Meigs

Androblastoma

Reproduce elementos testicula res. Produce andrógenos

Ginandroblastoma

Mixto entre los anteriores

17.8. Estadificación La estad ificac ión es siempre posquirúrg ica (Tabla 15).

Cápsula intacta, hay tumor en superficie externa Cápsula intacta, no hay tumor en superficie externa

Tabla 14. Tumores de ovario

Rotura de cápsula intraoperatoria Rotura de cápsula antes de operación

17.5. Tumores metastásicos

Células mal ignas en ascitis o lavado peritoneal Extensión a trompas uterinas

Suponen el So/o de los tumores ová ricos. Los más comunes son los de origen

Extensión a otros tejidos pélvicos

mül leriano: útero, trompa, ovario contralateral, peritoneo pélvico. Los prima-

Extensión tubárica o pélvica con cél ulas malignas en ascitis o lavado peritoneal

rios extramül lerianos más frecuentes son la mama y el tracto gastrointestinal, como colon, estómago, vía biliar y páncreas. Un ejemplo clásico de neoplasia

Sólo metástasis a linfonodos retroperitoneales

gastrointestinal metastásica en los ovarios es el tumor de Krukemberg, caracterizado por metástasis bilatera les compuestas por células en anillo de sello

Metástasis lO mm

productoras de mucina, con mayor frecuencia de origen gástrico.

Metástasis macroscópicas peritonea les < 2 cm

17.6. Tumores del mesénquima sexualmente indiferenciado

Metástasis macroscópicas peritonea les > 2 cm

IV Metástasis extraabdominales

Entre ellos encontramos los siguientes: sa rcomas, leiomiomas, heman-

Derrame pleural con citología positiva para células maligna Metástasis a órganos extraabdominales (hígado, linfonodos, bazo)

giomas, lipomas.

Tabla 15. Clasificación FIGO

17.7. Clínica

17.9. Diagnóstico

La característica clín ica fundamental de las tumorac iones de ovario es

El diagnóstico defin itivo del cáncer de ovario es histológico, aunque

la ausencia de sintomatología y su crecimiento lento, por lo que su

es posible apoyarse en la historia clín ica, en las técnicas de imagen y

diagnóstico suele ocurrir en etapas avanzadas o como hallazgo casual

en los ma rcadores séricos para tener orientación acerca del origen de

en el curso de una exploración por otro m otivo.

la masa.

17 · Cáncer de ovario

Ginecología y obstetricia Técnicas de imagen

1

11

El tumor carcinoide cursa con aumento del ácido 5-hidroxiindolacético en orina.

Ecografía-Doppler: actua lmente, es la técnica de imagen más

efectiva en el diagnóst ico del cá ncer de ova rio, con alta sensibilidad (cercana al 100%) pero moderada especificidad (83%). Los ha llazgos ecográficos que harán sospechar malignidad son los siguientes: Tamaño tumoral superior a 5-1 Ocm.

Los marcadores tumora les NUNCA son diagnósticos, pueden ayudar en la sospecha y se utili zan, sobre todo, en el seguimi ento tras el tratamiento.

Contenido heterogéneo refringente. Presencia de tabiques o septos gru esos (mayores de 3 mm), papilas o pa rtes sól idas en su interior. Presencia de líquido en cantidad moderada en Douglasoascitis Mu ltilocuraridad.

17.1 O. Vías de diseminación

Bajo índ ice de resistenc ia Doppler (< 0,4).

TC: la TC ha sido hasta ahora la técnica de elección en el estud io de

La vía de d isem inación m ás frecuente d el cáncer de ovario es la imp lantación directa por siembra peritoneal de célu las tumorales sobre peritoneo o epiplón, por lo que es la extensión peritoneal difusa la

extensión del cáncer de ovario. Perm ite una correcta identificación y caracterización de las tumoraciones ová ri cas, de su tamaño y del

forma más habitual de p resentación del cá ncer de ovario. La disem inación por vía linfá t ica es frecuente con afectación de los ganglios

estado de órganos próximos que pudieran esta r afectados, como la presencia de implantes peritoneales y/o intest inales. También po-

pa ra aó rti cos.

sibil ita el estudio de la posible afectación de los ganglios linfáti cos

La metástasis por vía hematógena es poco frecuente y excepcional que no vaya preced ida de diseminación peritoneal o linfática . De manera

Alto índ ice de pulsatilidad Doppler (> 1).

retroperitoneales.

RM: pa rece superar a la TC en la caracterización de las lesiones, en la evaluación de la exten sión local del tumor y en la definición de implantes tumorales en la superficie hepática o diafragmática.

anecdótica, el cáncer de ovario puede debutar con una masa umbilical (nódu lo de la herm ana Ma ría José).

Marcadores tumorales

17.11. Tratamiento

Los marcadores tumora les, si bien no confirman el diagnóstico de benignidad o de malignidad de la tumoración, son de gran ayuda para la orien-

Cirugía

tación terapéutica, el seguimiento posterior y la respuesta al tratamiento rea lizado. Ca-125: glicoproteína expresada fundamentalmente por los tumores epiteliales de ova rio. Tambi én se encuentra elevada en circunstancias

El tratamiento quirúrgico en el cáncer d e ovario ti ene una doble utilidad: diagnóstica, al permitir estadificar adecuadamente a las pacientes, y terapéutica, extirpando la mayor cantidad de masa tumoral

fisiológicas, como embarazo y menstruación; enfermedades ováricas ben ignas, por ejemplo endometriosis o enfermeda d inflamatoria pél-

posible.

vica; enfermedades extraováricas, como miomas uterinos, insuficien-

En el cáncer de ovario se pueden realizar diferentes tipos de cirug ía: Citorreducción primaria: tiene como objetivos estadificar el tumor

cia hepática o procesos irritativos peritoneales y en procesos mal ignos no ováricos (cáncer de endometrio, de mama y de colon). CEA (a ntígeno carcinoembriona ri o): es una proteína presente en tejidos fetales que desaparece paulatinamente tras el nacimiento.

y resecar la mayor masa tumoral posible antes del in icio de cua lquier otro tratamiento. Consistirá en:

Se asocia con cánceres de: colon, mama, páncreas, vejiga, ovario y cuello de útero. Dentro del cáncer de ova rio, sobre todo, aparece

Lavado y aspiración de líqu ido peritoneal. Histerectomía total con anexectomía bi lateral. Linfadenectomía pélvica y paraaórtica .

ligado a la estirpe mucinosa; sin embargo no es específico porque puede eleva rse también procesos como la enfermedad inflamatoria

Omentectom ía (extirpación del epiplón). Exploración de la superficie peritoneal y biopsia de lesiones sos-

intestinal, cirrosis y en grandes fumadoras.

pechosas, además de biopsiar el peritoneo vesical, el fondo de

CA 19.9: suele estar elevado en tumores de t ipo mucinoso si bien

saco de Doug las, los espacios parietocólicos y la cúpu la diafrag-

puede asociarse también al cáncer de colon. u-fetoproteína: se asocia a tumores de l seno endodérmico y en carcinomas embrionarios de ova ri o, si bien podría aparecer en otros

mática. Apend icectomía, especialmente en los tumores mucinosos.

tumores de la estirpe germinal, por ello es útil en el estudio de masas

Se considera rá completa si al finalizar la cirugía no hay evidencia macroscópica de tumor resid ual, óptima si el implante de mayor tamaño al finalizar la cirugía es menor de 1 cm y subóptima si el implante

anexiales en mujeres jóvenes. lnh ibina: secretada por las célu las de la granulosa, aumenta en tumores de las granulosas y en los mucinosos. HCG: está elevada en el 95- 100% de ca rci nomas em brionari os, que además presentan un aumento de u-fet oproteína en el 70% de los casos. El coriocarcinoma tambi én secret a HCG. Hormonas tiroideas: elevadas en el estruma ovárico. SCC (antígeno de crecimiento de células escamosas) en teratomas inmaduros.

de mayor tamaño es mayor de 1 cm. Ci ru gía secundaria: es aquella que se real iza un tiempo después de haber completado el t ratam iento primario (cirugía y/o quimio-

terapia).

"Second-look": en pacientes clínicamente libres de enfermedad tras haber completado su tratamiento primario inicial (ensayos clínicos).

Manual CTO de Medicina y Cirugía, 2. 8 edición

Citorreducción secundaria: rescate quirúrgico de la enfermedad recidivada o persistente. Cirugía de intervalo: pacientes a los que de manera electiva se decide inicialmente tratar con quimioterapia adyuvante y citorreducción poste rior como trata m iento primario. Se optará por esta modalidad de tratamiento en las pacientes co n tumores irresecables (metástasis viscera les extensas en hígado y pulmón, masas tumorales voluminosas en la raíz del mesenterio que afecten a la arteria mesentérica superior, gran afectación serosa intestinal que obl igue a una extensa resecc ión intestinal, conglomerados adenopáticos retroperitoneales voluminosos por encima de los vasos renales, nódulos diafragmáticos volu m inosos, con penetración en tórax o pacientes con mal estado general y/o con patología asociada seve ra). En pacientes jóvenes, con deseos reproductivos y estad io lAG 1, se puede practica r una cirugía más conservadora, realizando anexectomía unilateral y completando la histerectomía y anexectom íacontralatera l al cumplir los deseos genésicos.

ción clín ica, hematológica y bioquímica (para detectar tumores inducidos por la quimioterapia, ta les como leucemias) y determinación de marcadores tumorales (Ca-125). Las pruebas de imagen usadas serán: ecografía, TC o RM, según las necesidades.

17.13. Diagnóstico precoz Cribado A pesar de que el cá ncer de ovario es una enfermedad cuyo pronóstico mejoraría si fuese posible diagnosticarlo en estadios precoces, no se dispone de un método de screening útil para aplicarlo de forma rutinari a en todas las muje res. En m uj eres con uno o más fam ili ares de prim er grado afectas de cánce r de ovario o de cáncer de mama, au nque no hay evidencia científica de su utilidad, se aconsej a exploración, ecog rafía t ransvagina l y medición de Ca-125 sérico anualm ente, para desca rtar la presencia de alguno de los síndromes familiares que inc luyen cáncer de ovario, como el síndrome de Lynch tipo 11.

Quimioterapia La mayoría de las pacientes con cáncer de ova rio requieren quimioterapia, bien adyuva nte o neoadyuva nte. Los fármacos y protocolos varían en función del tipo histológico, del estad io, de la edad, de los tratamientos previos y del estado basal de la paciente. En los estad ios lA y lB, tanto G1 como G2, no se precisa ninguna terapia adyuvante. En los estadios lA y lB con grado G3 y en eiiC se darán tres ciclos de quimioterapia adyuvante. En los estadios 11, 111 y IV se adm inistran seis ciclos de quimioterapia adyuvante. En pacientes con estadio IIIC y citorreducción óptima la quimioterapia intraperitoneal parece tener un papel importante ya que aumenta la supervivencia aunque presenta mayor toxicidad que la qu imioterapia sistémica.

17 .12. Seguimiento

17.14. Patología benigna de ovario La patología benigna de ovario más frecuente es la sigu iente: Quiste folicular o folículo quístico: alguno de los folículos que se atresian en cada ciclo puede crecer y acumular contenido líquido o sem isól ido en su interior. Generalmente son pequeños y asintomáticos y aparecen en el periodo fértil. Suelen resolverse espontáneamente, por lo que la actitud habitual es la observación a las 4-6 semanas, esperando su desaparición. Quistes lúteos: Granulosa luteínicos: derivan de un cuerpo lúteo normal y pueden producir alteraciones menstrua les. Tecaluteínicos: aparecen tras estímulo con clomifeno (hiperestimulación) o HCG (mola). Son múltiples y bilaterales. No precisan tratamiento quirúrgico.

Es muy importante detectar precozmente las recidivas. Para ello, deben efectuarse revis iones periódicas. El seguimiento incluye: va lora-

Ideas clave /?! "

El cáncer de ovario tiene una baja frecuencia pero, sin embargo, es la primera causa de muerte por cáncer ginecológico. Está motivado, sobre todo, por la ausencia de síntomas hasta estadios avanzados y la ausencia de un programa de screening válido para el diagnóstico en etapas precoces.

"

Se consideran factores de riesgo: la edad avanzada, la historia familiar y la presencia de mutaciones de los oncogenes BRCA 1 y BRCA2. Son factores protectores: la multiparidad, la utilización de anticonceptivos hormonales, la histerectomía, la esterilización tubárica y la ooforectomía.

"

Los tumores epiteliales son la estirpe más frecuente de cáncer de ovario y suelen aparecer en mujeres de edad avanzada. De ellos, la variedad más habitual son los serosos, seguidos de los mucinosos.

17 · Cáncer de ovario

"

Los tumores germinales suelen aparecer en mujeres jóvenes. La gran mayoría corresponden al te rato m a quístico maduro o quiste dermoide, que es benigno. Otros tumores de este grupo son el disgerminoma (maligno que deriva de la célula germinal), tumor del seno endodérmico (produce AFP), carcinoma embrionario (produce AFP), coriocarcinoma (produce HCG).

"

Los tumores de los cordones sexuales-estroma son la estirpe menos frecuente y se caracterizan por la síntesis de hormonas esteroideas que permiten que se puedan diagnosticar más precozmente que otros tipos. Los más habituales son los tumores de la granulosa (síntesis estrogénica), seguidos de los tecafibromas (síntesis de andrógenos).

"

La clasificación del cáncer de ovario se hace mediante laparotomía exploradora, por lo que la cirugía se realizará a todas las mujeres. En la mayoría de los casos, encontraremos tumores en estadios avanzados, ya que la vía de disem inación es la implantación directa por siembra peritonea l de células tumorales.

Ginecología y obstetricia "

El tratamiento es fundamentalmente quirúrgico, realizando histerectomía con doble anexectomía, linfadenectomía pélvica y paraaórtica, apend icectomía, omentectomía, biopsias de peritoneo y lavado y aspiración de líquido peritoneal. En función de los hallazgos, se puede clasificar el cáncer de ovario: estadio 1:

Casos clínicos Una mujer de 54 años inicia estudio por molestias abdominales. El escáner descubre una lesión quística de 8 cm en el ovario derecho y dos nódulos abdominales que parecen depender del peritoneo. El Ca-125 es normal. ¿Cuál de estas actitudes le parece más acertada? 1) Laparotomía exploradora, para diagnosticar y tratar un proba-

ble cáncer de ovario. 2) Puede tratarse de un quiste lúteo, convendría realizar un tratamiento de prueba con anovulatorios y reevaluar en tres meses. 3) Se trata de un cáncer diseminado, la paciente debe recibir quim ioterapia sistémica. 4) Vigilancia, repitiendo la exploración con escáner en tres meses. RC: 1 Mujer de 60 años. En la revisión rutinaria se detecta una masa móvil en pelvis. El estudio ecográfico pone de manifiesto, como único hallazgo, una lesión quística de 70 mm con proliferaciones papilares internas y dependiente de ovario derecho. El marcador tumoral Ca-125 es de 70 U/m (normal< 35 U/mi). ¿Cuál es la conducta diagnóstico/terapéutica adecuada? 1) Control evolutivo mediante seriación ecográfica y del nivel del marcador. 2) Completar el estudio med iante marcadores de estirpe germinal (u -fetoproteína/gonadotropina coriónica).

Case Study . A 58-year-old woman, gravida 2, para 2, presented with a twoweek history of lower left abdominal pain and abdominal fullness. She underwent total abdominal hysterectomy because of a uterine myoma ten years ago. Physical examination revealed a palpable mass (25 x 20 cm) in the left lower abdomen. The level of CA 125 was 327.36 U/ml (normal range < 35 U/ml), while carcinoembryonic antigen and CA 19-9 levels were within normal

1

11

limitado al ovario, estadio 11: afectación pélvica, estadio 111: afectación de la superficie peritoneal y estadio IV: metástasis. "

La quimioterapia es la adyuvancia en el cáncer de ovario, y se aplicará en los estadios IA-IB-G3, IC, 11, 111 y IV.

3) Estudio citológico de la lesión mediante punción-aspiración con aguja fina. 4) Tratamiento quirúrgico: anexectomía y estudio intraoperatorio de la lesión. RC: 4 Una mujer de 52 años ha sido diagnosticada de un carcinoma de ovario seroso, que ha resultado inoperable por extensión abdominal masiva. La conducta a seguir más adecuada es: 1) 2) 3) 4)

Cuidados paliativos. Radioterapia abdominal. Quimioterapia. Progestágenos.

RC: 3 Paciente nuligesta de 23 años, diagnosticada de tumoración ovárica ecográficamente sospechosa de malignidad. Se informa de la posibilidad de realizar cirugía conservadora en caso de cumplirse ciertos requisitos indispensables. ¿Cuál de los siguientes supuestos desaconsejaría dicha actitud quirúrgica? 1) 2) 3) 4)

Biopsia de epiplón negativa. Tumor encapsulado. Valores de Ca-125 superiores a 35 U en estudio preoperatorio. Ascitis.

RC: 4

limits. Ultrasonography demonstrated a 28 x 19 x 1O cm multicystic mass with interna! echogenicity and septation. The most likely diagnosis is: 1) 2) 3) 4)

Serous cystoadenocarcinoma . Mucinous cystadenocarcinoma. Dysgerminoma . Ovarían granulosa cells tumor.

Correct answer: 1



Ginecología y obstetricia

Patología benigna de la mama

Tema de importancia intermedia. Sobre todo, no se debe dejar de estudiar el fibroadenoma y la mastopatía fibroquística.

pruebas de imagen para reducir intervenciones quirúrgicas innecesarias ya que la mayor parte de casos de telorrea responden a patologías benignas. Serán de utilidad la mamografía, la ecografía y la citología de la secreción. El tratamiento debe ser específico pa ra la etiología de la secreción.

18.2. Trastornos inflamatorios La patología ben igna mamaria es más frecuente que la ma ligna. Su conocimiento permitirá hacer un correcto d iagnóstico diferencial entre ambas.

18.1. Trastornos funcionales

Los trastornos inflamatorios de la mama son los siguientes: Mastitis: las mastitis puerperales agudas son la forma más habitual de aparición, aunque ha aumentado la frecuencia de mastitis agudas no asociadas con la lactancia. Los gérmenes más frecuentes son: Staphylococcus aureus, Staphy!ococcus epidermidis y estreptococos. Se presenta como dolor en un cuadrante de la mama, acompañado de

La telorrea es la secreción espontánea pero no fis io lóg ica procedente del pezón. Se encuentran diferentes tipos de secreción segú n el aspecto macroscópico de la misma : Secreción grumosa: aparece en la ga lactoforiti s. La secreción es pegajosa y de aspecto hemorrág ico. Puede ocurrir a cua lquier edad pero es más frecuente en mujeres menopáusicas o perimenopáusicas y en presencia de mastopatía fib roquística. Secreción purulenta: puede manifesta rse en los d istintos tipos de mastitis. Secreción acuosa: es poco frecuente y suele sugerir una lesión maligna, por lo que se aconsejan estud ios para descartar este d iag nóst ico. Secreciones serosas, serosanguinolentas y hemorrágicas (telorrag ia): son los tipos más frecuentes. Su causa principa l, sobre todo en aquellas pacientes que son un ilatera les y uniorific iales es el papiloma intraductal, pero también puede verse en carcinomas, enfermedad fibroquística o dilatación avanzada de los conductos. El pico de incidencia es en torno a los 40 años, pero a partir de los 50, la lesión causante más frecuente es el carcinoma y está indicada la biopsia siempre que se palpe algún nódu lo. Cuando no existe lesión palpable, el diagnóstico puede ser más difícil. Secreción lechosa o galactorrea: se caracteriza por una secreción bilateral y pluriorificial. Puede ir asociada a hiperprolactinem ia o no.

linfang itis mamaria y de fiebre elevada. La infección puede progresar y orig ina r un absceso que requiere drenaje quirúrgico. El diagnóstico d iferencial ha de establecerse con el carcinoma inflamatorio, que no suele acompañarse de fiebre; no obstante, en caso de duda, está ind icado rea lizar una biopsia. El tratam iento consiste en medicación sintomática y cloxacilina. En caso de alerg ia se pautará eritromicina. Un tipo de mastit is crón ica es la enfermedad de Mondor, que consiste en una tromboflebitis de las venas subcutáneas de la pared torácica secundaria a un t rauma loca l. Puede persistir varios meses, pero no req uiere tratam iento. Mast odinia: la mastod inia es el principa l síntoma de la mastopatía fibroqu ística (MFQ). Consiste en un dolor mamario cíc lico que aumenta en el periodo premenstrua l y que se alivia tras la menstruación. Debe distinguirse del dolor mamario en general, llamado mastalgia. Siempre que exista dolor, ha de descartarse la existencia de una patología mamaria subyacente.

18.3. Mastopatía fibroquística (M FQ) o displasia mamaria Se trata de una enfermedad benigna y crónica, caracterizada por una al-

En el diagnóstico, además de una adecuada historia clínica y de exploración buscando masas, ulceraciones, alteraciones de la estructura g landu-

teración en la proliferación del estroma y del parénquima mamario, desarro llando tumores o quistes palpables. Es la patología más frecuente en la

lar, erosiones o retracciones del pezón o cambios cutáneos, se ayudará de

mujer premenopáusica y es muy rara después de la menopausia.

Ginecología y obstetricia /

18

Se desconoce la causa; se ha propuesto la influencia de un desequilibrio hormonal, con hiperestrogeni smo, aunque no es consta nte.

Tratamiento

Clínica

a la paciente la benignidad del proceso y tranquilizarla.

No hay una clínica es pecífica de la MFQ. El síntoma más frecuente es el dolor mamario, que suele ser premenstrual y mejora con la regla (mastodinia). El dolor es bilateral. Pueden aparecer otros síntomas como indura-

Como tratamiento médico, se puede asociar progestágenos en la segu nda fase del ciclo para compensar el hiperestronismo; también parecen

ción mamaria o nódulos palpables.

aceite de onagra .

En genera l, la d isplasia mamaria no requiere tratamiento. Se debe explicar

El embarazo y la lactancia producen en la mama mastopática un estado de reposo.

útiles progestágenos en gel para rea lizar tratamiento local y la ingesta de

18.4. Tumores benignos de la mama

Clasificación Para determinar el riesgo de degeneració n a cá ncer de mama, se va lora la existencia de proliferac ió n del componente epitelial, y si hay atipia o no en esa proliferación, distinguiéndose tres tipos de mastopatía fibroquís-

Const ituyen alrededor del 20o/o de los casos de patolog ía mamaria. Los hay de conten ido sólido (fibroadenoma, lesiones papilares, etcétera ) y de contenido líqu ido (quistes, ectasia ducta l, ga lactocele .. ) (véase la Tabla 17 más adelante).

tica (Tabla 16):

Fibroadenoma

No proliferativa (68o/o). Proliferativa sin atipia (26o/o).

El fibroadenoma es el tumor benigno más frecuente de la mama, for-

Proliferativa con atipia (4o/o).

mado por la prolifera ción de elementos epiteliales y mesenquimatosos La mayoría de los cambios que se producen en la displasia no se relac ionan con un aumento en la incidencia de cáncer de mama. El riesgo de cáncer

bien diferenciados, predom inando su carácter fibroso frente al glandular. Es pseudoencapsu lado, bien del imitado del tejido mamario adyacente,

está moderadamente aumentado en los t ipos proliferativos con atipia, que son la hiperplasia ducta l atípica y la hiperplasia lobul illar atípica.

móvil, esféri co u ovoide, aunque puede ser multilobulado. En ocasiones (10-15%), es posible que aparezca en ambas mamas y ser numeroso. Es un tumor estrogenodepe ndiente, por lo que es posible que aumen te de tamaño con la toma de anticonceptivos ora les, en la gestación y en la

Adenos is

lactancia e incluso en la terapia hormona l sustitutiva . Su mayor incidencia

Ectasia ductal

se presenta entre los 15 y los 35 años.

Fibroadenoma Fibrosis Mastitis Hiperplasia sin atipias Quiste Metaplasia apocrina y escamosa Fibroadenoma con patrón morfológico complejo Hiperpla sia moderada o intensa sin atipia Adenosis esclerosan te

Su diagnóstico se basa, fundamenta lmente, en la clínica del nódulo de características benignas, confi rm ándose por las pruebas de imagen. La ecografía es la técnica de elección en estos casos al tratarse de mujeres jóvenes en las que la mamografía tiene un papel limitado, ya que la mama en estas edades es radiológicamente m uy densa, dificultando la va loración de las imágenes. El patrón ecográfico típico es la presencia de un nódulo sólido, regular, de bordes bien del imitados, ovoide, hipoecoico (Figu ra 29) . Puede presenta r ca lcificaciones, genera lment e grandes, gruesas, que au mentan con el tiempo. En la mamografía adopta n la forma típica en "palomitas de ma íz':

Papiloma sol itario (1,3)/múltiple (3,3) Riesgo moderadamente aumentado (4-5 veces) Riesgo con marcado aumento (8-1 O veces)

Hiperplasia ductal atípica Hiperplasia lobulillar atípica Carcinoma ductal in si tu Carcinoma lobulillar in si tu

Tabla 16. Riesgo relativo para carcinoma invasivo de mama

Diagnóstico El diag nóstico se basa en la clín ica, en técnicas de imagen (ecografía y mamografía) y en un estudio microscópico (ci to log ía e histología). Se rea lizará punción-aspiración en las formas de ma stopatía fibroquística de predomin io nodular-quístico, ya q ue ofrece garantía diagnóstica, puesto que, en la mitad de las ocasiones, cura el quiste tras la evacuación del conteni do. Si existe alguna duda sobre la naturaleza benigna del proceso, se rea lizará biopsia.

Figu ra 29. Ecografía de un fibroadeno ma

Manual CTO de Medicina y Cirugía, 2 .a edición

Se recomienda extirpación quirúrgica en los sigu ientes casos: tamaño superior a 2 cm, rápido crecimiento, duda diagnóstica, si la paciente se muestra preocupada (cancerofobia), si produce dolor o en mujeres mayores de 35 años. En el resto de los casos, basta con rea lizar un segu imiento clínico y por imagen anual. El hallazgo de un carcinoma en el interior de un fibroadenoma es extremadament e ra ro (Figura 30).

es tan habitual encontrar telorrea asociada. El tratamiento es la exéresis quirúrgica de una zona amplia alrededor de la lesión. Su importancia radica en que hasta un 40% de los casos se asocian con cáncer de mama.

Quiste mamario Son muy frecu entes. Se suelen presenta r en el contexto de la mastopatía fibroqu ística. La máxima incidencia se da entre los 40-50 años. Habitua lmente son tumorac iones indoloras; en la palpación se objet iva una forma redondeada, lisa, móvil, no adherida, sensib le y, a la presión, se nota tensión e incluso fluctuación . El d iagnóstico de lesión es ecográfico, visual izándose como nódulos anecogénicos de límites muy precisos, morfología regu lar y refuerzo posterior (Figura 31 ). Los ga lactoceles son una variante de éste: se trata de quistes de leche en mamas lactantes que cesan su actividad (Figura 32).

Figura 30. Extirpac ión de un fibroadenoma por crecimiento (también está indicado si hay alteración estética o cancerofobia)

Adenoma Constituye un tumor bien delimitado, compuesto de elementos epiteliales diferenciados con escaso estroma. No es frecuente y afecta sobre todo a mujeres jóvenes. El tratamiento de los adenomas es quirúrgico y consiste en la extirpación completa de la lesión.

Papiloma intraductal Se trata de proliferaciones exofíti cas de células epiteliales de los cond uctos ga lactóforos sobre un eje fibrovas-

Figura 31. Ecografía de quiste mamario

cu lar. Hay que distinguir dos tipos: Papiloma solitario: afecta a los conductos principales, localizándose a 1-2 cm del pezón. Suele aparecer en mujeres premenopáusicas. Cu rsa con telorrea serosangu inolenta uriorificial espontánea, sin tumor pa lpable. Es la ca usa más frecuente de telorrea. El tratamiento es quirúrgico y consiste en la exti rpación de la zona previa loca lización del con-

Quiste mamario

ducto que produce la telorragia. No se asocia con un riesgo aumentado de padecer cáncer de mama, ni suele recidivar. Papilomatosis múltiple: afecta a varios cond uctos y se ori gina en las un idades lobul illares terminales. A menudo se asocia a hiperplasia epitelial atípica . Aparece en mujeres más j óvenes que el papi loma sol itario y no

18 · Patología benigna de la mama

Figura 32. Quiste mamario y cáncer de mama

Cáncer de mama

Ginecología y obstetricia

Ectasia ductal Adenoma (pezón/mama)

Baja frecuencia. Variable

Fibroadenoma

Joven. El más frecuente antes de la menopausia

Tumor duro, móvil, ovoide

Tumor filodes

4•-5• década

Tumor de crecimiento rápido

Ectasia ductal

La mastopatía fibroquística se caracteriza por una alteración en la proliferación del estroma y del parenquima mamario, produciendo quistes y tumores palpables, todos ellos benignos. Es la enfermedad benigna más frecuente de la mama y, sobre todo, se encuentra en mujeres premenopáusicas.

Menopausia

Secreción hemática

"

El dolor es el síntoma más frecuente de la mastopatía fibroquística. Suele aparecer premenstrual y desaparece tras la regla. Además, es posible encontrar otros síntomas como induración mamaria o nódulos palpables.

"

La mastopatía fibroquística no precisa tratamiento, pero es necesario asegurar el diagnóstico con PAAF o biopsia. Para el control de la mastodinia, se puede aplicar progesterona tópica en la segunda fase del ciclo.

"

El fibroadenoma es el tumor benigno más frecuente. Suele aparecer en mujeres entre los 15 y los 35 años. Se caracteriza por aumentar de tamaño durante el embarazo y la toma de anticonceptivos hormonales por ser un tumor estrógenodependiente.

"

El tratamiento del fibroadenoma es mantener una conducta expectante. La cirugía se indicará cuando exista duda diagnóstica, si crece, si es de tamaño grande, si produce síntomas o si la paciente lo desea.

" El riesgo de cáncer de mama se relaciona con la hiperplasia ductal atípica y la hiperplasia lobulillar atípica.

Casos clínicos

Ante una mujer de 20 años, que presenta una tumoración de 2 cm de diámetro en el cuadrante inferoexterno de la mama derecha, indolora, de consistencia firme, superficie lisa, forma ovoidea, móvil y bien delimitada del parénquima vecino, sin antecedentes de derrame por el pezón, sin "piel de naranja" ni retracción del pezón, ¿cuál sería su diagnóstico de presunción? 1) 2) 3) 4)

No doloroso

Tabla 17. Patologías benignas de la mama

" La telorrea o secreción por el pezón es un signo importante. Sobre todo, hay que analizar si es uni o bilateral y el aspecto que presenta. Además, es necesaria la realización de citología de la secreción. La telorrea unilateral puede ser debida a ectasia ductal, carcinoma ductal, papiloma intraductal o quistes solitarios. La secreción bilateral fundamentalmente hará pensar en trastornos funcionales, fisiológicos o farmacológicos.

1

Tumor

Malignidad variable

Ideas clave fo!S

"

Secreción hemática No palpable

Suele aparecer en mujeres perimenopáusicas y menopáusicas, más frecue ntemente en fumadoras. Se caracte riza por la dilatación de los duetos subareola res principales dando lugar a una secreción por el pezón que, si se infecta, puede abscesificarse y fistulizar a piel. El tratamiento es qu irúrg ico.

3.•-4.• década

18

Clínica

Edad Papiloma intraductal

1

Absceso. Carcinoma. Ectasia de los conductos mamarios. Fibroadenoma.

Mujer de 25 años con nódulo mamario palpable de aparición brusca. La ecografía revela un nódulo anecogénico, de límites muy precisos, morfología regular y refuerzo posterior, único, de 3 cm de diámetro. ¿Cuál de los que se relacionan es el diagnóstico más probable? 1) 2) 3) 4)

Quiste. Displasia fi brosa. Fibroadenoma. Cáncer.

RC: 1

RC: 4

Case Study A 33-year-old female with a history of fibrocystic changes of the breast, presented a palpable lump in her right breast. She had no family history of breast cancer. Clinical examination revealed a non-tender, mobile, well circumscribed cystic lesion in the lower inner quadrant of the right breast. Mammography revealed a well circumscribed mass lesion with thickened wall. On sonography, the mass was hypoechoic and sharply defined, reminiscent

of a cystic lesion with thick and irregular wall. lndicate the most correct option for this patient: 1) 2) 3) 4)

Follow-up every year. Fine needle aspiration cytology. Biopsy. Excision.

Correct answer: 2



Ginecología y ob_ste_trjcia__

Cáncer de mama

Es un tema fundamental. Los apartados más importantes son Jos factores de riesgo, Jos factores pronósticos y, sobre todo, el tratamiento.

El resto de factores de riesgo son más discutibles: las d ietas ricas en grasas, el primer embarazo ta rd ío, la no lactancia, la nuliparidad, los antecedentes persona les de cáncer, sobre todo de endometrio, la hormonoterapia sustitutiva o con anticonceptivos hormonales, la exposición a radiaciones, la inmunodepresión, etcétera.

19.3. Diagnóstico

19.1. Epidemiología

El diagnóstico se basa en los siguientes aspectos: El cáncer de mama es el tumor ma ligno más frecuente en la mujer, especialmente en los pa íses desarrollados. Existe un incremento progresivo en su incidencia en los últ imos años, y este aumento es más significativo

Autoexploración: no ha demostrado util idad en la disminución de la morta lidad; por tanto, no es úti l en el d iagnóstico precoz de este tipo de cáncer.

entre las mujeres de menos de 40 años.

Exploración clínica: es muy importante. Hay que pa lpar la mama y las áreas linfáticas. Son signos sospechosos de malignidad: un nó-

En México, el cánce r de mama constituye la neoplasia maligna más co-

du lo duro, fijo

mún y es la causa más frecuente de muerte por cá nce r en la mujer, por lo

pezón, la secreción sang uinolenta y unilatera l y la presencia de adenopatías d uras, fijas y homolatera les.

que supone el 20-25% de todos los casos de cá ncer en la mujer y co ntribuye con un 15-20% de la morta lidad por cánce r.

y de contorno irregula r, la retracción de la piel o del

Mamografía: actua lmente es el método imprescind ible en el d iagnóstico precoz del cáncer de mama. Se consideran signos mamográficos de posible malignidad los siguientes:

19.2. Factores de riesgo

M icrocalcificaciones agrupadas anárquicamente en número igual o superior a cinco, no diseminadas, lineales o ramificadas, de d istribución segmentaría y de tamaño simétrico. Las micro-

Son mú lt iples,

y es muy importante el factor genético en aquellas pacien-

calcificaciones son el sig no sospechoso de mal ignidad que apa-

tes con el antecedente fami liar de carcinoma de mama, sobre todo, en eta-

rece más precozmente en la mamografía.

pas anteri ores a la menopausia. Dos genes, BRCA 1 y BRCA2, parecen ser los

Presencia de un nódulo denso, espicu lado, de bordes impreci-

responsab les de más de la mitad de los cánceres de mama hereditarios.

sos, con retracción de la piel o con edema cutáneo. Este edema es debido al bloqueo de los gang lios linfáticos subdérm icos por las cé lulas ca nce rosas; a veces, este hallazgo es observado más

+++ +++ ++ ++

Pérdida de la arqu itectura y desestructuración del tej ido mamario.

Menarquia precoz

+

Densidades foca les asimétricas.

Nuliparidad

+

Primer embarazo ta rdío Lactancia corta o artificial Adiposidad

+ + +

Estatus social alto Antecedentes personales de otros cánceres

+ +

Antecedentes familiares (BRCA 1, BRCA2) Patología mamaria previa Menopausia tardía Irradiación repetida

Tabla 18. Factores de riesgo de cá ncer de mama

precozmente que la exploración clínica, cuya manifestación es la típica piel de naranja.

En genera l, la indicación de cuándo debe realizarse una mamografía es la siguiente: En mujeres asintomáticas de 40 a 69 años de edad, mamografía cada dos años. En los casos que se sospeche cáncer de mama hereditario (personas portadoras de mutaciones BRCA 1/BRCA2, dos o más fa-

Ginecología y obstetricia miliares de primer grado afectados por cáncer de mama, his-

1

19

Tipos histológicos

toria familiar de cánce r en más de una generación, varón en la fam ilia afectado con cánce r de mama, fami liar con cáncer de

La OMS clasifica el cáncer de mama según su lugar de origen (ductales o

mama y ova ri o, familiar con cáncer de mama bilateral o varios

lobulillares), según su carácter (in si tu o invasivo) y por su patrón estructural:

casos de cáncer de ovario en la familia) la mamografía se reali-

No invasores (Tabla 19):

zará anualmente, empezando entre cinco y diez años antes del

Carcinoma intraductal o carcinoma ductal in situ (CDIS): re-

diagnóstico más precoz de cáncer de mama en la familia, pero

presenta el 80% de los casos de carc inomas in situ. Se trata de una proliferación celula r de aspecto maligno dentro de la unidad

no por debajo de los 25 años de edad.

ducto-lobul illar que no ha atravesado la membrana basal y, por tanto, no invade el estroma circundante. El60% de los carcinomas

La evaluación de las mamog rafías, así como el seguimiento a realizar, se basa en el sistema internacional BIRADS que clasifica los hallazgos

infiltrantes tienen asociado componente in situ. La edad media de

en siete grupos:

aparición es de 49 años. La forma de presentación más habitual

BIRADS O: estudio de imagen incompleto; se necesitan otras

(75 -85%) es asintomática, identificándose en las mamografías,

pruebas de imagen.

aunque también pueden debutar como tumor pa lpable, secre-

BIRADS 1: mamog rafía negativa. La paciente puede continua r

ción y/o enfermedad de Paget. El objetivo del tratam iento quirúr-

el programa de cribado con norma lidad.

g ico es obtener un control local óptimo para prevenir las recidivas

BIRADS 2: mamografía con hallazgos compatib les con les iones

ya que la mitad de las mismas adoptan un patrón infiltrante y la

benignas. La paciente puede seg ui r el programa de cribado con

otra mitad son nuevamente carc inoma in situ. La elección entre

normal idad.

cirugía conservadora, asoc iada o no a rad ioterapia, o mastecto-

BIRADS 3: mamografía co n hal lazgos probablemente ben ignos

mía se real izará en func ión de las características clínicas e histoló-

pero pa ra co nfir marlo es p reciso hacer nu eva mamografía en

g icas y para ello es preciso basarse en el índice pronóstico de Van

seis meses.

Nuys modificado (Tabla 20). No es necesa ri a la linfadenectomía

BIRADS 4 : mamografía con hallazgos probablemente mal ignos

axilar. El empleo de la biopsia selectiva del gangl io centine la tam-

por lo que es necesaria la realización de biopsia diagnóstica.

poco está indicado de manera sistemática y se reserva únicamen-

BIRADS 5: mamografía con hallazgos altamente sospechosos

te para casos seleccionados de tumores de gran tamaño y con

de malignidad y, por el lo, también es precisa la biopsia para con-

alto grado nuclear por ser más probable que contengan focos de

firmar el diagnóstico.

microinvasión o que vayan a ser sometidos a mastectomía.

BIRADS 6: diagnóstico firme de cáncer de mama mediante la

biopsia de tej ido mamario. Lobulillar in situ

Ductal in situ Ecografía: es un método d iagnóstico complementario a la mamo-

grafía, que en ningún momento sustituye a la misma en el cribado

Edad

Menopáusicas

Premenopáusicas

Multicéntrico

No



Bilateral

No



Clínica

Sí / No

No

Mamografía

Alterada

Normal

Factores de riesgo

+

+++

de cáncer de mama, aunque resu lta útil, sobre todo, para diferenciar entre una lesión sólida y una quística. También permite gu iar la punción de lesiones tanto para estud io cito lógico como histológico y realizar el estudio axilar para descartar posibles adenopatías. Resulta imprescindible en el estudio de mamas densas, siendo la prueba fundamenta l para el m ismo en mujeres jóvenes, en las que la mamografía pierde eficacia.

oo oQ

Resonancia: su indicación fundamental es la detección de multifo-

calidad, de recidivas en cicatriz y de carcinoma oculto. Punción aspiración con aguja fina (PAAF): ante toda sospecha de

Tabla 19. Tipos histol ógicos no invasores

patología mamaria, se debe practica r una PAAF con estud io citológ ico del materia l obtenido. Es una prueba económ ica, con pocas compl icaciones y gra n va lo r diagnóstico; ti ene una sensibilidad del 91% y una especificidad del 96%. Biopsia con aguja gruesa: su sensibilidad es del 89% y su especifi-

cidad del 100%, y no precisa patólogos especialmente entrenados, al contrario de co mo ocurre co n la PAAF, perm itiendo d isting uir las formas invasoras de las no invasoras. En tu m ores m uy pequeños, ad-

Tamaño tumoral

,;; 15 mm

16-40 mm

40 m U/m i, y estrad iol < 20 pg/ml.

neficios en la enfermedad coronaria . Algunos de ellos incluso han evidenciado un incremento significativo de infartos en las usuarias de THS comb inada, aunque no se ha incrementado la mortalidad

20.4. Tratamiento

por cardiopatía isquém ica. Tromboembolismo venoso (TVP o TEP): existe evidencia de que el riego está incrementado, sobre todo en el primer año de uso.

Las d iferentes opciones de tratam iento q ue muestran posibilidad de uso

Accidente cerebrovascular: se ha demostrado un incremento sig-

en el cli materio suponen un tema con troverti do y en constante revisión.

nificativo de ACV en usuarias de THS.

20 · Menopausia y climaterio

Ginecología y obstetricia

1

20

La tibolona es un esteroide sintético que tiene propiedades estrogénicas, gestagénicas y androgén icas útil en el tratamiento del síndrome climatérico. Entre sus ventaj as destaca una reducción significativa

efectos son positivos ya que ca usan síntomas vasomotores y aumentan el riesgo de tromboembol ismo venoso.

del riesgo de fracturas ve rtebra les, descenso del riesgo de cáncer de mama y del cáncer de endometrio. Sin em ba rg o, se observó un riesgo aumentado de ictus, sobre todo si se utilizaba en m ujeres mayores de 70 años.

Los fitoestrógenos so n sustancias no hormonales que se une n a receptores de estrógenos. Son una altern ativa para las mujeres que no pueden utilizar terapia hormonal y padecen un sínd ro me climatéri co florido.

Los modu ladores selectivos de los receptores estrogénicos (SERMs) son un grupo de fármacos que se usan pa ra la prevención de la osteoporosis y el tratamiento y prevención del cá ncer de mama. El raloxifeno presenta efectos agonistas estrogénicos en hueso. Sin embargo, no todos sus

Ideas clave

~

" En la menopausia se produce un agotamiento folicular en el ovario. La clínica depende fundamentalmente del descenso de los estrógenos. Hasta que no disminuyen, no aparecen los sofocos, las sudoraciones, las palpitaciones, el insomnio, la lab ilidad emocional, la irritabilidad, la dism inución de la libido . . . En la premenopausia no suelen existir síntomas, aunque sí hay una gran variabilidad de una mujer a otra . El marcador analítico es la FSH, que va a ir aumentando paulatinamente, mientras que el resto de hormonas están dentro de la normalidad. "

Nunca se deben dar estrógenos sin asoc iar progestágenos excepto en histeroctomizadas.

"

El tratamiento hormonal sustitutivo sólo está indicado en las mujeres sintomáticas, en la mínima dosis eficaz y durante el menor tiempo posible, ya que un efecto secundario importante es el aumento de la incidencia de cáncer de mama, al t ratarse de un estímulo estrogénico. Otro riesgo importante al que están expuestas las pacientes usuarias son la cardiopatía isquémica y el tromboembolismo venoso y pulmonar.

"

El raloxifeno es un fármaco modulador selectivo de los receptores estrogénicos utilizado en la prevención y el tratamiento de la osteoporosis por su efecto agonista en el hueso. Otra ventaja es su efecto antiestrogénico a nivel del endometrio, por lo que no induce hiperp lasia de endometrio. Su principal inconveniente son los sofocos y la atrofia urogen ital, que empeoran con su utilización.

Durante la menopausia aparece un aumento de la pérdida de masa ósea, por lo que se incrementa el riesgo de osteoporosis y de fracturas. Otros factores implicados son : raza blanca, ooforectom ía temprana, delgadez, baja ingesta de calcio, vida sedentaria, alcohol, café, esteroides ...

Casos clínicos , ¿Cuál de los siguientes tratamientos es el más adecuado en una mujer de 55 años, menopáusica desde hace uno por habérsele practicado una histerectomía con doble anexectomía, que padece un síndrome climatérico importante?

clorotiazida 25 mg/día y atenololl 00 mg/día. Hace 3 años, sufrió una fractura de Colles derecha. Entre sus hábitos, destaca desde siempre un muy bajo consumo de leche y derivados, un alto consumo de proteínas y una vida sedentaria. A raíz de un cuadro de dolor dorsal agudo, ha sido diagnosticada por su médico de aplastamiento vertebral secundario a osteoporosis. ¿Cuál de los siguientes conjuntos de factores pueden haber favorecido el desarrollo de la osteoporosis?

1) Calcio oral y benzodiacepina s. 2) Estrógenos y progestágenos 19 norderivados.

3) Estrógenos y vera lipride. 4) Estrógenos excl usivamente.

RC: 4 Paciente de 64 años, fumadora de 20 cigarrillos/día y bebedora de 60-90 g de alcohol/semana. Menopausia a los 41 años. Es hipertensa conocida desde hace un año, y se controla con hidro-

ea S e

St udy

~--

A 49-year-old woman has been experiencing intermittent hot flashes and difficulty sleeping for the past year. The severity and frequency of hot flashes have increased recently. She has frequent waking episodes and is having problems concentrating. Menses had occurred approximately every six to eight weeks for the past two years, but her most recent period was 12 months ago. She has tried soy supplements without adequate relief. She also notes recent vaginal dryness and pain on intercourse. She

1) La menopausia precoz, el tabaqu ismo y el consumo de tiazidas. 2) La menopausia precoz, la ingesta baja de calcio y el consumo de tiazidas. 3) La menopausia precoz, la ingesta baja de calcio y el sedentarismo. 4) El consumo de tiazidas, el consumo bajo de calcio y el consumo elevado de proteínas.

RC: 3

has no personal or family history of heart disease, stroke, venous thromboembolism or breast cancer. Pelvic examination reveals a normal cervix, slightly pale vaginal mucosa and no adnexal masses. The most likely treatment is: 1) 2) 3) 4)

Calcium and vitamin D. Benzodiazepines. Estrogen therapy. Estrogen-progestin therapy.

Correct answer: 4

Ginecología_y obstetricia_



Fisiología del embarazo

ORIEi'JTACIÓi'J

ENARM

Tema en el que, sobre todo, no hay que dejar de estudiar las modificaciones fisiológicas del embarazo.

El blastocisto es una estructura esférica rodeada por una cubierta llamada trofoblasto, que va a tener la misión de «excavar» el endometrio para efectuar la implantación. Después de la implantación, el endometrio se modifica y pasa a llamarse decidua . La zona embrionaria que está en contacto con la decidua uterina se modifica y se llama corion, que evolucionará hasta formar la

21.1. Fecundación e implantación

placenta.

El ovocito liberado tras la ovulación es recogido por el pabellón tubárico y

21.2. Placenta

conducido hacia el tercio externo, donde tiene lugar la fecundación. Los espermatozoides que llegan hasta el ovocito se fijan a la zona pelúcida y desencadenan la reacción acrosómica, que consiste en la liberación de las enzimas

La placenta ejerce una actividad decisiva en la nutrición, el crecimiento

contenidas en el acrosoma y que producen la hidrólisis de la zona pelúcida.

y el metabolismo fetal y desarrolla una actividad endocrina muy importante. Su formación comienza a partir del noveno día posfecundación

El primer espermatozoide que llega al espacio perivitelino se fusiona in-

(trofoblasto), pero no alca nzará su estructura definitiva (división en cotile-

mediatamente con el ovocito. Esta fusión provoca un cambio en el po-

dones) hasta el qu into mes.

tencial de membrana que constituye la seña l para que el ovocito se active. Al mismo tiempo se liberan al espacio perivitelino enzimas hidrolíticas

La placenta actúa como barrera, como lugar de intercambio materno-

que provocan cambios en la zona pelúcida que la hacen impermeable a

feta l y en la fo rm ación de sustancias con funciones endocrinas:

los demás espermatozoides (reacción cortica l). Med iante este mecanismo se garantiza la fecundación con un solo espermatozoide.

Función de barrera: evita la comunicación directa entre la circulación materna y la fetal, así como el paso de determinadas sustancias.

El óvu lo una vez fecundado es transportado a su lugar de implantación

Transferencia placentaria: la llegada de nutrientes maternos al feto

en el endometrio, cuando aún está en la fase de blastocisto. La implanta-

puede producirse gracias a diversos mecanismos: Difusión simple: pasan por gradiente de concentración, al

ción ocurre seis o siete días tras la ovulación (Figura 37).

encontrarse en mayor concentración en sang re materna. Es el mecanismo usado por los gases (0 2, CO), agua y la mayoría de electrólitos. No consume energ ía. Difusión facilitada: es característica de la glucosa. Aunque hay diferencia de gradiente (la cantidad de g lucosa es mayor en la sangre materna que en la feta l), se asegu ra su paso al feto, por medio de difusión fac ilitada. 4 días

Transporte activo: hierro, aminoácidos y vitaminas hidroso/ubles. Están en mayor concentración en sangre feta l, por eso necesitan luchar contra grad iente para pasar al feto consumiendo de ese modo energía. Pinocitosis: molécu las de gran tamaño (li poproteínas, fosfolípidos, anticuerpos lgG, determinados virus).

12 a 24 horas

Figura 37. Fecundación

e implantación

5-6 días implantación

Soluciones de continuidad: la existencia de pequeñas roturas placentarias puede permitir el paso de células intactas (como hematíes).

Ginecología y obstetricia

21

1

Acción fisiológica: Asegura el suministro de glucosa: estimula la lipólisis ma-

Insuli na, heparina e lgM no atraviesan la placenta

terna, y los ácidos grasos formados disminuyen la acción de la insulina, produciendo así hiperglucemia materna y con ello un mayor paso de glucosa al feto (sobre todo a partir de la segunda

Función endocrina (Tabla 23 y Figura 38)

m itad del embarazo).

Preparación de la glándula mamaria para la Jactancia, aun-

Gonadotropina coriónica humana (HCG)

que su importancia es muy inferior a la de la prolactina. Pequeña acción como hormona de crecimiento fetal.

Es la encargada de mantener el cuerpo lúteo. Tiene una función biológica parecida a la LH. Se detecta en la sangre materna t ras la implan-

Interés clínico: es prácticamente nulo, a excepción de su relación

tación.

con la masa placentaria.

Sus niveles aumenta n a lo largo del primer trimestre, alcanzando el máxiNacimiento

mo alrededor de la déc ima sema na (50.000 mUI/ml), pa ra d isminuir posteriormente.

-'E

Acción fisiológica:

S

Cl. ---

I

Función luteotrófica: sin duda la m ás im port ante. Ma ntie-

0\

e

ne el cuerpo lúteo las prim eras sema nas hasta q ue la pla-

o

ce nta es capaz de sin tetizar progestero na (síntes is q ue en

100

las primeras semanas de gestación correspo nde al cuerpo

160

< 120

10 a 25

5 a 10

< 5 o sin usoidal

Presentes

Ausentes

Ausentes

Deceleraciones

DI PI

DIP umbilica les

DIP II

Prueba de Pose

< 20% DIP 11 en 1Ocontracciones

20 a 30% DIP 11 en 1Ocontracciones

> 30% DIP 11 en 1Ocontracciones

pH sangre fetal

7,25 a 7,45

7,20 a 7,25

< 7,20

> 30%

10-30%

< lOo/o

pH prepatológico: 7,20-7,25; en este caso, es preciso repet irl o en 15-30 min . En est e in-

Buen pronóstico

Saturación 0 2 por pulsioximetría

Tabla 26. Eva luación del bienestar fetal

arteria umbilical. Presentan mayores tasas de morbimorta lidad perinatal.

Ideas clave 16 "

El diagnóstico de gestac ión se puede hacer con HCG. Su detección en sangre es el diagnóstico más precoz, mientra s que en orina es el diagnóstico habitual de gestación. Si se quiere un diagnóstico de certeza , este se hará mediante la ecografía.

"

La ecografía del primer trimestre permitirá d iagnosticar que existe embarazo, si es único o múltiple, si es intrauterino o extrauterino y la vitalidad del embrión. Ademá s, permite determinar la edad gestaciona l mediante la medición del CRL.

"

El Doppler permite conocer el estado de vasodilatación del feto y, por tanto, si existe o no bienestar fetal. El estadio 1corresponde a un aumento del índice de resistencia, el estadio 11 a la ausencia del flujo d iastólico y el estadio 111 a la presencia de un flujo diastó lico reverso. Estos dos últimos estadios tien en mal pronóstico y se finalizará la gestación.

"

El cribado de cromosomopatías consiste en pruebas no invasivas que permiten calcular el riesgo individ ual, y así decidir realizar o no una prueba invasiva .

"

Los marcadores ecográficos de cromosomopatías del primer trimestre son: translucencia nucal > 3 mm, higroma quístico, alteraciones del flujo en el conducto venoso de Arancio, ausencia de hueso nasal, y alteración de la morfología de la vesícula vitelina.

"

El cribado bioquímico se puede realizar en el primer trimestre, determinando HCG (patológica, si aumentada) y PAPP-A (patológica, si disminuida) o en el segundo trimestre, determinando AFP (patológica, si disminuida). estrio l (patológ ico, si d ism inuido) y g lucoproteína bl (patológ ica, si aumentada).

"

La ecografía del segundo trimestre servirá para evaluar el crecim iento y la morfología feta l. Es la ecografía del diagnóstico prenatal y presencia de insufic iencia tricuspídea .

"

El cribado aconsejado en la actualidad consiste en la combinación de edad materna, marcadores bioquímicos del primer trimestre y la medición de la translucencia nucal por ser la combinación con mayor sensib li dad.

"

La ecografía del tercer trimestre permitirá diagnost icar el crecimiento fetal y evaluar la p lacenta y el líquido amn iótico. Se pueden encontrar fetos pequeños para edad gestac ion al (PEG) constitucionales en los que no existen alteracione s estructurales ni en el Doppler. Los PEG anómalos presentan anoma lías genéticas, estructura les o secundarias a infección. Los fetos CIR son aque ll os co n una función placentaria alterada identificado po r una alteración del Doppler a nivel de la

"

El diagnóstico invasivo del cariotipo fetal se puede realizar med iante biopsia coria l, amniocentesis o funiculocentesis. Se elegirá fundamentalmente en función de la edad gestacional y del rie sgo de aborto asociado. La técnica más utilizada es la amniocentesis, por ser la más segura de todas ellas. La funiculocentesis se reserva para embarazos de edad gest aciona l avanzada, y además permitirá admin istrar tratamiento al mismo tiempo.

23 · Evaluación gestacional

Ginecología y obstetricia "

El registro cardiotocográfico es la prueba de valoración del bienestar fetal habitual, tanto anteparto como intraparto. Se valorará la FCF basal, la variabilidad, los ascensos y las desaceleraciones.

"

La presencia de una variabilidad silente o sinusoidal tiene muy mal pronóstico, e indicará la extracción fetal inmediata.

Casos clínicos Una mujer de 25 años consulta por un retraso menstrual de 3 semanas y metrorragia es·casa desde hace 24 horas. Su estado general es bueno, no tiene dolor, y la exploración ginecológica no muestra alteraciones, excepto el sangrado escaso procedente de la cavidad uterina. El test de embarazo es positivo y la determinación de j3 hCG en plasma es de 600 mUI/ml. Por ecografía transvaginal, se observa un útero normal con un endometrio homogéneo de aspecto secretor, de 12 mm de espesor. En el ovario derecho hay una formación que parece un cuerpo lúteo normal. No hay líquido libre en la cavidad abdominal. ¿Cuál de las siguientes es la indicación más correcta? 1) 2) 3) 4)

"

En un registro cardiotocográfico intraparto, aparecen dips 11 repetidos en relación con todas las contracciones, seguidos de una bradicardia fetal moderada. La paciente está recibiendo estimulación oxitócica i.v. y se registran 3 contracciones intensas cada 1O minutos. La exploración revela presentación cefálica (variedad O.I.I.A.) en 11 plano y una dilatación cervical de 6 cm. ¿Cuál es la conducta obstétrica adecuada en esta situación? 1) 2) 3) 4)

Retirar la estimulación oxitócica. Proceder a la extracción fetal inmediata mediante cesárea. Realizar una microtoma de sangre fetal para determinar el pH. Administrar dosis altas de 13-miméticos.

RC: 3

RC: 1

A 38-year-old primigravida was admitted to the hospital at 40 weeks gestation for a normal vaginal delivery. She had no history of medica! illness or trauma. lnitial fetal heart rate was 120150 beats/min and showed good reactivity and variability. When the diameter of the cervical opening was approximately 4 cm the fetus showed severe bradycardia. The most likely treatment is:

23

La presencia de al menos dos ascensos cada 20 minutos es necesaria para considerar reactivo el trazado. Su ausencia indicará la necesidad de realizar la prueba de Pose (o registro cardiotocográfico estresante), que consiste en administrar oxitocina y provocar contracciones, eva luando el número de desaceleraciones. Se considera positivo si aparecen > 30% de DIPS 11 y, en estos casos, se debe finalizar la gestación.

Repetir seriada mente cada 2-3 días la ecografía y la 13-hCG. Reposo absoluto y repetir la ecografía a las 2-3 semanas. Legrado uterino. Tratamiento con metotrexato por vía sistémica.

Case Study

1

1) 2) 3) 4)

Place the patient in lateral decubitus position. Determine fetal capillary pH. Forceps delivery. Emergency cesarean section.

Correct answer: 4

Ginecología y obstetricia_



Hemorragias del primer trimestre

ORIENTACIÓN

ENARM

Es un tema importante. Estudia sobre todo el embarazo ectópico.

Factores maternos: suelen ser abortos euplo ides. Infecciones: TORCH (t oxop las m osis, sífili s, rubéola, citomegaloviru s, herpes sim ple) .

Enfermedades crónicas graves. Endocrinopatías (d iabetes mell itu s no contro lada, hi potiroid ism o, defi ciencia de progesterona).

24.1. Aborto

Desnutrición grave (défi cit de vitami na A o de ácido fólico). Tabaco, alcohol. Toxinas ambientales (arsén ico, plomo).

El aborto es la te rminación de la gestación antes de las 20 semanas. Más del SO% de los casos ocurren antes de la 12• semana, por lo que se deno-

Factores inmunológicos: los anticuerpos antifosfolípidos, el an-

m ina entonces aborto precoz. Se denomina aborto ta rdío si sucede entre

ticoagulante lúpico y los anticuerpos anticardiol ip ina están rela-

las semanas 12• y 20•.

cionados con una mayor tasa de abortos. El t ratamiento de estas muj eres con hepa rina y aspi ri na en bajas dos is disminuye la tasa de

Incidencia

los mi sm os.

Factor uterino: m iomas, sínd rome de As herman, ma lformaciones La incidencia del aborto es d ifícil de eva luar, ya que muchas mujeres abor-

ute rin as.

tan sin ser co nscientes de ello. La incidencia del aborto clínico se aproxima

Incompetencia cervical : la in suficiencia cé rvico-ístmi ca es la etiolo-

al 10%, aunq ue es basta nte mayor si se t ienen en cuenta las gestaciones

g ía m ás frecuente del aborto tardío de repetició n.

preclín icas, que es el caso de lo comentado al in icio. En mujeres co n embrión vivo co nfirmado por ecog rafía a las ocho semanas, el aborto posterior

Clínica

sólo representa un 3%, por lo que el grueso de los casos de aborto clínico se produce antes de las ocho sema nas de gestación. El riesgo de aborto aumenta con el número de embarazos y co n la edad de los progen itores.

Se define mediante la exploración y los dat os ecográficos (Figura 48 y Tabla 27):

Amenaza de aborto: es la aparición de una metrorragia en la primera

Etiología

mitad de una gestación. Puede ir acompañada de dolor hipogástrico d iscontinuo leve. Es muy frecuente (20-25% de las gestantes). Aborta

Los factores que determ inan la etio logía del aborto son los siguientes:

menos de la m itad, y no hay daño feta l en los nacidos tras amenaza

Factores ovulares: la ca usa más frecue nte de aborto en la pobla-

de aborto. La sangre proviene de vasos sa ngu íneos maternos para pla-

ción genera l es la presencia de ano malías ovulares, siendo las altera-

centarios rotos. El orificio cervica l interno (OCI) perma nece cerrado y,

ciones cromosó micas el 50-60% de ellas:

por ecografía, se confi rma la vita lidad em brionaria o fet al.

Trisomía autosómica: es la ano malía m ás frecuente. Las m ás

Aborto inevitable (inm inente o en curso): hem orrag ia vag ina l y OCI

habit uales so n las que afect an a los cro m osom as 16, 22, 21, 13

abierto co m o co nsec uencia de la d inám ica uterin a.

y 18. La más rara es la tri somía q ue afecta al cromoso ma 1. La

Aborto consumado completo: ya se ha prod ucido la expu lsión to-

frecuenc ia aumenta co n la edad materna.

ta l de los restos.

Monosomía X (45,X o sín dro me de Turner): se asocia a edad

Aborto consumado incompleto: las contracciones uterinas han ter-

materna más joven.

m inado. Hay expu lsión de partes ovu lares, pero el útero no está vacío.

Triploidías, tetraploidías.

Aborto diferido: se ha producido la retención de una gestación no evolutiva en el útero durante varias semanas. Existe riesgo de

Otras causas son las anomalías del t rofoblasto, las mutaciones ge-

coagulopatías, como la coagu lación intravascular d iseminada por

néticas aisladas, los factores poligén icos y el envejecimiento de los

liberación de trombop lastinas placentarias, por lo que se deben de-

gametos.

terminar productos de degradación del fibrinógeno (PDF o DfMERO-

Ginecología y obstetricia

1

24

D).En la clínica, se llama aborto diferido cuando se detecta muerte

Por otro lado se puede sospechar un embarazo ectópico cuando la eco-

embrionaria o su ausencia (huevo huero) antes de que comience un

grafía transvagi nal obj etiva un útero vacío y la gonadotropina coriónica

aborto en cu rso. Aborto recurrente o habitual (0,3%): se de nom ina as í en caso

en sa ngre es superior a 1.800 m iU/mL

de ocurrir tres o más abortos espont áneos consecutivos o cinco

La hemorrag ia del primertrimest re ha de p lantear un diagnóstico diferen-

alternos.

cial con cérvix friable, traumatismos, pólipo u otros procesos malignos.

Se deben estudiar alteraciones cervicouterinas, enfermedades metabólicas y cariotipo de los progenitores. Estas mujeres t ienen mayor

Diagnóstico

riesgo de pa rto pretérmino, placenta previa y malformaciones fetales en embarazos posteriores.

El test de embarazo y la ecografía son los métodos de elección: se compru eba si existen viab il idad feta l.

Tratamiento Amenaza de aborto: ningún tratamiento ha mostrado verdadera eficacia. Se suele recomendar reposo re lativo y abstinencia sexual. No se ha demostrado que los progestágenos ni los uteroinhibidores tengan ninguna util idad. Aborto: se puede optar por un tratam iento qu irú rgico o por una evacuación farmaco lóg ica . El t rat amiento qu irúrg ico consiste en Amenaza de aborto

Aborto inevitable

rea liza r un leg rado baj o anestesia general; es el proced im iento más sencillo de evacuación uteri na: si el cuel lo está cerrado, se procede a su dilatación con tal los de Hegar y si está dilatado, la actuación es directamente evacuar el conten ido con la cuchari lla o legra (Figura 49) . Para la evacuación farmaco lóg ica se utilizará misoprostol administrado por vía vaginal. Está ind icado en abortos en gestaciones con menos de 1O semanas y después de la semana 12 por los riesgos que presenta el legrado en estas semanas de gestación . Está contra ind icado en pacientes con g la ucom a y asm a. Otra alternativa es el em pleo de oxitócicos so los o asoc iados a p rostag landinas. No se

Aborto diferido

Aborto incompleto

debe olvida r la admi nistración de inm unoglobulina anti-D en caso de que la mujer sea Rh negativo. Para pacientes con amenaza de aborto o aborto menor de 13 semanas,

Aborto completo

se administrará 50 mcg i.m. de gammaglobu lina anti D. Si la gestación es mayor de 13 semanas, la dosis asciende a 300 mcg. Figura 48. Tipos de aborto

Amenaza de aborto

Aborto en curso

Aborto diferido

Metrorragias

Escasa

Abundante

+1-

Dolor

Leve

Intenso

+1-

Cérvix (OCS)

Cerrado

Abierto

Cerrado

Ecografía

Embrión normal (con latido)

Restos de embrión o no

Embrión muerto (sin latido)

El único tratamiento para la amenaza de aborto es el reposo.

Tabla 27. Diagnóstico diferencial de los tipos de aborto Embarazo de viabilidad incierta: saco intrauterino menor de 20 mm sin vesícula vitelina o sin embrión. O bien, un embrión menor de 6 mm, sin latido cardíaco. Para confirmar la viabilidad de la gestación ha de realizarse nueva en ecografía en una semana. La gonadotropina coriónica puede detecta rse hasta en niveles de 25 UI!L (n ivel aproxi mado a los 9 días de la fecu ndación (23-28 del ciclo genita l femenino). Su determinación cada 48 horas es útil para el manejo del aborto, ya que los niveles se duplican en una gestación normoevolutiva y descienden en un embarazo interrumpido.

Figura 49. Legrado uterino

Manual CTO de Medicina y Cirugía, 2. 8 edición

Complicaciones del aborto

Diagnóstico: se realiza mediante la clínica y los antecedentes: historia previa de dos o más abortos tardíos y d ilatación del OCI de 2-3

Coagulación intravascular diseminada con f ra caso renal.

cm en la exploración .

Aborto sépti co: se debe lleva r a cabo leg rado inmediato y trata-

Tratamiento: el tratamiento de la incompetencia cervicouterina es

miento antibiótico de amplio espectro (aerobios y anaerobios). La

el cerclaj e cervica l entre las sema nas 14- 16, que consiste en una serie

demostración de C/ostridium es indicación de histerectom ía.

de técn icas quirúrgicas para «cerra r>> el cérvix.

Síndrome de Asherman (sinequ ias uterinas poslegrado).

Se rea liza previamente una ecog rafía para confirmar que el feto está

Perforación uterina durante el legrado: si hay estabilidad he-

vivo y excluir ma lfo rm aciones. El cerclaj e se retira a las 38 semanas o

modinámica, se puede intentar un tratamiento conservador, pero

antes, si se desencadena el parto o si apa rece infección intrauterina.

si apa recen signos de gravedad o inestabil idad hemod inámica, se practicará cirugía. Pérdidas gestacionales de repetición

Síndrome antifosfolípido El tratam iento en la gestante es controvertido, aunque se puede resumir

Enfermedades crónicas descontroladas

de la siguiente manera: Presencia de anticuerpos antifosfolípido sin antecedentes de trombosis o aborto: no req uieren tratamiento aunque algu nos au-

Tratamiento

tores recomiendan admin istrar AAS en dosis bajas. Presencia de anticuerpos antifosfolípido y abortos precoces: AAS y heparina, aunque algunos autores ind ican ún icamente AAS en dosis bajas. Presencia de anticuerpos ant ifosfolípido con trombosis o muerte fetal: AAS y heparina (ante tratam ientos prologados, se recomien-

Antecedentes de preeclampsia, insuficiencia placentaria o trombosis en emba raz0s previos

da suplementar con ca lcio y vitam ina 0 3).

Incompetencia cervical Se produce d ilat ació n indolora del cuello uterino (no es provocada por

Malformaciones uterinas

Incompetencia cervica l

Corrección quirúrg ica

Cerclaje

Descartar SAF o lupus

contracciones co mo el aborto en cu rso) durante el segu ndo trimestre de la gestación con prolapso de las membranas, amnio rrexis y expulsión de un feto inmaduro. La d ilatació n cervica l pocas veces se hace evidente antes de las 16 semanas (Figura 50).

Figura 51. Manejo del aborto recurrente

24.2. Gestación ectópica La gest ació n ectópica es aquella que está im plantada fuera de la cavidad endometri al.

Etiología El denom inador com ún es el retraso en el transporte del óvulo, ya que se implanta all í donde se encuentre en el 6. 0 -7. 0 día posfecundación. Son factores de riesgo los sigu ientes: Ant ecedentes de gestación ectópica. Cirugía t ubárica previa. Enfermedad inflamatoria pélvica. Insuficiencia

Dispositivo intrauterino. Exposició n a d ietiletilbestro l. Tabaquismo. Técnicas de reproducción asistida (inducción de la ovulación, etcétera).

Figura SO. Inco mpetencia cervical

Frecuencia

Etiología: no siempre es clara, pero a veces se relaciona con trauma-

Su frecuencia se encuentra entre el 1-2%. Hay un incremento en los últimos

t ismos cervica les como la conización o el desa rrollo cervical anóma-

años, aunque ha descendido su mortalidad. La coexistencia de embarazo ec-

lo (exposición previa al estil bestrol intraútero).

tópico y eutópico es excepcional (1 /30.000) y se llama embarazo heterotópico.

24 · Hemorragias del primer trimestre

Ginecología y obstetricia

1

24

Localización

La HCG se duplica cada dos días durante las primeras semanas del embarazo normal, llegando a un máximo hacia la décima semana. En los

La loca lización más habitua l es en la t rompa de Fa lopio (97%): en la porción ampu lar (78%). Le siguen en frecuencia: istmo, fimbrias, interst icio,

embarazos ectópicos, la HCG crecerá más despacio de lo habitual (aumenta aproximadamente un 50% en 48 horas). La HCG también ayuda en

ovario, cavidad abdominal (Figura 52).

el d iag nósti co diferencial con procesos como abdo men ag udo o aborto comp leto. El diagnóstico de confirmación es la laparoscopia y anatomía patológica, pero se practicará en caso de d uda diagnóstica. La punción del fondo de saco de Doug las (cu ldocentesis), aunque útil, se ha abandonado por d isponer de técnicas mejores. La clásica demostración de decidua y de ausencia de ve llosidades coriales (signo de Arias-Ste lla) no es conside rada patog no m ón ica, pero sí altamente sospec hosa .

Evolución En el 10% de los casos, hay resolución espontánea: aborto tubárico (casi exclusivamente los ampu lares). En el 90% restante, aparece rotu ra tubárica, por la poca d istensibilidad de sus paredes. Suele haber intensa hemorrag ia porque el trofoblasto invade vasos arteriales.

Figura 52. Incompetencia cervical

Tratamiento Clínica Actua lme nte se dis pone de tres pos ib ilidades en el trata mi ento: conNo hay ningún signo ni síntoma patognomónica y la cl ínica es muy inespecífica. En general, es dolor en anejo y pélvico, j unto a signos de gestación inci-

ducta expectante, tratam iento méd ico o t ratamiento qu irú rgico. Conducta expectante: puede ser util izado sólo en pacientes asin-

piente: amenorrea de unas seis a ocho semanas, útero aumentado de tamaño, pero menor que amenorrea, doloroso a la movilización cervical y anejo

tomáticas, con diagnóstico ecográfico de embarazo tubárico sin evidencia de sangrado y niveles de P-HCG menor a 1.000 Ul/ml. Se

discretamente tumoral, doloroso. Es frecuente un escaso sangrado vaginal,

realizará seguimiento con P-HCG y ecografía semanal para asegurar la dism inución del tamaño de la masa anexial y de las concentracio-

oscuro, intermitente. En el caso de rotura tubárica, se añade intenso dolor en fosa ilíaca, Doug las sensible y signos de peritonismo, así como la clínica

dad, y en el 1Oo/o de los casos hay resolución espontánea (aborto tubá rico).

nes de P-HCG. Tratamiento médico: el t ratamie nto con metotrexa t o está ind icado en pac ientes con estab ili dad hemod inámi ca, embarazo ectóp ico no roto, sin datos de sangrado activo intra-abdomina l

Diagnóstico

y concentracio nes bajas de P-HCG (menores de 2.000 Ul/m l). Las contra indicaciones absolutas para el uso de metotrexato son la

La ecografía transvaginal es el primer paso en la exploración d iagnóstica. Puede dar un diagnóstico de segu ridad en caso de demostra r la presen-

enfermedad ácido péptica e h ipersens ib il idad al fármaco . El saco gestacional mayor a 3,5 cm y la actividad cardíaca embrionaria

cia de saco gestaciona l en la trompa, con embrión y latido cardíaco, lo

son contra indicaciones re lativas pa ra el tratamiento médico. Se añade ác ido folínico para preven ir la toxic idad. El segu imiento

secundaria a la hemorragia (abdomen agudo). Puede ser fu lminante y entrar en shock en minutos. Tras la crisis, se alivian los síntomas, aunque no la grave-

lactancia, d isfunción renal, hepática, pulmo nar o hematológica,

que sucede en el 5% de los ca sos (Figura 53).

se hará tamb ién co n d ete rminacio nes seriad as de P-HCG y ecografía. Tratamiento quirúrgico: se realiza rá preferentemente mediante laparoscopia, au nque dependerá de la paciente, de sus deseos genésicos y de la local izac ión del emba razo ectópico. La técnica debe ser conservadora y consiste en la salpingostomía linea l que se real iza en el borde libre y con aspirac ión del conten ido ovular

(Figura 54). En los casos en los que la trompa esté muy dañada, haya gestación ectópica recurre nte o los deseos ge nésicos estén cum pl idos, se extirpa la trompa afectada: sa lpinguectom ía total. Si hay inestabilidad hemodinámica, se practicará una laparotomía ur-

Figura 53. Gestación ectópica ampular vista en ecografía transvaginal

gente.

Manual CTO de Medicina y Cirugía, 2.a edición

Mola parcial: existe tej ido embrionario y/o amn ios además del tejido trofoblástico con degeneración hid rópica. Existe material genético materno y el cariotipo es triploide como consecuencia de la fecundac ión del óvulo por dos espermatozoides (69,XXY). El riesgo de ETP es menor (4-8%).

Diagnóstico Se basa en la clínica, en la exploración, en la ecografía y en la determinación de HCG. La prod ucción de HCG es mucho mayor que en una gestación norma l debido al desarrollo trofoblástico excesivo, incluso llegando al millón de unidades, siendo de gran util idad t anto en el diagnóstico como en el seguimiento de la enfermedad trofoblástica gestacional. Clínica: Figura 54. Laparoscopia en gestación ectópica

Metrorragia: es el signo más habitual y el principal motivo de consulta. Puede ir acompañado de dolor hipogástrico. La expulsión de vesículas es un signo infrecuente pero, de apa recer, es patognomón ica.

24.3. Enfermedad trofoblástica

Náuseas, vómitos e hiperemesis gravídica: se debe al incremento en los niveles de HCG. El término enfermedad trofoblástica engloba una serie de patologías en las que hay una proliferación anormal re lacionada con la gestación. Se ha visto cómo el trofoblasto invade el endometrio en busca de oxíge-no.

Preeclampsia: ocurre en el 25% de las pacien tes con mola. Es una de las pocas circunstancias en las que esta comp licación t iene lugar antes de la semana 20.

Cuando la vellosidad trofoblástica encuentra una zona rica en oxígeno,

Hipertiroidismo: se explica por la simi litud de la HCG con la

detiene su crecimiento.

TS H.

Si una zona de trofoblasto no tiene mesodermo feta l que produzca vasos sanguíneos, no hay buena transm isión de oxígeno y la tendencia a la

Exploración: útero de tamaño mayor amenorrea: es regular, simétrico y de consistencia blanda .

proliferación permanece: enfermedad trofoblástica. La ausencia de vasos linfáticos permite que se acumule líqu ido extracelular, por lo que el tejido adopta una disposición quística. Por tanto, la enfermedad t rofoblástica es

Tumoraciones ováricas: son los qu istes tecaluteínicos (Figura 55) y se deben a la estimu lación ovárica de la HCG

quística y avascu lar.

(recordad q ue HCG, FSH, LH y TSH tienen una subunidad a común) . Ocurren en el 30% de los casos. No req uieren tratamiento, ya que regresan espontá neamente al se r evacuada la mola. Son producto res de progesterona.

Epidemiología La enfermedad t rofoblástica gest acio nal ocu rre en 1/1.500 emba razos. La mayoría regresa espontáneamente (80%). El 15% evoluciona a enfermedad trofoblástica persistente (ETP) no metastásica y el So/o a ETP metastásica. La mola invasora (enfermedad trofoblástica con capacidad de traspasa r el endometri o) sucede ap roximadamente en 1/15.000 emba razos. La mitad de los coriocarcinomas (enfe rmedad trofoblástica ind iferenciada, con capacidad de traspasa r el endometrio y sin tendencia a formar vellosidades) provienen de una mola, el 25% de abortos y el 20% de un emba razo norma l. Metastatiza por vía sang uínea.

Clasificación Existen dos entidades muy diferentes: Mola completa: es la forma m ás frecuente de presentación. Se caracte riza por la ausencia de tej ido embriona ri o y de am nios y la degeneración del tejido trofoblástico. Se origina como consecuen-

Figura 55 . Quiste tecaluteínico ovárico en enfermedad trofoblástica

cia de la fecundac ión de un óvulo con material genético ausente o inactivo. Pcir el lo, la carga genéti ca es exclusivamente de origen pa-

Ecografía: es la técnica diagnóstica de elección . Se observa cavidad ocupada por mu ltitud de ecos de baj a amplitud, que corresponden

tern o. La mayor pa rte de los casos, la fecundación se prod uce por un espermat ozoide 23,X, que se divide da ndo un ca ri otipo 46,XX. En un pequeño porcentaje de casos se p roduce la fecundac ión por

al tej ido t rofoblástico pro liferado. Es la imagen típica de "copos de nieve" (Figura 56).

dos espermatozoides (23,X y 23,Y) dando un ca riotipo 46,XY. El ri es-

El diagnóstico de certeza lo proporciona la anatomía patológica tras el

go de ETP es del 15-20%.

legrado.

24 · Hemorragias del primer trimestre

Ginecología y obstetricia

1

24

Enfermedad trofoblástica ---+ Legrado Enfermedad trofoblástica persistente ---+ Quimioterapia+ Legrado l.J

100

u I

o:..

75

50

25

Figura 56. Mola hidatiforme

Figura 57. Seguimiento de la enfermedad trofoblástica

Tratamiento

Se define como curación la ausencia completa de evidencia clínica y ana lítica de enferm edad durante cinco años.

El tratamiento consta de dos partes: la evacuación de la mola y el segui-

Enfermedad trofoblástica persistente

miento posterior de la enfermedad. El tratamiento evacuador de elección es el legrado por aspiración.

Se caracteriza por la retención de tejido molar y la elevación contin uada de los niveles de HCG, pasadas ocho semanas desde la evacuación. Se encuentra una persistencia de los signos y los síntomas. En estas circunstancias, es necesario descartar enferm edad con afectación extrauterina.

En mujeres con mayor riesgo de degeneración maligna, elevada paridad o en aquellas con deseos genésicos cumplidos y con más de 40 años, se practicará histerectom ía total con mola in situ. No está indicada la quimioterapia en esta etapa .

Si los métodos exploratorios son negativos, se presume que la enfermedad está li mitada al útero. El tratamiento consistirá en:

En el80o/o de los casos, la en ferm edad regresa después de haber evacuado el útero. Los niveles de HCG descienden rápidamente. Cuando estos

Metotrexato o actinomicina: se asocia a ácido folínico para disminuir los efectos secundarios. Interfiere en la embriogénesis, por lo

niveles no bajan o permanecen elevados a las ocho semanas, se habla de enfermedad persistente.

Seguimiento (Figura 57) Determ inaciones semanales de HCG hasta la remisión completa

(cese de la hemorragia, útero involucionado, anejos normales y niveles normales de HCG durante tres semanas).

Enfermedad trofoblástica

t

j

Multíparas > 40 años Deseos genésicos cumplidos

t

Exploración clínica repetida Histerectomía simple ~ Determinaciones semanales de P-HCG Rx tórax, pruebas de función hepática, con mola in situ TC craneal (si clínica)

-

Legrado con aspiración

Después de la remisión completa, se harán determinaciones de HCG mensua lmente durante seis

meses y bimensuales durante otros se is meses.

p-HCG elevada 8 semanas después de la evacuación

Remisión completa

Cese hemorragia útero involucionado Anejos normales 13-HCG normal 3 semanas

J

Exploración clínica ca da dos semanas, hasta que se produzca la remisión completa. Radiografía de tórax en el mo-

P-HCG mensual6 meses P-HCG bimensual otros 6 meses Anticonceptivos 1 año

mento de ser evacuada la mola,

de anticonceptivos orales (una gestación aumenta las cifras de

Alteraciones

Normal

t

t

para descartar afectación extrauterina. Se recomienda evitar el embarazo durante un año con la toma

Rx tórax Pruebas de función hepática TC craneal

..,.a..., liz-ad lo-ca"'"" dad ,... fe-rm_e..,.. ""'"n.,... ,..E

---------------+

Enfermedad trofoblástica

Mola invasora o coriocarcinoma

Bajo riesgo

Metotrexato

Metotrexato

1

persttente

P-HCG e impide saber si la enfer-

Metotrexato

medad ha regresado). Inicio rápido de la quimioterapia

+

+

Legrado/ histerectomía

Histerectomía

si los niveles de HCG persisten.

Enfermedad metastásica* Alto riesgo

1

Poliquimioterapia

* En la enfermedad metastásica la histerectomía no tiene utilidad

Figura 58. Tratamiento de la enfermedad trofoblástica

Manual CTO de Medicina y Cirugía, 2.a edición

HCG: > 100.000 m U/mi.

que se debe esperar un año, tras final izar el tratam iento, para queda r embarazada.

Duración mayor de cuatro meses.

Legrado: se realizará en el terce r día de la qu imioterapia.

Metástasis cerebra les o hepáticas.

Histerectomía: en multíparas o en mujeres con paridad satisfecha.

Fracaso de qu im ioterapia previa . Edad materna superior a 40 años.

Enfermedad trofoblástica metastásica Buen pronóstico: no presenta ningún factor de riesgo. Es aquella situac ión en que la enfermedad sobrepasa los límites del útero. Las metástasis más frecuentes son en pulmón (?So/o) y después en vag ina

El tratamiento cons istirá en:

(SOo/o), ce rebro e hígado. Pueden ser los primeros síntomas de la enferme-

Mal pronóstico: el tratamiento más eficaz es: qu imioterapia com-

dad tras un aborto o un embarazo ectópico. Toda mujer con hemorragia

binada. El régimen EMA-CO (et opóxido, m etotrexato, actinom icina,

o tumor en cua lquier órgano q ue tenga una hi storia reciente o remota de

ciclofosfam ida y vincristina) es el m ás utilizado y co nsigue tasas de

embarazo molar, aborto o parto, debe ser sometida a un análisis de HCG

supervivencia del 80- 1OOo/o.

para descarta r una enfermedad t rofoblástica gestacional.

Buen pronóstico: m o not erapia con metotrexato o actinom icina. Curaciones del1 OOo/o. Es menos tóxico que la combinada.

La enfermedad trofoblástica metastásica se clas ifica en:

Mal pronóstico: presencia de algún factor de ri esgo de entre los

En la Tabla 28 se resume el d iagnósti co diferencia l de las hemorragias del

sigu ientes:

primer trimestre.

-

Hemorragia

Dolor

Aborto

Embarazo ectópico Inminente

En curso

Escasa

Roja con coágulos

Escasa, intermitente, oscura

Roja con coágu los

Continua

Amenaza

Leve

Otros síntomas

Complicado Continua A veces vesículas (esto es diagnóstico)

Con las contracciones

Leve

Contracciones uterinas

Signos subjetivos de gestación

En puñalada

Leve

Fiebre

Anemia

Afectación del estado general

Hiperemesis

Anemia

(Hipertiroid ismo)

Preeclampsia

Hipotensión

Shock Exploración

Dilatación cervica l

útero gestaciona l normal

OCI abierto

OCI cerrado ~-H CG

Normal

Disminución progresiva

útero menor que amenorrea, tacto vaginoabdominal doloroso, a veces se palpa masa anexial

Tacto vagina-abdomina l muy doloroso, signos de irritación peritonea l

· Cifras bajas para amenorrea

útero mayor que amenorrea

Muy elevada

· Crece menos del 66o/o a las 48 h Ecografía

Latido feta l presente

Latido ausente

Saco gestacional en trompa, útero vacío

En incompleto o diferido, se ven restos Tratamiento

Reposo relativo

Legrado

Abstinencia sexual

Tratamiento de las complicaciones

Latido ausente No saco gestacional Imagen "en copos de nieve"

Salpingostomía por laparoscopia En casos seleccionados, actitud expectante o metotrexato (si cifras de ~-HCG son bajas, diámetro< 4 cm y no rotura)

Salpinguectomía

Legrado por aspiración Seguimiento con HCG Quimioterapia si ETP o metastásica

Tabla 28. Hemorragias del primer trimestre

Ideas clave

z

"

El aborto y la amenaza de aborto es la causa m ás frecuente de hemorragia del primer trimestre.

"

La causa más frecuente de los abortos precoces (primer trime stre) son las anomalías ovulares (la mayor parte son anomalías cromosó micas).

24 · Hemorragias del primer trimestre

"

La causa m ás frecuente de los abortos tardíos (segundo trimestre) es la incompetencia cervical. Se relaciona con los antecedentes de conización por displasia cerv ical y con malformaciones cervi ca les. No tiene tratamiento, pero sí prevención, mediante la colocación de un cerc laje cervical en las semanas 14-16.

"

El síndrom e antifosfolípido puede originar también abortos de repetición. El tratam iento en las pacientes portadoras de anticuerpos positivos es co n aspirina y heparina. Recordad que los

Ginecología y obstetricia

" El tratamiento del aborto consiste en la realización de un legrado bajo anestesia tras dilatación cervical. En los casos de abortos tardíos, se prefiere la administración de prostaglandinas que abren el cérvix y posteriormente realizar el legrado. La amenaza de aborto se trata con reposo relativo y abstinencia de relaciones sexuales. " Son factores de riesgo de embarazo ectópico: antecedentes de ectópico, la cirugía tubárica previa, la enfermedad inflamatoria pélvica, la esterilidad y las técnicas de reproducción asistida, el uso del DIU y la endometriosis.

"

La enfermedad trofoblástica es la causa más rara de hemorragias del primer trimestre. La clínica es la presencia de metrorragia, útero mayor que amenorrea y, sobre todo, mucha sintomatología típica de embarazo (náuseas y vómitos). En ocasiones puede aparecer preeclampsia, hipertensión precoz y síntomas de hipertiroidismo.

"

El diagnóstico de sospecha se realiza mediante ecografía, con la visualización de una cavidad uterina ocupada por una imagen multivesicular (imagen en nevada), aunque la confirmación será anatomopatológica, tras la rea lización de un legrado por aspiración.

"

Se debe realizar un seguimiento poslegrado mediante la determinación semanal de HCG. Cuando tres valores son negativos, se pasa a realizar controles mensuales. Sin embargo, si no se negativiza, nos indicará que sigue existiendo enfermedad y hay que buscarla. En el estudio de extensión, se rea lizará una Rx de tórax, TC abdominal y TC cerebral. Si todo el estudio es negativo, se estará ante una enfermedad trofoblástica persistente que se tratará con metotrexato y un segundo legrado.

" La localización más frecuente es la porción ampular de la trompa. La clínica característica es la aparición de metrorragia escasa asociada a dolor abdominal. " El diagnóstico se realiza fundamentalmente mediante ecografía, al visualizar una cavidad uterina vacía y una imagen anexia l sugestiva/diagnóstica de gestación. En caso de duda, es posible ayudarse con la determinación de HCG. En el embarazo ectópico presentará niveles más bajos, y además aumentará más lentamente que en el embarazo normal. El tratamiento del embarazo ectópico podrá ser conservador cuando sea pequeño (< 4 cm), tenga valores bajos de HCG y

"

Casos clínicos Primigesta de 32 años que acude a urgencias refiriendo sangrado genital y prueba de embarazo en farmacia positiva. Se realiza exploración ginecológica, y los hallazgos ecográficos y analíticos sugieren el diagnóstico de mola vesicular. ¿Cuál sería el tratamiento de elección? 1) 2) 3) 4)

Quimioterapia con metotrexato. Legrado uterino con legra de Recamier y pinza Winter. Legrado por aspiración. Histerectomía simple (conservado ovarios).

24

no haya sangre libre en la cavidad abdominal. Cuando no cumpla estos requisitos o cuando haya fracasado el tratamiento con metotrexato, se indicará el tratamiento quirúrgico. La laparotomía urgente se reserva para las pacientes que llegan con un abdomen agudo e inestabilidad hemodinámica.

tratamientos prolongados con heparina necesitan de la sup lementación de calcio y vitamina D3 • " Las complicaciones que se relacionan con el aborto son la coagulación intravascular diseminada (por liberación de tromboplastinas placentarias), el aborto séptico, la perforación uterina y el síndrome de Asherman (presencia de sinequias uterinas que provocan amenorrea secundaria) .

1

" La enfermedad metastásica se trata sólo con quimioterapia. El tipo de quimioterapia lo determinarán los factores pronósticos (HCG > 100.000, > 4 meses, metástasis cerebrales o hepáticas, fracaso de quimioterapia previa y edad> 40 años). Si tiene buen pronóstico, se utilizará el metotrexato o actinomicina como único quimioterápico, mientras que si tiene mal pronóstico, se aplicarán protocolos de poliquimioterapia.

Paciente que presenta amenorrea de 7 semanas, asintomática, sin evidencia, mediante ecografía vaginal, de útero ocupado. Se determina la fracción b de la gonadotropina coriónica humana, obteniéndose una cifra de 2.500 m U/mi. Ante estos datos, en primer lugar habrá que pensar en: 1) Gestación de evolución normal correspondiente a la amenorrea. 2) Gestación normal con menor tiempo de evolución del correspondiente a su amenorrea. 3) Gestación ectópica de evolución asintomática . 4) Aborto precoz completo, con expulsión total de restos ovulares intrauterinos.

RC: 3

RC: 3

Una mujer en su cuarto mes de embarazo comienza a sangrar, y el médico le detecta un útero más grande que el esperado para su edad gestacional. Decide interrumpir el embarazo, y le practica un legrado. El patólogo, al examinar la muestra, observa numerosos quistes avasculares con proliferación significativa del trofoblasto, no hay feto y no se detecta invasión del miometrio. ¿Cuál será el diagnóstico?

Una paciente, con antecedente de aborto tardío anterior, ingresa a la 18• semana de gestación por dolor hipogástrico leve, no acompañado de hemorragia. En la exploración se aprecia cérvix dilatado y bolsa íntegra prominente. Dos horas más tarde, se produce un aborto consumado incompleto. ¿Cuál es la etiología más probable de este cuadro?

1) 2) 3) 4)

Mola invasiva. Mola hidatiforme parcial. Mola hidatiforme completa. Tumor trofoblástico del lecho placentario.

1) 2) 3) 4)

Síndrome antifosfolípido primario. Insuficiencia cérvico-ístmica . Mioma uterino de localización submucosa. Síndrome de Asherman.

RC: 2 RC: 3

Manual CTO de Medicina y Cirugía, 2.a edición

Case Study

··

A 20-year-old primigravida presented to the emergency department with amenorrhea of six weeks duration, acute pain in the abdomen, dizziness, vomiting and vaginal bleeding of one day's onset. Physical examination showed very poor general condition. She was severely anemic, with pulse 120/minute and blood pressure 90/60 mmHg. There was severe abdomen and pelvic region tenderness. Vaginal examination revealed a bulky uterus, with a mass of 4 x S cm which was felt in the right fornix

24 · Hemorragias del primer trimestre

with cervical motion tenderness and slight bleeding. Sonography revealed a right adnexal mass with a large amount of free fluid in the peritoneal cavity. The most likely treatment is: 1) 2) 3) 4)

Curettage. Methotrexate. Salpingostomy by laparoscopy. Salpingectomy by laparotomy.

Correct answer: 4

Gin eeo1ogi_a_y_ob ste t ei.___a----· n____,·

Hemorra gias del tercer trimestr e

ORIENTACIÓN

ENARM

Tema importante para el conjunto de la asignatura.

Marginal: la placenta llega al borde del OCI, pero no lo sobrepasa. Inserción baja: el borde placentario se encuentra en el segmento inferior, a menos de 7 cm del OCI.

Etiología

25.1. Placenta previa La causa específica de la placenta previa es desconocida. Son factores favorecedores los siguientes: La placenta previa consiste en la inserción de la placenta en el segmento inferior del útero, para lo que puede ocluir el orificio cervica l interno. Es la primera causa de hemorragia del tercer trimestre. Se clasifica de acuerdo a la distancia entre el borde placentario y el orificio cervica l. Según la re lación de la placenta con el orifi cio cervical interno, se clasifican en los siguientes tipos (Figura 59):

Embarazo múltiple: la p lacenta es de mayor tamaño, lo que aumenta el riesgo de que llegue a ser previa. Cicatriz uterina anterior: la incidencia crece con el número de cesáreas previas, dado que la placenta, al no poder anidar sobre la cicatriz, "busca" asiento en otras regiones, llegando hasta niveles más bajos de la cavidad uterina.

Multiparidad: la paridad y la edad avanzada incrementan el riesgo de placenta previa, por un mecanismo similar al anterior. Mujeres fumadoras: se d uplica el riesgo (probablemente porque la hipoxemia conlleva una hipertrofia p lacentaria compensadora).

Clínica La hemorragia roja, abundante, discontinua, recidivante e indolora es el síntoma típico de la placenta previa. De inicio, suele cesar espontáneamente pero, por lo genera l, se repetirá y con mayor intensidad. La sangre es de color rojo brillante, con tendencia a formar coágulos. No existe afectación feta l a no ser que aparezca hipovolemia materna secundaria a la hemorragia. La prematuridad es la mayor amenaza para el feto.

Diagnóstico La ecografía transabdominal o transvaginal es el método diagnóstico de elección. Localiza la placenta y evalúa la estática fetal. No debe hacerse nunca un tacto vaginal a no ser que todo esté preparado para realizar una cesárea de inmediato.

Figura 59. Tipos de placenta previa Central total: la placenta cubre la totalidad del OCI aún con dilatación. Central parcial: la placenta cubre el OCI cuando el cuello está cerrado pero cuando hay una dilatación igual o mayor a 3 cm sólo cubre parcialmente el OCI.

En la placenta previa no debe hacerse tacto vaginal.

Manual CTO de Medicina y Cirugía, 2 .a edición

Tratamiento

Trau mat ismos: a veces se asocia a un traumatismo o a una amniocentesis. También se ha asociado a la rápida reducción del tamaño

La placenta previa centra l es indicación de cesárea cuando haya ma-

uterino, al romper la bolsa amniótica en un polihid ram nios, o a la

durez pulmonar fetal.

cortedad del cordón.

La d istancia entre el OCI y la placenta por ecografía vag inal después de la semana 35 de gestación ayudará a decid ir la vía del parto en las placentas previas ma rg inales y de inserción baja. Cuando el borde

Nutricional: déficit de ácido fólico. una mayor incidencia de abruptio placenta rio.

de la placenta se encuentre a > 20 mm del OCI se puede ofrecer un

Hipofibrinogenem ia congén ita: parece ser un factor de riesgo.

parto vaginal con altas posibilidades de éxito. Una distancia menor se asocia con un porcentaje más alto de cesárea aunque en los casos seleccionados se puede dejar el parto vagina l. En caso de feto pretérmino, la actitud será expectante. Se ingresará a la paciente, pautándose corticoides para la madurez pulmonar fetal

El consumo de tabaco, alcohol y cocaína se han relacionado con

Fisiopatología La placenta se desprende, provocando gran sangrado. Para dism inuir la hemorrag ia, el miometrio se contrae y comprime las arterias espirales. Esta contracción uterina es tan intensa y generalizada que impide la cir-

y, si tiene contracciones y el sangrado no es importante, se pueden admin istrar tocolíticos. No obsta nte, si a pesa r de ser el feto pretér-

culación uteroplacentaria, por lo que apa rece hipoxia fetal y mayor ten-

m ino, la hemorragia es muy abundante o hay signos de sufrim iento

dencia al desprendim iento (Figura 60).

fetal, se realizará una cesárea urgente. Si se detecta muerte fetal, debe intentarse un parto vaginal, salvo en el caso de una placenta centra l. El abruptio es la causa más frecuente de sangrado intraparto.

Complicaciones Es frecuente la hemorragia posparto, ya que en el segmento inferior es

Signos y síntomas

más difícil conseguir la hemostasia.

25.2. Abruptio placentae Es el desprendimiento prematuro de la placenta normalmente inserta (suele abreviarse como DPPNI). Suele ocurrir en el tercer trimestre, pero se puede ver desde la 20• semana.

Abruptio incipiente: la zona desprendida es aún menor del25% del tota l de la zona de inserción de la placenta. No hay afectación feta l ni materna. La sangre puede salir al exterior en forma de hemorragia vaginal escasa, o es posible que quede retenida como hematoma retroplacentario. Como consecuencia del aumento del tono de miometrio para contener la hemorrag ia, el útero puede no relajarse completamente entre las contracciones. Existe la posibilidad de una vaga molestia en hipogas-

Constituye la segunda causa de hemorrag ia del tercer trimestre. El desprendimiento puede ser total, parcial o sólo estar afectado el borde pla-

trio y de dolor a la palpación, aunque no siempre ocurre. En algunos casos, la hemorragia externa, norma lmente oscura, es el único signo.

centario (rotu ra o hemorragia del seno margina l).

Abruptio avanzado: se desprende entre 1/4 y 2/3 de la inserción placenta ria. Cursa con dolor uterino continuo, de aparición brusca o gradual, que va seguido de sangrado genital oscu ro. Puede o no haber signos de shock, aun cuando la hemorragia externa sea escasa. El útero presenta hipertonía y es claramente doloroso a la pa lpación. A causa de la contracción uterina mantenida, los tonos card íacos feta les pueden auscultarse con d ificultad. Es posible

El DPPNI se asocia a CID .

Etiología Es poco conocida, au nque se sabe que influyen los sigu ientes aspectos: Multiparidad: ocurre con más frecuenc ia en grandes multíparas que en nul íparas. Edad: es más habitual en mujeres mayores de 35 años. Enfermedad vascular: la

úte ro Acidosis fetal

pree-

clampsia predispone claramente a esta complicación, especialmente en pacientes con enfermedad vasculorrenal subyacente: diabéticas, nefró-

Feto muerto por abruptio. Obsérvese g ra n he matoma placentario

patas, hipertensas, hasta el punto de que la HTA es el factor más claramente relacionado con el DPPNI.

Figura 60. Abruptio placentae

25 · Hemorragias del tercer trimestre

Ginecología y obstetricia

1

presentar coagulopatía y daño renal, aunque es raro.

Útero de Couvelaire.

Abruptio masivo: la sepa ración es superior a 2/3 de la inserción

Embolia de líquido amn iótico: es excepcionalmente rara.

25

placenta ri a. El comienzo suele ser brusco, apareciendo dolor muy intenso y continuo. Se asocia frecuentemente a útero de Couvelaire (infiltraciones hemo-

25.3. Rotura de vasa previa

rrágicas en el miometrio). El feto está casi siempre muerto. El shock se instaura con rapidez y a no ser que la situac ión se controle; son muy frecuentes las complicaciones como oliguria y coagulopatía.

La rotura de vaso previa es la causa menos frecuente de hemorragia del tercer trimestre. Se trata de la inserción del cordón en la bolsa amnióti ca

Diagnóstico

en vez de en la placenta (fenómeno denominado inserción velamento-

Aparte de la clínica, el método diagnóstico de elección es la ecografía, en la que se visualiza el hematoma retroplacentario.

sa), de manera que los vasos umbilicales cruzan por delante de la presentación de forma anómala, para después introducirse en la placenta. En el momento de la amniorrexis, ya sea espontánea o artificial, y coin-

Tratamiento

cidiendo, por tanto, con la expulsión de líqu ido amniótico, se produce

En general, el tratamiento consiste en terminar la gestación lo antes posi-

fetal; la sangre es de origen fetal, por lo que la mortalidad fetal es muy

ble, contemp lando además:

elevada (75%) .

la rotura de los vasos umbilicales previos con hemorragia y sufrimiento

Mantener las constantes vitales maternas.

Sospecha diagnóstica : visualización de vasos que laten en la bolsa

Cruzar y reservar sangre para realizar una posible transfusión.

amniótica.

Hacer un estudio de coagu lación.

Tratamiento: cesárea urgente.

Como regla general, se realizará una cesárea urgente. Si el feto está muerto, se prefiere la vía vaginal, siempre que las condiciones maternas lo per-

25.4. Rotura uterina

mitan, y controlando la posible aparición de comp licaciones.

Complicaciones

En la Tabla 29 se resume el diagnóstico diferencial de las hemorragias del tercer trimestre.

Coagulación intravascular diseminada (1 Oo/o): el DPPNI constitu ye la cau sa más frecuente de trastornos de la coagulación en el embarazo. Fracaso renal agudo (1-3%) . Placenta previa

~

. . ..

'

La causa más frecuente de la rotura uterina es la dehiscencia de una cicatriz de cesárea previa, aunque es posible con cicatrices uterina s de Rotura uterina

Vasa previa

~

Desgarro del canal

Comienzo

Lento

Brusco

Brusco coincide con amniorrexis

Brusco, antes o durante el parto

Tras salida del feto

Sangrado

Rojo, abundante, discontinuo, recid ivante, tendencia a coagular

Escaso, oscuro

Líquid o amn iótico teñido de sangre

Rojo, cuantía variable

Estado general materno

Bueno

Malo

Bueno

Hemorrragia vaginal variable, shock hipovolémico, hemoperitoneo Muy malo (shock) Dolor intenso

Estado fetal

No afectado, riesgo de prematuridad

Afectado, riesgo de anorexia, muerte

Sufrimiento fetal, elevada mortalidad (la sangre es de origen fetal)

Muy afectado, alta mortalidad

Bueno

Dolor

No



No



Va riable

Tono uterino

Normal

Hipertonía, tetania

Normal

Atonía

Normal

Inserción velamentosa del cordón

Cicatriz uterina: la dehiscencia de la cesárea anterior es la causa más frecuente

Parto instrumental

Sospecha: vasos que laten en la bolsa amniótica

Se palpan las partes fetales, cese de la dinámica uterina

Cesárea urgente

Cesárea, urgente+ reparar/ histerectomía

Asociaciones

Diagnóstico

Tratamiento

Embarazo múltiple

Preeclampsia

Cicatriz uterina

HTA

Multiparidad

Polihidramnios

Tabaco

Cortedad de cordón

Edad avanzada

Déficit de ácido fólico

Ecografía transabdominal o transvaginal Central: cesárea Marginal o inserción baja: si > 20 mm entre placenta y OCI parto vaginal

Alcohol, tabaco, multiparidad Clín ico (más importante) Ecografía Cesárea urgente (si feto muerto, vía vaginal)

Tabla 29. Diag nósti co diferencial entre las hemorragias del terce r tri mestre

Bueno

Macrosomía fetal

Manual CTO de Medicina y Cirugía, 2. a edición cualqu ier otra cirugía uterina (miomectom ía, corrección de malformaciones ... ). Se presenta de forma bru sca durante el embara zo y/o parto con la aparición de hemorragia vag inal escasa (la sa ngre habitualmente vierte a la cavidad abdominal) y afectación g rave del estado ge nera l asociado a dolor intenso, cese de la dinámica uterina y tend encia a la atonía uterina.

Ideas clave 2S

Es posible la pa lpación de partes feta les a través de la pared abdominal y la presentación feta l se aleja del estrecho superior. Esta complicación requiere de la rea lización de cesárea urgente, siendo posible, en ocasiones, reparar la rotura uterina. En caso contra ri o, se practi ca rá histerectomía.

"

El abruptio placentae tiene una etiología poco conocida, aunque se han descrito muchos factores favorecedores implicados: multiparidad, edad avanzada, preeclampsia, traumatismos, polihidramnios, cortedad del cordón, déficit de ácido fólico, tabaco, alcohol, cocaína, hipofibrinogenemia congénita .

"

La clínica se caracteriza por la aparición de un sangrado vaginal oscuro, asociado a dolor abdominal difuso brusco, con aumento del tono uterino y mal estado tanto materno como fetal.

"

La placenta previa es la causa más frecuente de hemorragias del tercer trimestre. Se caracteriza por la aparición de un sangrado rojo intermitente e indoloro. No hay afectación fetal ni alteraciones en el tono uterino.

"

Son factores favorecedores el embarazo múltiple: la presencia de cicatrices uterinas, la multiparidad, la edad avanzada y el tabaco.

"

El diagnóstico, además de la clínica, se realiza con la ecografía, que permite ver el tipo de placenta previa y, por tanto, decidir la vía del parto.

"

En los ca sos de abruptio placen toe, hay que realizar una cesárea urgente, salvo en los que se haya producido la muerte fetal intraútero, en los que se dejará un parto vaginal.

"

En los casos de placenta previa central, se debe realizar una cesárea, mientras que cuando la placenta previa la placenta previa esté a > 20 mm del OCI se puede perm itir un parto vaginal con altas posibi li dades de éxito.

"

Son frecuentes la aparición de complicaciones como la insufici encia renal, la coagulación intravascular diseminada, mientras que otras complicaciones, como el útero de Couvelaire y la embolia de líquido amniótico, son más raras.

Casos clínicos . =--. Paciente gestante de 38 semanas en periodo de dilatación (4 cm, primer plano); disfruta de anestesia epidural. Hace un año se le realizó cesárea por situación transversa. Estando previamente bien y de forma súbita presenta un sangrado que coincide en el registro cardiotocográfico fetal con 4 desaceleraciones tipo DIP 1 seguidos de una bradicardia fetal a 70 latidos por minuto. La presentación fetal se palpa sobre el estrecho superior de la pelvis y la dinámica uterina prácticamente ha cesado. Su actitud es: 1) Sospecha de abruptio placentae; realiza cesárea. 2) Sospecha de placenta de inserción baja; esperará a la normalización de la frecuencia cardíaca fetal y permitirá seguir el curso del parto.

Case Study . · A 30-year-old woman, gravida 3, para 2, presented at 32 weeks gestation with a history of dull aching pain in the abdomen irradiating to her back. Fetal movements were normal. The patient had two previous emergency cesarean sections. In the current pregnancy, there were no symptoms of headache, tlickering in front of the eyes or epigastric pain. On examination, maternal blood pressure was 110/ 70 mmHg and normal pulse. Suddenly, she experienced worsening lower abdominal pain and little blee-

25 · Hemorragias del tercer trimestre

3) Sospecha de rotura uterina; realiza cesárea. 4) Sospecha de rotura de vasa previa; realiza cesárea .

RC: 3 Gestante de 38 semanas que ingresa con trabajo de parto. Durante el periodo de dilatación, presenta cuadro de dolor brusco. En la exploración, usted objetiva metrorragia escasa y aumento del tono uterino a la palpación abdominal, que resulta muy doloroso. ¿Cuál sería su diagnóstico? 1) 2) 3) 4)

Rotura uterina. Placenta previa. Desprendimiento de placenta. Crioamnionitis hemorrágica.

RC: 3

ding per vaginum. On examination, her pulse and blood pressure remained normal, but her abdomen felt very tense. The abdomen was very tense and tender on examination; fetal parts were not clearly palpable. The most likely diagnosis is: 1) 2) 3) 4)

Placenta previa. Abruptio placentae. Uterine rupture. Vasa previa.

Correct answer: 2

_Ginecología y obstetricia._--..

Alteraciones de los anejos ovulares

ORIENTACIÓN

ENARM

Tema poco representativo. Es preciso estudiar las Ideas Clave.

Nudos de cordón Nudos falsos: son espiras exageradas de las arterias umbi licales, engrosamientos de la gelatina de Wharton o dilataciones varicosas. Carecen de sign ificación clínica .

26.1. Patología del cordón umbilical

Nudos verdaderos (1-2%): se dan con mayor frecuencia en cordón largo, polihidramnios, fetos pequeños y gemelos monoamnióticos.

La patología del cordón um bi lical eng loba sobre todo quistes y t umores. Los quistes pueden ser verdaderos, de origen embrionario, o falsos, como los derivados de la gelatina de Wharton (la gelatina de Wharton es la matriz extracelular del cordón que rodea a las dos arterias y a la vena

Circulares de cordón So n más frecuentes alrededor del cuel lo. Ocurren en el 15o/o de lo s partos y se asocian con las circunstancias que favorecen la movilidad

umbilica l).

fetal (polihidramnios, fetos pequeños, multiparidad, cordón largo). Pueden diagnostica rse int raútero (ecografía) o sospecha rse por el

Pueden aparecer angiomas, qu ist es dermoides ... pero son excepcio-

RCTG (DIPS variables o t ipo 111). Ocas io nalmente, causan sufr imiento

nales.

fetal.

Alteraciones de posición Anomalías vasculares Vaso accesorio: ocupa un extremo del cordón, desapareciendo en la gelatina . Arteria umbilical única: frecuente en embarazos múltiples (3-So/o) y en diabéticas. Se asocia en un 15-20% de los casos con ma lformaciones (vascu lares, digestivas, genitourina ri as).

Procidencia o laterocidencia: cons iste en un desce nso del cordón a través del estrecho superior, sin sobrepasar la presentación. Prolapso: el cordón sobrepasa la presentación. Está favorecido por adaptación pélvico-fetal defectuosa (pe lvis estrecha, multiparidad, presentació n podálica, situac ió n t ransversa, gemelaridad, po li hid ram ni os) . Requi ere termi nación inm ed iata del parto mediante cesárea, sa lvo si el feto está m uerto, o se trata de una

Anomalías de inserción Inserción velamentosa: el cordón nace de las membranas. Se acompaña frecuentemente de anoma lías de la inserción placentaria .

mu ltípara en expuls ivo y con posib il idad de parto vag inal inmed iato. Procúbito: es el prolapso o la procidencia del cordón con bolsa íntegra.

Vasa previa (véase el Capítulo de Hemorragias del tercer trimestre).

Anomalías de longitud

26.2. Alteraciones de la placenta

La long itud norma l es entre 4S y 60 cm: Cordón corto: menor de 30 cm (1 ,So/o). Puede ca usar, aunque es poco frecuente, presen taciones anóma las, abruptio, dificultad en el

La inserción norma l de la p lacent a es sobre el endom etrio, que durante la gestación se mod ifica y se llama, como se ha visto, decidua.

descenso feta l o sufrimiento feta l. Cordón largo: mayor de 65 cm. Puede favorecer circulares, nudos y prolapso.

La patología que puede aparecer a nivel de la placenta es la siguiente (Figura 61 ):

Manual CTO de Medicina y Cirugía, 2. 8 edición

Placenta accreta (total o parcial): la placenta no se inserta sobre el endometrio, sino sobre el miometrio, por insuficiente decidua lización del pri mero. La incidencia es muy baja. Pred isponen para esta

26.3. Patología del líquido amniótico

alteración : la mu lt iparidad, la placenta previa, los legrados, la cirugía previa, los miomas, etc. Clínicamente, hay una fa lta de desprend i-

Hidramnios o polihidramnios

miento placentario tras el parto y hemorragia du rante el alumbramiento. El tratam iento consiste en rea liza r masaje uterino, intentar la extracción manua l, practicar leg rado y, si no se consigue el despren-

Se llama así al exceso de líqu ido amn iótico (> 2.000 mi). No obstante, aunq ue ese es el punto de corte más comúnmente aceptado, puede no ser clínicamente significativo hasta llegar a va lores comprendidos entre

dimiento, realizar histerectomía. Placenta increta: la placenta no sólo se inserta sobre el miometrio,

3.000 y 4.000 mi. La etiología no se conoce con exactitud. Las fuentes de líquido amn iótico son el propio amnios, la exudación desde el plasma

sino que penetra en él. Placenta percreta: la placenta atraviesa el endometrio, el miome-

materno, y la orina fetal. La eliminación se consigue por la absorción a t ravés de las membranas feta les y la deglución feta l.

trio, y alcanza la serosa peritonea l, pudiendo afectar a órganos vecinos. Placenta succentu riata o accesoria: existencia de lóbu los placentarios adiciona les, a distancia de la placenta pri ncipa l, con la cua l mantienen conexiones vascu lares. Pueden retenerse esos lóbu los tras el alumbramiento. Se sospecha al ver vasos desgarrados en la

Etiología Entre las mú lt iples patologías asociadas, se destacan las sigu ientes: las anoma lías en la deg lución: atresia esofág ica, atresia duodena l, enfermedad neuromuscular...

bolsa amniótica. El tratamiento consiste en la extracción placentaria

El exceso de orina fetal por déficit de hormona antidiurética, que

manual. Placenta membranácea: se desa rro lla la placenta alrededor de todo el huevo, siendo esta delgada y membranosa. Orig ina aborto,

puede aparecer a su vez en casos de anencefa lia o encefa locele. La diabetes materna: posiblemente por hiperg lucemia feta l con poli uria feta l.

metrorragia y simula placenta previa. Placentas extracoriales: la placenta no está rodeada completamente por las membranas, sino que hay parte de tejido placentario que no está recubierto. Los vasos placentarios no llegan hasta el borde.

Diagnóstico útero mayo r que ame norrea. El feto es d ifícil de pa lpar. La ecografía nos lo confirma y puede detectar ma lformaciones fetales asociadas al cuadro.

Conducta Hay que descartar alteración fetal. Los diuréticos no son eficaces y están contra ind icados en el embarazo. Para aliviar la disnea o el dolor materno, se puede rea lizar amniocentesis evacuadora (500-750 mi) que, efectuada de forma peri ód ica, puede evita r el desencadenam iento de un parto prematuro. Si la evacuación es rápida, es posible que se produzca un abrup-

tio placen toe.

Oligoamnios Cantidad de líquido amniótico inferior a 500 mi. Es necesario considerar la edad gestacional, ya que en las últimas semanas la cantidad de líquido amniótico se reduce considerablemente de fo rma fis iológica.

Etiología Malformaciones renales: agenesia, displasia o atresia del riñón, uréter, vej iga o uretra (por escasa producción de orina). Ret raso de crecimiento intrauterino (por oliguria al centralizar el flujo). Rotura prematura de membranas (espontánea o postamniocentesis). Posmadurez. Pulmonares (h ipoplasia). Secuencia de Potter: oligoamnios, facies arrugada, extremidades comprimidas en flex ión, hipoplasia pu lmonar. Provocada por cua lquier causa de oligoamn ios grave en etapas precoces de la gestación (Figura 62). Figura 61 . Tipos de inserción placentaria

26 · Alteraciones de los anejos ovulares

Síndrome de Pott er: es la suma de agenesia renal más la secuencia Potter.

Ginecología y obstetricia

1

26

Diagnóstico El diagnóstico se sospecha por la exploración y es ecográfico, siendo difíc il la va loración, ya que la escasez de líqu ido dificulta la ecografía. Se puede practica r am nioinfusión, que es la infusión de líq uido por amniocentesis como contraste para mejorar el estudio; esta técnica ta mbién puede usa rse con intención terapéutica, introduciendo 500-600 mi de suero fis iológ ico para intentar au menta r el volumen de líquido amniótico, aunque su util idad está aú n en estud io.

Pronóstico

Figura 62. Secuencia de Potter

Se acompaña de un incremento en la patología perinata l: CIR, malformaciones fetales-renales, compresión fu nicular, hipoplasia pulmonar, etcétera. El pronóstico fetal es malo, por lo que es peor en casos de oligoam nios precoz.

miometrio, de ahí que no se produzca el alumbramiento normal de la placenta. Está favorecido por la multiparidad, la placenta previa, los antecedentes de legrados o las cirugías previas y los miomas uterinos. Puede provocar hemorragia una vez conseguido el alumbramiento, por lo que se debe tratar con masaje uterino y legrado. En raros casos, hay que recurrir a la histerectom ía .

Ideas clave Jf!S "

La presencia de arteria umbilical única se asocia en un 15-20% con malformaciones, por lo que se considera un marcador eco gráfico de cromosomopatías.

"

La inserción velamentosa del cordón predispone a la aparici ón de hemorragia cuando se produzca la amniorrexis.

"

La laterocidencia cons iste en un descenso del cordón umbilical a través del estrecho superior sin sobrepasar la present ación, mientras que en el prolapso de cordón sí la sobrepasa. Está favorecido por la multiparidad, la presentación pelvia na, la situación t ransversa, el embarazo gemelar y el polihidramnios.

"

El acretismo placentario se caracteriza porque existe una insuficiente decidualización, por lo que la placenta se inserta sobre el

Case Study A 23-year-old woman, gravida 3, para 2, with hypertension was admitted at 36 weeks gestation with a four-day history of spott y vaginal bleeding. The patient hada history of two prior cesarean deliveries. Ultrasound demonstrated full placenta previa. Magnetic resonance imaging revealed complete placenta previa with the placenta severely thinning the myometrium, extending both anteriorly and inferiorly in the region of the bladder. Large ves-

"

El polihidramnios suele asociarse a situaciones en las que existe una producción excesiva de líquido amniótico (déficit de hormona antidiurética, como en anencefalia o encefalocele o poliuria fetal por hiperglucemia materna) o una reabso rción deficiente (anomalías en la deglución: atresia esofágica, atresia duodenal, hernia diafragmática ... ).

"

El oligoamnios se asocia a malformaciones renales (agenesia, atresia o displasia), retraso del crecimiento intrauterino, rotu ra prematura de membranas y posmadurez.

seis were also seen supplying the placenta in the region of the cervi x. The most likely diagnosis is: 1) 2) 3) 4)

Placenta previa. Placenta accreta. Placenta increta. Placenta percreta .

Correct answer: 2

1

Ginecología y obstetricia

Gestación múltiple

ORIENTACIÓN

ENARM

Es un tema poco importante. Hay que estudiar, sobre todo, el apartado de la vía del parto en el embarazo gemelar.

Dicigót icos o bivitelinos: son gemelos que proceden de la fecundación de dos óvulos distintos por dos espermatozoides. Tienen diferente genotipo y pueden ser del mismo o de distinto sexo. Se parecen entre sí como dos hermanos cualesquiera. Los gemelos dicigóticos son siempre bicoriales-biamnióticos.

27 .1. Clasificación La más frecuente es la monocorial -biamniótica. La gestación mú ltiple se clasifica en los sigu ientes ti pos (Figura 63): Monocigóticos o univitelinos: proceden de un mismo óvulo que se divide tras ser fecundado por un sólo espermatozoide; por tanto, tienen idéntico genotipo y sexo. Seg ún el momento en el que se

27 .2. Incidencia

produzca esta división, se pueden distingui r: Bicorial-biamniótica: con dos place ntas y dos sacos am nióticos. Esto ocu rre cuando la división t iene lugar en los primeros

En general, se hab la de una frecuencia de uno por cada 90 embarazos,

tres días tras la fecundac ión. La frecuencia es del 30%. Monocorial-biamniótica: es el más habitual (65o/o). Una sola

siendo más frecuente las gestaciones dicigóticas que las monocigóticas.

placenta y dos bolsas amn ióticas. Este tipo aparece cuando la división ocurre entre cuatro y ocho días tras la fecundación.

27 .3. Factores etiológicos

Si la división tiene lugar pasados ocho días tras la fecundac ión, el tipo de placentación será monocorial-monoamniótica. En casos muy ra ros, la división se produce pasados 13 días o más. A consecuencia de esto, aparecen los gemelos siameses.

~ Embrió n

La frecuenc ia de aparición aumenta con la edad materna, la paridad, el abandono reciente de la contracepción oral o los tratam ientos inductores de la ovu lación en parejas estériles (Figura 64).

---------------.

t

o t

o t

~ Bicorial biamniótica



Monocorial b iamn iót ica

Figura 63. Clasificación de la gestación gemelar

Monocorial monoamn iót ica

Figura 64. Ecografía 3D de embarazo gemelar

Ginecología y obstetricia

1

21

27 .4. Patología asociada a la gestación gemelar Una gestación gemela r es un embarazo de alto riesgo, con g ran probabi lidad de comp licaciones que alt eren el curso de la gestación y con aumento de la morbilidad, tanto materna como fetal:

Crece el riesgo de aborto, sobre todo, en gemelos monocigotos. En ocasiones, una de las gestaciones se interrumpe, mientras la otra prosigue y llega incluso a térm ino. Cuando la interrupción sucede de forma precoz, el embrión puede reabsorberse. Si la interrupción estardía, el feto sufre un proceso de momificación y se denomina feto papiráceo. Esto puede resu ltar pelig roso tanto para la madre como para el feto superviviente, debido a la liberación de tromboplastinas feta les

y placentarias, que es posible que desencadenen un cuad ro de CID. Complicaciones asociadas al estado de hiperplacentosis y al aumento en la secreción hormonal: es má s frecuente la apa rición

Figura 65. Circunferencias abdominales en un sínd rome de tran sfu sión feto-fetal en gemelos con retardo de crecimiento del feto transfusor

de hiperemesis g ravídica (q ue depende.fundamentalmente de la secreción de HCG du rante el primer trimestre) y de preeclampsia, que además debuta de forma más precoz (antes de las 20 semanas).

27 .S. Diagnóstico

También es habitual la aparición de hipertensión inducida por el emba razo q ue, a diferencia de la preeclampsia, cursa sin proteinuria. El origen más probable de esta afecció n es la excesiva expansión del volumen intravascular.

Amenaza de parto pretérmino: la ca usa principal de morbimortalidad neonatal asociada a la gestación gemelar es la prematuridad. El trabajo de parto pretérmino se presenta con mucha frecuenc ia y se debe a: Sobredistensión uterina que aumenta la irritabil idad de las fibras miometriales y desencadena las contracciones. Polihidramnios, que aparece hasta en un 15% de los casos, siendo más frecuente en gemelos monocigotos, y que producirá

Anamnesis: los antecedentes fam iliares o personales de gestación gemela r, así como el uso de inductores de la ovulación. Signos clín icos: útero de mayor tamaño que el que corresponde al tiempo de amenorrea.

Pruebas com plementarias: la ecografía, practicada durante el primer trimestre de emba razo se emplea para confirmar dicha gestación múltiple, ver el tipo de placentación y el número de bolsas, ya que el pronóstico varía considerablemente, según se trate de una gestación mono o biamniótica, y según el número de embriones: doble, triple, etcétera .

también la irritación por distensión de las fibras miometriales antes comentada.

27 .6. Conducta obstétrica Rotura prematura de membranas: es más frecuente que ocu rra, y a la vez, influye en la mayor frecuencia de partos pretérmino. Crecimiento discordante de ambos gemelos: afecta al 30% de las gestaciones gemelares, siendo más habitua l entre gemelos monocigotos. Una de las causas principales es el sínd rome de transfusión gemelo a gemelo. Se da casi exclusivamente en gemelos monocoria les, y es necesario que se desa rrollen anastomosis vasculares entre ambos fetos, fundamentalmente arteriovenosas, lo que da lugar a un feto transfusor y a un feto transfundido. El feto transfundido puede llegar a un estado de policitemia ta l que desencadene una insuficiencia ca rdía ca de alto gasto, y que desarrolle una diferencia de peso a su favor con el otro gemelo de más del 20%, así como polihidramnios. A su vez, el feto t ran sfusor desarrollará anemia, CIR y o ligoamn ios. Cuando el sistema circulatorio del feto transfundido adquiere predominio sobre el del feto transfusor, se puede llegar a la situación de que este último se convierta en un feto aca rdio. Puede tratarse con coag ulación por láser de las anastomosis vascu lares (Figura 65). Las malformaciones congén it as presentan t res veces mayor incidencia en gestaciones gemela re s, principa lmente las cardiopatías congénitas y los defectos del tubo neural. Con frecuenc ia, aparece patología de cordón, sobre todo prolapso de cordón, en casos de gestaciones monoamnióticas o con pol ihidramnios. El abruptio placen toe es un riesgo frecuente durante el parto, debido a la rápida descompresió n brusca del útero tra s el nacimiento del primer gemelo o tras la evacuación de un polihidramnios coexistente.

Gemelos monoamniót icos: debido al considera b le número de complicaciones que pueden apa recer (prolapso de cordón al romper la bolsa, entrecruzam iento de cordones colisión de ambos gemelos en el cana l del parto), se recomienda cesárea electiva a las 32 semanas de gestac ión tras un ciclo de corticoides para inducir la madurez pulmonar, independientemente de la estática de los gemelos. En el caso de gemelos biamnióticos, la vía del parto está con d icionada principalmente a la estática de los fetos y el peso estimado: Por debajo de un peso estimado menor de 1.500 g, sólo se aceptará el parto por vía vagina l si ambos gemelos están en situación longitudinal y presentación cefá lica. Si uno o ambos fetos adoptan la presentación podál ica, se realizará una cesárea electiva . Por encima de un peso estimado mayor de 1.500 g, se permi ti rá el pa rto por vía vagina l siem pre que el primer gemelo se encuentre en presentación cefá lica: 1o cefálica-2. 0 cefá lica : es el tipo más frecuente: parto vaginal. 1o cefá lica-2. 0 podálica : parto vagina l. Tras la expu lsión del primer gemelo, se intentará el parto en podálica del 2.0 (de elección).

Manual CTO de Medicina y Cirugía, 2.8 edición

1.0 cefálica-2° transversa: parto vaginal. El segundo gemelo puede rotar espontáneamente a long itud inal. Si no lo hace, se realiza una versión interna y gran extracción del 2° gemelo (de elección).

drome de transfusión feto-fetal. Está provocado por la p resencia de anastomosis vasculares entre ambos fetos, en los casos en los que existe una placenta monocorial. Da lugar a un feto transfusor (anemia, CIR y ol igoamn ios) y a un feto transfundido (policitemia, insuficiencia cardíaca de alto gasto, po lih idramnios).

Ideas clave RS "

"

El embarazo múltiple es un embarazo de alto riesgo por el riesgo de aparición de mayor número de complicaciones. Es posible encontrar mayor frecuencia de abortos, malformaciones, hiperemesis gravídica, hipertensión arterial, rotura prematura de membranas, amenaza de parto pretérmino, patología del cordón y abruptio placentae.

"

Una complicación específica es el crecimiento discordante de ambos gemelos, producido en muchos casos por el sín-

Casos clínicos . Acude a su centro una mujer primigesta de 35 años por sensación de dinámica regular. Se trata de una gestación gemelar monocorial, biamniótica en la semana 36 + 4, cuyo curso ha sido normal; la exploración revela un cérvix borrado, con 5 cm de dilatación, blando y centrado; primer gemelo en presentación cefálica, segundo en transversa, bolsa íntegra y amnioscopia negativa. ¿Cuál es la actitud correcta de cara al parto?

Case Study A 30-year-old woman, gravida 1, para O, was referred to our hospital at 31 weeks and six days gestation due to the presence of a discordant fetal growth pattern and polyhydramnios in a twin pregnancy. lndicate the incorrect statement about the disease you believe the patient has:

27 · Gestación múltiple

La vía del parto en las gestaciones monoamnióticas será siempre la cesárea. Sólo se admitirá el parto vaginal en las gestaciones biamnióticas en las que el primer gemelo esté en cefál ica, si son fetos de más de 1.500 g, m ientras que si son menos de 1.500 g, sólo se hará si ambos fetos están en cefálica.

1) Realizar una cesárea urgente, ya que se trata de una gestación biamniótica. 2) Dejar evolucionar el parto por vía vaginal. 3) Dejar evolucionar el parto vaginal del primer gemelo, y realizar una cesárea no electiva del segundo. 4) Iniciar la administración de tocolíticos, puesto que se trata de una gestación de 36 semanas, y es previsible que los fetos sean muy inmaduros. RC: 2

1) 2) 3) 4)

This syndrome appears in monochorionic twin pregnancy. The recipient infant has anemia. The recipient infant has polyhydramnios. Fetoscopic laser photocoagulation is the twin-to-twin transfusion syndrome treatment.

Correct answer: 2

_Ginecoi_Dijía y obstetricia



Parto pretérmino

ORIENTACIÓN

ENARM

Atender, sobre todo, al apartado de manejo de la amenaza de parto pretérmino.

Factores neuroendocrinos

Oxitocina-vasopresi na

Aunque se desconocen los mecanismos exactos que desencadenan el

Factores miometriales

parto, se sabe que ti enen gran infl uencia la oxitocina, las hormonas su-

· Distensión · Receptores oxitocina ·GAP junctions · Prostagland inas

prarrena les materno-fetales y el tono simpático. Por el contrario, la adecuada hidrat ación, el reposo y la progesterona actúan como uteroinhibidores. Se desconoce qué factor rompe defin itiva-

Factores fetales

mente el equilibrio, aunque el reAej o de Fergu son (aumento de la contractilidad tras el estímulo ce rvical) puede tener alg ún papel. Se entiende por parto prematu ro aquel que se produce antes de las

Presión/ manipulación

~

37 semanas cump lidas de gestación y por amenaza de parto prematuro (APP), junto con la aparición de dinám ica uterina reg ular antes de las 37 semanas de gestación, acom pañada de m od ificaciones cervica les. La tasa de parto pretérmino actualmente oscila entre 8- 1Oo/o de todas las

· Estrógenos/progestágenos · Receptores de oxitocina en amnios y decidua · Prostaglandinas · Oxitocina y vasopresina

Factores cervicales · Estrógenos/progestágenos _ · Prostaglandinas ' ·Relaxina

gestaciones, correspo ndiendo la mayoría a una prematuridad leve (32-36

· Colagenasas · Ácido hialurónico, dermatán-sulfato y ~ condroitín-sulfato ·Agua L___ _ _ __ _ j

semanas de gestación).

Figura 66. Fisiología de la contracción uterina

28.1. Etiología

28.2. Diagnóstico

Se puede clasifica r los partos pretérm ino segú n su orig en en los sig uientes ti pos:

Pretérmino espontáneo o idiopático (SO%): se trata de un cuadro de etiología compleja, con múltiples factores que interactúan entre sí di-

Es importante identificar precozmente a las pacientes con una APP. Se basará el d iagnóstico en los aspectos que se citan a co ntinuación: Los antecedentes persona les de parto pretérm ino.

ficultando su identificación y su prevención. Son factores de riesgo los

La pérdida prematura del ta pón mucoso (m oco cervical), con pérdida

siguientes: edades extremas, bajo nive l socioeconómico, consumo de

de las funciones antimicrobianas y anti proteolíticas que este tiene.

tabaco, alcohol o cocaína, déficit nutricional, infección urinaria, cervica l

Las contracciones uterinas: se conside ran necesarias cuatro en 20-30

o del líquido amniótico, gestación múltiple, polihid ramn ios, miomas, etc.

m inutos o bien seis en 60 minutos, dolorosas, palpables y de al me-

Pretérmino asociado a rotura prematura de membranas (RPM)

nos 30 segundos de d uración. Es importa nte d isting ui rlas de las con -

(25%).

tracciones fi sio lóg icas (de Braxton-Hi cks) que aparecen en el tercer

Pretérmino por intervención médica o iatrogénico (25%): debi-

trimestre de la gestación.

do a patolog ía materna o fetal (preeclampsia, retraso del crecimiento

Estas suelen ser esporádi cas, irregulares, menos de tres por hora y

intrauterin o, sufrimiento feta l, enferm edades maternas) que aconse-

de aparición genera lmente durante la tarde o primeras horas de la

jan la finalización del embarazo antes de su término.

noche.

Manual CTO de Medicina y Cirugía, 2.a edición

Las modificaciones cervicales: el método más utilizado para la valo-

Dinámica y cérvix sin modificar: en estos casos es razonable man-

rac ión del cuello del útero sigue siendo el tacto vagina l (test Bishop).

tener a la paciente en observación, en reposo una o dos horas. Pasado ese t iempo, se reeval uará a la gest an te y la persistencia o no

Sin embargo este método es subjetivo y presenta variaciones importantes interobservador y suele infraestima r la lo ngit ud rea l del cérvix. Por ello, se ut il iza rá la ecog rafía t ransvag inal y la fi bronectina feta l como técn icas de apoyo a la va loración clínica.

Valoración ecográfica del cérvix

de la d iná mica. Es en estas pacientes donde la determinación de la fibronect ina puede ayudar a tomar la actitud más adecuada.

Dinámica y cérvix modificado: ingreso hospita lario, hidratación, reposo absoluto, instaurar tocólisis, descartar coriomanionitis y corticoterapia para maduración pulmonar feta l.

La medición de la longitud cervical mediante ecografía transvaginal es una técnica fáci l, no invasiva, reproducib le y de bajo coste. Un cérvix con una long itud superior a 30 m m posee alto va lor pred ictivo negativo, lo q ue ayuda a evitar m uchos trata m ie ntos q ue pod rían ser innecesarios dado q ue estas pacientes t ienen un ri esgo bajo de desa rro llar un parto pretérmino.

WJP• 24-31+6 semanas

t

t

La presencia del fenómeno del embudo o ins inuación de las membranas en el canal cervical se asocia a elevada probabilidad de parto pretérmino.

32-34+6 semanas

t

t

35-36+6 semanas

t

t Tocólisis corticoides

Tocólisis corticoides

Marcadores bioquímicos La fibro nectin a es una g licoproteín a fo rmada por las membranas feta les que se det ecta de manera fi siológ ica en cérvix y vag ina hasta la sema na 20 (probablemente sirve como anclaje de la interfase pla-

Falso trabajo prematuro de parto Observación/fibronectina

Figura 67. Tratamiento pa ra la amenaza de parto pretérmino (APP)

centa y corioamnios con la decidua). La principal ut ilidad de la determinación de fib ronectina feta l en el diagnóstico de APP es intentar identificar a aquellas pacientes con resultado negativo. Dado el alto va lor predictivo negativo del test, estas gestantes con fibronectina negativa t ienen un reducido riesgo de tener parto pretérmino (inferior al 1% en las dos semanas siguientes a la realizac ión del test). La interleucina 6 en moco cervica l aparece sólo en un tercio de las amenazas de parto pretérm ino. IGFBP-1 es una prot eína secret ada por las células decidua les maternas. Su detección en las secreciones vag ina les de mujeres con sintomatología de APP a partir de las 20 semanas de gestación se ha asociado a un incremento del riesgo de parto prematuro.

28.3. Conducta obstétrica El t ratam iento de la amenaza de parto prematuro se basa en la eli minación de las contracciones (tocólisis) y en la ace leración de la madurez pulmonar fet al media nte corticoterapia. El tratamiento tocolítico estará indicado en pacientes con APP a partir de las 24 semanas hasta las 34 semanas. A partir de esta edad gestaciona l, dado que el feto será pu lmonarmente maduro, no se instaurará tratamiento tocolítico y se dejará evolucionar el parto. Si se produce rotura prematura de membranas se añadirá tratamiento antibiótico. Ante una gestante q ue refiera d inám ica uterina subjetiva, se debe rea lizar una anamnesis d irigida y una exploración g inecológ ica y un registro card iotocográfico externo. En función de los ha llazgos se pueden tener va rias situaciones:

Dinámica inexistent e y cérvix sin modificar: la paciente podrá ser dada de alta ind icando reposo domiciliario sin necesidad de ninguna otra actuación terapéutica .

28 · Parto pretérmino

28.4. Tocólisis La tocólisis constituye el conjunto de métodos empleados para frenar o pa ra hacer desaparecer la dinám ica uterina. So n los siguientes: Hidrat ación, sedación y reposo absoluto hospit ala rio, en cama, en posició n decúbito latera l izqu ierdo. No hay evidencia científica sobre el nivel de eficacia de estas medidas.

Antagonistas de la oxitocina: atosiban. Actúa como inhibidor competitivo de la oxitocina, inhibiendo su acción contractora uterina. Hoy en día es considerado el fármaco de elección. No se han descrito contra indicaciones salvo una posible alergia al fármaco. Sus efectos secundarios más frecuentes son náuseas, vóm itos y cefalea. ~-miméticos (terbutalina): actúa estimu lando los receptores ~ 2 presentes en el múscu lo uterino, pero también produce estimu lación ~ - 1 por lo que sus efectos secundarios más frecuentes son taquicardia, hi potensión, temblo r y descom pensación g lucém ica. Está contraind icado en caso de placenta previa con hemorragia grave y en el abruptio. También lo está en caso de mujeres d iabéticas, cardiópatas, hipertensas graves e hipertiroideas. Es necesa rio tener especial precaución con el uso junto a corticoides, ya que la taquicardia baja el gasto cardíaco y favorece el edema de pulmón toda vez que el corticoide aumenta la permeabilidad vascular y también favorecería el edema de pul món. Antagonistas del calcio (nifedip ino): ej ercen su acción tocolítica al inh ibir la entrada de ca lcio extracelular y difi cultar así la contracción. Los efectos secundarios más frecuent es son mareos, cefalea, hipotensión y rubor facial. Está contraind icado en casos de insuficiencia renal o cardíaca y no se recomienda en pacientes digita lizada s. Ant iprostaglandínicos (indometacina): inhiben a la prostaglandinsintetasa por lo que consiguen reducir la dinámica uterina. Entre sus efectos secundarios se encuentran vómitos, hemorragia digestiva y

Ginecología y obstetricia hemorragia posparto. Puede inducir un cierre prematuro del ductus arterioso por lo que no se recomienda usarlos a partir de las 32 semanas. Están también contraind icados en gestantes con rotura prematura de membranas.

La ecografía resulta de utilidad en la predicción del riesgo de parto pretérmino. Sobre todo sirve para evaluar la longitud cervical. Una longitud inferior a 3 cm supone un claro riesgo de parto pretérmino. Otros parámetros a valorar son la dilatación del orificio cervical interno y la protrusión de las membranas a través del OCI (imagen en embudo).

"

La fibronectina es una proteína que se produce en las membranas de manera fisiológica hasta la semana 20. Cuando se detecta más allá de la citada semana, implica riesgo de parto pretérmino.

"

La actitud obstétrica ante la amenaza de parto pretérmino va a depender de la edad gestacional, de la modificación cervical y de si existe o no rotura de membranas. No se utilizarán los fármacos tocolíticos en gestaciones de más de 34 semanas, ya que

Casos clínicos _, · Una gestante de 34 semanas, por lo demás normal, ingresa por rotura de membranas desde 8 horas antes y amenaza de parto pretérmino. No tiene fiebre ni signos de infección. El cuello uterino no está dilatado. Se objetivan contracciones uterinas. El feto está vivo, no tiene malformaciones por ecografía y el registro de la frecuencia cardíaca fetal es normal. ¿Qué actitud tomaría? 1) Tratamiento de uteroinhibición y antibióticos. Administración a la madre de glucocorticoides para inducir la maduración pulmonar fetal y de antibióticos para prevenir la infección. 2) Tratamiento de uteroinhibición. 3) Tratamiento de uteroinhibición sólo. 4) Dejar evolucionar el parto.

RC: 4 Gestante secundigesta, con antecedente de un parto anterior en la semana 36, que consulta en la semana 32 por percibir contracciones. En la exploración se comprueba un cérvix permeable al dedo, borrado en 30% y con 3 cm de longitud ecográfica. La monitorización cardiotocográfica fetal revela una frecuencia cardiaca de 140 lpm y una contracción uterina cada S minutos. El test de fibronectina exocervical es negativo. ¿Cuál de las siguientes afirmaciones es INCORRECTA?

la única ventaja de este tratamiento es demorar el parto hasta alcanzar la madurez pulmonar que se induce con la administración de corticoides. "

El atos iban es un antagonista de la oxitocina con escasos efectos secundarios: náuseas, vómitos y cefa lea.

"

Los ~-miméticos son fármacos de demostrada eficacia, pero con un elevado número de efectos secundarios y, por tanto, de contraindicaciones (cardiópatas, diabéticas, placenta previa, hipertensión, hipertiro idismo ... ). Un efecto secundario peligroso de su utilización, sobre todo al asociarlo a los corticoides, es el edema pulmonar por el aumento de la permeabilidad vascular.

"

Los antiprostaglandínicos (indometacina) también se pueden utilizar como fármacos toco líticos, pero con precaución por los efectos secudarios fetales que pueden presentar: cierre precoz del ductus arterioso y oligoamnios.

2) El test de fibronectina negativo se asocia a un riesgo de parto pretérmino inferior al 1% en las dos semanas siguientes. 3) Una longitud cervical de 3 cm supone un elevado valor predictivo negativo para el parto pretérmino. 4) Se trata de una amenaza de parto pretérmino incierta que requiere nueva evaluación clínica a los 60-120 minutos.

RC: 1 Paciente de 34 semanas de gestación, que acude a urgencias por presentar contracciones uterinas con la sospecha de amenaza de parto pretérmino. Se realiza registro cardiotocográfico fetal, comprobándose la existencia de dos contracciones en 20 minutos y una longitud cervical medida por ecografía de 3 cm. ¿Cuál sería la actitud correcta? 1) Ingresar a la paciente y dejar evolucionar, pues el feto ya está maduro bajo el punto de vista pulmonar. 2) Dejar que se desplace nuevamente a su domicilio advirtiéndole que realice reposo y que acuda de nuevo a urgencias si la dinámica se hace más frecuente. 3) Ingresar a la paciente y administrarle tocolíticos para frenar la dinámica. 4) Ingreso en la sala de expectantes repitiendo, si sigue la dinámica, la ecografía vaginal para ver si existe modificación del cérvix.

RC: 2 1) Se trata de una amenaza de parto pretérmino establecida que

obliga a realizar tocólisis y maduración pulmonar fetal con ~-metasona .

Case Study A 27 year-old woman, gravida 1, para O, was admitted at 32 weeks of gestation with a two-day history of uterine contractions and preterm rupture of membranes. Fetal movements were normal. The initial fetal heart rate was 130-150 beats/min and showed good reactivity and variability. The most likely treatment is:

28

La maduración pulmonar se hace con ~-metasona en- ---~..- .. · tre la semana 24 y 34 de gestación.

Ideas clave 16 "

1

1) 2) 3) 4)

Tocolytics. Tocolytics and corticosteroids. Tocolytics and antibiotics. Tocolytics, corticosteroids and antibiotics.

Correct answer: 4



Ginecología y obstetricia

Gestació n cronológ icamente prolonga da

ORIENTACIÓN

ENARM

Es un tema poco importante. Se deben estudiar las Ideas Clave.

como la fecha de la últ ima reg la, la fecha del positivo en el test gestacional, y la biometría fetal; pero, sin duda, el parámetro más preciso para datar la gestación es la med ición del CRL por ecografía vaginal en el primer trimestre.

Se habla de gestación prolongada cuando el embarazo dura más de 42 semanas. Su incidencia se sitúa alrededor del 1Oo/o de los casos.

29.1. Etiología

29.4. Valoración y tratamiento Las tendencias más conservadoras realizan un control ambu latorio cada 48 horas de:

RCTG no estresante. En la mayoría de las ocasiones, la ca usa de la gestación prolongada es desconocida. Puede que en muchos casos no se trate rea lmente de un embarazo prolongado, sino de un error en la estimación de la edad gestacional. Parece que hay una cierta tendencia genética en algunas fami lias a tener embarazos prolongados. También se asocia a fallo en los factores hor-

Ecografía, pa ra ver la cantidad de líqu ido amn iótico y el grado de madurez placentaria. Amnioscopia: que es negativa cuando el líquido es t ransparente y está en cantidad norma l. Valoración del índice de Bishop: a mayor puntuación, más inducible es el parto (véase Apartado 28.5./nducción).

mona les que desencadenan el parto. En la anencefa lia sin polihidramnios asociado ocurre, probablemente, en relación con el papel del eje

Se procederá a la inducción del parto cuando alguna de las pruebas es patológica o el test de Bishop es mayor de S puntos. Hay controversia

hipotá lamo-h ipofisario y suprarrena l en el desencadenam iento del parto.

sobre la actuación en caso de normalidad en las pruebas de control del bienestar feta l. Pa ra algunos autores es recomendab le finaliza r toda ges-

29.2. Clínica Embarazo fisiológicamente prolongado: el feto es norma l, pero de mayor tamaño. No hay anoma lías ni sufrim iento fetal. Puede dar problemas sólo por la distocia que supone el gran tamaño. Embarazo patológicamente prolongado: el feto deja de crecer, aparecen signos de insuficiencia placentaria, afectación fetal (h ipoxia, hipog lucem ia, maceración de la piel), envejecim iento de la placenta y oligoamnios. Signos inespecíficos habituales en el recién nacido postérmino son: uñas largas, abundante pelo en la cabeza y disminución del vérmix.

tación a pa rt ir de la 4 1.a semana, ya que supone una d isminución en la mortalidad perinatal sin aumentar el riesgo de cesárea .

29.5. Inducción La inducción consiste en provocar el parto. Según sea electiva o no, se habla de:

Inducción electiva: se provoca el parto no por indicación médica sino por otros motivos persona les, sociales: dom icilio alejado, carencia de medios de transporte, etcétera. Inducción terapéutica o no elect iva: se realiza por interés materno o fetal (preeclampsia, diabetes, CIR, rotura de membranas, corioamnionitis, muerte feta l, embarazo prolongado, etcétera).

29.3. Diagnóstico

Contraindicaciones

Al ser el diagnóstico exclusivamente cronológico, se debe establecer con precisión la edad gestacional, para lo cual se tend rán en cuenta datos

La inducción está contraindicada en aquellos casos en los que el parto vaginal sea más peligroso para el feto o para la madre que la cesárea como en el caso de que exista una estática feta l anómala (situación

Ginecología y obstetricia transversa). Está también contra ind icada, aun qu e co n menos consenso, en situaciones como cicatr iz uterina (por ejemplo, por cesárea anterior), embarazo múltiple, sospecha de desproporción cefa lopélvica, sufrim iento fetal (la inducción es un proceso lento

Causa materna Preeclampsia V1

Diabetes gestacional

U!

e

o



Insuficiencia card íaca

u

Esteatosis hepática aguda gravídica

ro

'6 e

fetal es una ext racc ión inmed iat a)

Eclampsia V1

w e

o



ro u

Pronóstico

'6 e

La evolución de una inducción depende de las cond iciones del canal del

Causa obstétrica Gestación prolongada con cérvix maduro, amnioscopia positiva, oligoamnios

Causa obstétrica

Amenaza de parto pretérmino

Rotura de membranas Amnionitis

Desproporción pélvico-cefálica

Diabetes

Amn ionitis

Cardiopatía

Presentación anóma la

HTAgrave

Placenta previa con hemorragia

Cesárea previa

Hipertiroidismo (contraindican el uso de -2 agonistas, pero no otros métodos)

Embarazo múltiple

~

Sufrimiento fetal grave

e

o u

parto. Estos parámetros son valo rados por el índice de Bishop (Tabla 31) que

Causa materna

Insuficiencia placentaria

y lo que interesa ante un sufrimiento

(Tabla 30) .

29

Tocólisis

Inducción



1

Abruptio placentae Muerte fetal

Tabla 30. 1nducción y tocólisis

es un sistema d e puntuación q ue barema el estado del cérvix y la al tu ra de la presentación feta l. Va lora estos

Métodos de inducción

cinco parámetros:

Posición cervical: posterior, mediana o anterior: un cue llo anterior

Los métodos de inducción son los siguientes:

Métodos físicos:

está más dispuesto al parto.

Amniorrexis (rotura de la bolsa amn iótica): se emplea en cuellos muy favorables, como complemento de la oxitocina. Hay

Consistencia: dura, mediana o b landa : un cue llo blando está empezando a mod ificarse para el parto.

Borramiento: cuanto más borrado está un cuel lo, más ava nzada está la mad uración ce rvica l. Dilatación: a mayor d ilatación, más avanzado se encuentra el

que tener precaución para evita r un prolapso d e cordón. Maniobra de Hamilton: co nsiste en despegar las membranas

parto. Altura de la presentación (planos de Hodge): a mayor descenso de la presentación fetal, más ce rcano está el parto.

cal de prostaglandinas.

mediante un masaje intracervical para favorecer la liberación lo-

Métodos químicos locales, madu ración cervical: Prostaglandinas E2: son el método recomendado para la inducción del trabajo de parto. Pueden usarse en gel, tabletas o pesarios de

-- -- Dilatación

o

1-2 cm

3-4cm

5-6 cm

Borra miento

0-30%

40-50%

60-70%

>80%

Posición

Posterior

Media

Anterior

Consistencia

Dura

Media

Blanda

Altura de la presentación

Sobre estrecho superior

1-11 plano Hodge

11 1plano Hodge

liberación controlada. Madu ra el cuello y faci lita el parto posterior.

Métodos químicos sistémicos: Oxitócicos: producen contracc iones en casi toda circunstancia y son el método más eficaz cuando el cuello es muy maduro, cuando la bolsa está rota o si el resto están contraindicados. No se emplean o se usan en bajas dosis si hay cicatr ices o malfor-

IV plano Hodge

maciones uterinas, por el riesgo de rotura . Misoprostol: debe ser usado sólo como método de inducción del trabajo d e parto en mujeres co n muerte fetal intrauterina debido a su alto ri esgo de producir h iperestimulación y aumen-

Tabla 31 . fndice de Bishop

tar la aparición de meconio. Los valores oscilan de O a 13, conside rándose cérvix inmadu ro (desfavorable) cuando es igual o menor a 6, y favorable cuando es mayor a 6. Para mejorar el pronóstico y disminuir la tasa de cesáreas por fracaso de inducción, está indicada la maduración cervica l con prostaglandinas en cérvix inmad uros antes de p roced er a la infusió n de oxitoc ina.

Ideas clave "

z

Se entien de por embarazo cronológicamente prolongado cuando dura más de 42 semanas. Su etiología es desconocida.

Se han descrito algunos casos de muerte materna con el uso conjunto de prostaglandinas en gel y oxitocina i.v., por lo que se deben esperar al menos 6-12 horas desde la admin istración de p rostag land inas hasta el uso de la oxitoc ina.

"

Se debe realizar un control ambulatorio del estado de bienestar fetal mediante registro cardiotocográfico basal, ecografía y amnioscopia y decidir el momento de la inducción.

"

La inducción se realizará con prostaglandinas cuando el cérvi x esté desfavorab le (Bishop ~ 6) o con infusión de oxitocina y amniorrexis cuando el cérvix está favorable (Bishop > 6).

Manual CTO de Medicina y Cirugía, 2.a edición

Case Study A 31-year-old female, gravida 1, para O, was admitted at 42 weeks of gestation for induction labor. lnitial fetal heart rate was 140 beats/min and showed good reactivity and variabilit y but not uterine contractions. On exploration Bishop Score is 2. The most likely treatment is:

29 · Gestación cronológicamente prolongada

1) 2) 3) 4)

Laminaria cervica l dilators. Prostag land ins. Oxytocin. Nifedi pine.

Correct answer: 2

Elementos de tocología

ORIENTACIÓN

ENARM

Tema del que es imprescindible recordar lo que se resume en las Ideas Clave.

Estrecho superior

~-.:..__-

Transverso (13 cm)

30.1. Canal del parto La pelvis ósea se divide en pelvis mayor (o falsa pelvis, por su escasa relevancia en el parto) y pelvis menor. La verdadera pelvis obstétrica es la pelvis menor, que debe ser considerada como un cil indro óseo con una Planos de Hodge

forma pecu liar.

Estrecho inferior

El cilindro está acodado hacia delante, por lo que la cara anterior es mucho más corta y los planos de la abertura superior (estrecho superior) e inferior (estrecho inferior) no son pa ralelos. En la ca ra posterior hay un resalte (el promontorio) que hace que el estrecho superior sea elíptico, con el diámetro mayor en sentido transverso. El diámetro conjugado obstétrico o verdadero (del promontorio al punto más posterior del pubis) mide 10,5 cm, mientras que el diámetro transverso obstétrico mide 12 cm, por lo que el estrecho superior es, como se

Figura 68. Canal del parto y planos de Hodge

ha dicho, una elipse transversa . El estrecho inferior es el único que es elástico y t iene fo rm a de rombo con el diámetro mayor en anteroposterior. Está limitado latera lmente por las

30.2. Elementos fetales

tuberosidades isquiáticas, y anteroposteriormente por el coxis y el borde inferior del pubis. El diámetro longitudinal subsacropubiano mide de 9 a 12 cm, pues el coxis puede rechazarse, aumentando entonces dicho diámetro. El diámetro transverso interespinoso mide 11 cm. Los planos de Hodge son imaginarios y sirven para loca lizar la cabeza feta l con respecto a la pelvis. El primer plano pasa por el borde superior del pubis hasta el promontorio (es el plano superior del estrecho superior). El segundo plano es paralelo al anterior, pero por debajo del borde inferior

La cabeza del feto a término es un ovoide, de 9 cm por 13 cm de diámetro. Entre los pa ri etales y los frontales se encuentra la fontanela mayor (bregmática), y entre los parietales y el occipital, la fontanela menor (lambdoidea) que es el otro vértice del ovoide. Los diámetros transversos de la cabeza fetal (biparietal: 9,5 cm y bitemporal: 8,5 cm) son el obstáculo habitual.

30.3. Estática fetal

(Figura 69l

pubiano. El tercer plano pasa por las espinas ciáticas, y el cua rto, por el

La posición y la colocación del feto intraútero se describe con los siguien-

extremo del coxis (Figura 68) .

tes parámetros:

Manual CTO de Medicina y Cirugía, 2.a edición

Situación: es la relación entre el eje mayor del feto y la vertical ute-

ind icada la in ducción del parto cuando los beneficios de final izar

rina: longitudinal, transversa u ob licua según si dicho ej e es paralelo,

la gestación para la sa lud de la madre y del feto son mayores que

perpendicular u ob licuo con respecto a dicha vertica l.

los beneficios de permitir que el embarazo continú e. Se ll ama

Presentación: es la pa rte feta l que está en relaci ón con la pelvis ma-

maduración cervica l al proceso por el que se mejoran las carac-

terna, la que se «presenta» a la pelvis: de tal forma que puede ser

terísticas de l cérvix para que el parto se desarrolle por vía vagina l,

cefálica o pelviana (también denominada podálica).

dismi nuyendo el tiempo del parto y la tasa de cesáreas.

Posición: orientación de la presentación respecto a la pelvis: ante-

Cesárea: se emplea cuando las condiciones impiden una inducción

rior o púbica, posterior o sacra, derecha, izquierda.

o un parto con suficiente seguridad. Puede ser electiva, si se practica

Actitud: es la relación que tienen entre sí las diferentes partes feta-

de manera programada, o urgente (Figura 70) (Tabla 32).

les. En las presentaciones cefálicas corresponde al grado de flexión

El parto vaginal tras cesárea previa está contraindicado en caso

de la cabeza fetal: vértice, sincipucio, frente y cara.

de: cesá rea mediante laparotomía en "T", cirugías previas transfúndicas, historia obstétrica de rotura uterina, complicación médica que contraind ique el parto vag in al o antecedente de dos o más cesá reas previas. La probabilidad de parto vagina l tras cesárea previa dism in uye en caso de ind ucción del parto, obesidad, mujeres mayores de 40 años, fetos de más de 4.000 g, entre otros. Sin embargo, los beneficios de parto tras cesárea se asocian a una estancia hospitaria menor, menos sangrado, menor tasa de infección y disminución de episodios tromboembó-

Posició n

licos. Parece que tras una pru eba de parto fallida en pacientes con cesárea previa se describe una mayor in cidencia de rotura uterina, infecc iones e incluso transfusiones. Como toda cicatriz uterina, la hi storia obstétrica de cesárea previa, con ll eva a un Actitud

aumento de la in cidencia de placenta previa y acreti smo placentario. Prueba de parto: consiste en colocar a la paciente en la mesa

Presentación

de partos, para que al estar en decúbito supino con flexión de los miembros inferiores, se aplane la lordosis y el eje columna-pelvis sea más favorable. Se emplea en la parte final de la dilatación y en Situación

los casos en que existe duda fundada de que pueda evolucionar por vía vaginal (sospecha de desproporción).

Figura 69. Estática fetal

30.4. Condiciones generales del parto Las condiciones del cérvix se miden con el índice de Bishop. Se considera que el parto comienza cuando se alcanzan 2 cm de dilatación, cuell o borrado o acortado un 50% y d inámica activa: al menos dos contracciones cada 1O m inutos de mediana intensidad. El periodo de di latación termina con la di latación compl eta (ausencia de cérvix alrededor de la presentación feta l). Antes de llegar a la di latac ión completa, no se pueden emplear fórceps, ventosa o espátu las, por lo que se practicará cesárea en caso de ser necesaria la extracción fetal en

Factores de riesgo materno

Placenta previa

~yW~!) ~

Desprendimiento prematuro de placenta

Presentación anómala del feto

esa fase. El periodo expulsivo comienza con la dilatación completa y termina con la expulsión fetal. Tras la expulsión fetal se inicia el alumbramiento (la expulsión de la placenta) . Rotura de membranas: se denomina rotura prematura de membranas a la rotura antes del inicio de l trabajo de parto. Desde el

D"pmpo"ióo pelvicocefálica

momento del in icio del mismo, hasta la di latación comp leta, la rotura se ll ama precoz. En d il atac ión completa, recibe el nombre de tempestiva. Si la bolsa está íntegra en el momento de la expulsión feta l, se llama tardía . La rotura oportun a sería la que ocurre en el periodo de dilatación, es decir, englobaría a la precoz y a la tempestiva. Inducción: procedimiento dirigido a desencadenar contracciones uterinas por med ios mecánicos, méd icos o ambos, en un intento de que el parto tenga lugar por vía vagina l. Se considera

30 · Elementos de tocología

Figura 70. Indicaciones de la cesárea

(W) ~

Ginecología y obstetricia Indicaciones de cesárea electiva

Indicaciones de cesárea urgente

Placenta previa centra l

Abruptio placentae (DPPNI)

Gemelos monoamnióticos

Rotura de vasos previos

Alteraciones de la estática fetal: presentación de frente, de cara, situación transversa

Rotu ra uterina

1

30

signo de que el diámetro biparietal ha sobrepasado el estrecho superior. Es un instrumento rápido, que resuelve malrotaciones fetales (tra ctor y rotador), lo que le co nvierte en instrumento ideal en ca so de sufrimiento fetal (siempre q ue se den las condiciones que hagan posible su apl icac ión) (Figura 72). Los fórceps pueden ser tractores (Sim pson), retadores (Kjelland) o m ixtos (Sa linas). Para la distocia de cabeza última en parto de nalgas

Presentación en pelviana

Prolapso de cordón

Factores de riesgo materno que puedan perjudicar al feto (por ejemplo, VIH)

Sufrimiento fetal (pH < 7.2 va riabilidad silente o sinusoidal)

Historia obstétrica desfavorables (dos cesáreas previas, miomectomía con apertura de cavidad ...)

Distocia o falta de progresión del parto

Desproporción pelvicefálica

Mal estado materno

se utiliza el fórpces de Piper. Por su aplicaciones, en relación al nivel de los planos de Hodge de la pel vis en que se aplican y el diámetro biparieta l, únicamente hoy en la obstetricia moderna clas ifican en: Fórceps bajo: cuando el d iámet ro biparietal de la cabeza fetal

se encuentra en el pi so pél vico o a nivel del IV plano de Hodge. Fórceps medio- bajo: cabeza encajada, pero el DBP se encuentra en el /11 plano de Hodge.

Tabla 32. Indicac iones de la cesárea

Los demás t ipos por su altura segú n los planos de Hodge como el alto o libre están proscritos hoy en día.

En general, pueden nacer por vía vag inal todos los fetos con presentación cefálica, excepto las variedades de frente,

y los fetos con presentación de

cara variedad mento posterior, que será n ind icación de cesá rea (Figura 71 ).

-

¿Cuándo?

Expulsivo (cabeza deflexionada)

Ventosa

Fórceps

Espátulas

Situaciones mal rotación

Menor trauma y no precisa anestesia

Expulsivo

Plano de Hodge

IV Plano

111 Plano

11 Plano

Función

Acorta r expulsivo

Ideal si SF. El más rápido

Es lenta para SF

Tabla 33. Espátulas, fó rceps y ventosa

Figura 71. Parto vaginal en presentación cefálica

30.5. Parto instrumental Se entiende por parto instrument al el empleo de fórceps, ventosa o espátulas que aplicados general mente sobre la cabeza fetal

y, util izando la

fuerza de tracción, suplementan el trabajo de parto, asistiendo a la madre en el periodo expulsivo. Estará ind icado en situación de amenaza ma-

Figura 72. Extracció n fetal co n fórcep s

terno-feta l q ue se alivie por el pa rto, siempre y cuando se pueda realizar de forma segura. Los requ isitos para rea lizar un parto instrumenta l son:

Vacuoextractor o ventosa: se aplica en la cabeza fetal una cazoleta

presentación cefá lica, dilatación cervica l comp leta, membranas rotas, ca-

conectada a un sistema de vacío que ejerce una presión negativa.

beza encajada y conocida la posición exacta (Tabla 33).

Este instrumento abrevia el periodo expulsivo, pero es más lento que

Espátulas: son semejantes a dos cucharas no articulada s que facilitan la

el fórceps, por lo que ante un sufrimiento fetal agudo se prefiere el

deflexión de la cabeza fetal, por lo que se emplean en la parte final del expulsivo en cuarto plano de Hodge. Su principal indicación es abreviar un expulsivo largo (agota miento materno) o con patología fetal.

fórceps o la cesárea, si no se dieran las condiciones obstétricas para la utilización de este. En cambio, presenta la ventaja de ser menos traumático y de poderse aplicar sin anestesia.

Fórceps: es un método de extracción fetal que se emplea durante el periodo expulsivo, conceptualmente semejante a unas tenazas. Re-

quiere presentación cefáli ca (sal vo el raro caso del fórceps de cabeza última, en el pa rto de nalgas compl icado) y cabeza más o menos normal (no en hidrocefa lias o anencefa lias). Para poderse aplica r con seguridad, el punto g uía debe alcanzar el tercer plano de Hodge,

Para hacer un parto instrumental es necesari o: dilatac ió n co mpl eta, bo lsa rota, presentac ión cefál ica y cabeza fetal si n anomalías.

Manual CTO de Medicina y Cirugía, 2. 8 edición

30.6. Parto en presentación pelviana Su frecuencia es de aproximadamente uno de ca da 30 partos. En la variedad de nalgas puras (Figura 73 ), los mu slos están fl exionados sobre el tronco y las piern as extendidas por delante del mismo, presentando a la pelvis única mente las nalgas. Es la va ri edad más frecuente en las gestantes a términ o. En la va riedad de nalgas completas, el feto se encuentra en una actitu d semejante a la presentación de vértice (m uslos flexionados sobre el t ronco y piernas fl exionadas sobre los muslos), pero ha inverti do la polaridad, presentando a la pelvis las na lgas y ambos pies. El resto de prese ntaciones en pelviana se denom inan como nalgas incompletas (nalgas y un pie). El diagnósti co se sospecha por tacto vag inal, pero debe hacerse una ecografía abdominal. Un estud io mult icéntrico publicado a fina les del año 2000 observó peores resultados perinata les en los partos de na lgas vía vag inal que en aquellos terminados por cesárea, por lo que se acepta la cesárea electiva como terminación de elección de los pa rtos

Incompleta

Figura 73. Variedades de presentación pelviana

en pelvia na. La ve rsió n ext ern a es un a mani obra obstét rica q ue intenta co nve rti r una prese ntac ión de nalgas en una cefálica, más favorab le de ca ra al parto vag inal y así disminu ir el núme ro de cesáreas realizadas con esta indicació n. Se reco mienda que se practique a parti r de la semana 37. Está contrai ndicada en mujeres con cicatriz uterin a por el riesgo de rotura uteri na. Figura 74. Presentación en pelviana: cesárea (izqu ierda) y parto vagina l (derecha)

Ideas clave

asociado. Cuando sólo tien en una cicatriz uterina, se pued e plantear un part o va g inal en casos seleccionados.

~

"

La situación es la relación entre el feto y la vertical uterina y puede ser longitudinal, transversa u oblicua .

"

La presentación es la parte fetal que está en relación con la pelvis materna. La gran mayoría son presentaciones cefálicas. De estas, todas pueden se r un parto vagin al, excepto la presentación de frente y de cara variedad m entoposterior.

"

La posición es la orientación respect o a la pelvis, y puede ser anterior o posterior, derecha o izquierda.

"

La actitud nos informa del grado de flexión de la cabeza fetal. Cuanto más deflexionada esté, más difícil es el parto por vía va ginal (vértice, sincipucio, frente y cara).

"

Las pacientes que tienen dos o más cicatrices uterina s no deben tener un parto vaginal por el riesgo de rotura uterina

30 · Elementos de tocología

"

Las espátulas se emplean en la parte final del expulsivo. Re quieren la dilatación completa y que la presentación esté en un cuarto plano de Hodge. Su principal indicación es abreviar un expulsivo largo o con patología fetal.

"

El fórceps se emplea para la extracción fetal cuando presenta dilatación completa y la presentación está en el tercer plano de Hodge. Es un instrumento rápido que permite realizar tracción y rotación, lo que lo convierte en un instrumento muy útil en casos de sufrimi ento fetal.

"

La ventosa ejerce una presión negativa, y así permite que se traccione la cabeza fetal. Precisa dilatación completa y que la presentación esté en un segundo plano de Hod ge.

"

El feto en presentación pelviana es indicación de cesárea. La versión externa intenta reducir el número de cesáreas por esta indicación pero está contraindicada en mujeres con cicatriz uterina.

Ginecología y obstetricia

Casos clínicos Paciente en periodo expulsivo de 34 semanas de gestación, presentación en tercer plano de Hodge, posición OIIT y aparición de una bradicardia fetal de 60-70 lat/min, constatándose la existencia de una acidosis respiratoria (pH = 7, 18, PC0 2 = 68 mmHg, EB = -2 mEq/1). ¿Cuál sería la actitud correcta? 1) Realización de una cesárea . 2) Realización de un vacuum. 3) Realización de un fórceps. 4) Colocación de un pulsioxímetro para conocer la saturación de 0 2 .

RC: 3 La matrona avisa al ginecólogo para atender a una gestante a término en trabajo de parto que lleva una hora en el período expulsivo. En la exploración, se objetiva feto único en presentación de cara mentoposterior de 111 plano. La conducta a seguir es: 1) Aplicación de fórceps. 2) Aplicación de vacuum extractor.

Case Study : A 28-year old primigravida woman presented at our hospital at 36 weeks gestation with a central placenta previa and vaginal bleeding. She was admitted for observation but the patient presented premature rupture of membranes and the bleeding increased. Fetal heart rate was 155 beats/min and there were regular uterine contractions. The most correct option is:

1

30

3) Versión interna y gran extracción . 4) Cesárea. RC: 4 Una paciente secundigesta, con antecedente de parto vaginal anterior, ingresa en su 34• semana de gestación por parto en curso. Mediante tacto vaginal se diagnostica dilatación de 6 cm y presentación pelviana (nalgas puras), variedad S.I.I.A., en 11 plano. Ecográficamente, se estima que el peso fetal estará comprendido entre 2.100 y 2.300 g. Señale la conducta obstétrica correcta: 1) Permitir la evo lución espontánea, evitando la ayuda manual durante el periodo expulsivo. 2) Dejar evolucionar hasta periodo expulsivo, y practicar maniobras de ayuda manual para abreviar el periodo expulsivo, tras la sa lida del ángulo inferior de la escápula. 3) Permitir la evolución hasta periodo expulsivo, y finalizar este mediante aplicación de fórceps, para evitar la distocia de cabeza última. 4) Terminar el parto mediante cesárea. RC: 4

1) 2) 3) 4)

Antibiotics. Tocolytics and antib iotics. Vaginal delivery. Cesarean section.

Correct answer: 4

..---._ G,____inecología y obstetricia

Posparto y puerperio

ORIE~JTACIÓN

ENARM

Es un tema poco importante, salvo el apartado de la hemorragia posparto, al que sí hay que prestarle atención.

tos instrumentales que lesionan el cana l blando del parto, aunque también pueden aparecer en partos espontáneos muy rápidos o por macrosomía fetal (Figura 75).

El puerperio es el periodo que comprende desde el final del parto hasta la primera menstruación. Dura aproximadamente 40 días, denom inándose puerperio inmediato a las primeras 24 horas posparto. Entre las complicaciones que pueden aparecer en este periodo, se destacan las que vienen descritas en los apartados expuestos a continuación.

31.1. Hemorragia posparto Se define como el sang rado vag ina l excesivo (> 500 mi tras un parto va-

> 1.000 mi tras una cesá rea) y se d ivide en hemorrag ia posparto precoz (antes de las 24 h) y tardía (de 24 h hasta seis semanas pospa rto). Aparece en el S-8% de los casos, siendo la primera causa de trasfusión

ginal o

obstétrica. Puede producir el síndrome de Sheehan: necrosis hipofisaria posparto que ocasiona hipogalactia, amenorrea, disminución del vello pubiano

y axi lar, hipotiroidismo e insuficiencia suprarrenal.

Figura 75. Desgarro cervical en el parto útero: es rara la rotura uterina, siendo la dehiscencia de la cesárea anterior la causa más frecuente de la misma. Cursa con mala contracción uterina. Cérvix: laceraciones y hemorragias tras la expulsión fetal; persis-

Etiología Atonía uterina (SO%): es la causa más frecuente de hemorragia posparto temprana. Una vez expulsada la placenta se produce una contracción permanente de la musculatura uterina que apresa entre las fibras uterinas las bocas de los vasos sang rantes (ligaduras de Pinard) favorec iendo el cese de la hemorragia. Si el miometrio no se contrae, no se forman dificultando la constricción vascular y la formación local de trombos sanguíneos. Los factores predisponentes de la atonía uterina incluyen: sobredistensión uterina (por embarazo múltiple, polihid ramnios o macrosomía fetal), g ran multiparidad, uso prolongado de oxitocina, parto prolongado, manipu lación uterina,

ten, aun, con buena contracción uterina.

Vagina: las de ligamento ancho precisan laparotomía . Vulva: peligro en los hematomas que no se d iagnostican y producen gran hipovolemia silente en el periodo posparto. Retención placentaria (5-1 0%): aparece más en placenta accreta (apoyada sobre el miometrio) y en succenturiata (cotiledón en islote). Si se ve tejido intrauterino en la ecografía, debe realizarse legrado puerperal. Coagulopatías: aparecen en el contexto de abruptio, aborto diferido, embolia de liquido amniótico, sepsis, preeclampsia y grandes transfusiones. A veces aparece en la enfermedad de von Willebrand o en la trombocitopenia autoinmun itari a.

útero miomatoso, útero de Couve laire (extravasación de sa ngre al miometrio), infección amn iótica, uso de relajantes uterinos (su lfato de magnesio, agentes anestésicos halogenados), etcétera.

Prevención

Lesiones del canal del parto (20%): constituyen la segunda causa de hemorragia en el posparto. Suelen estar re lacionados con par-

El manejo activo del tercer periodo del parto ha sido úti l como un medio efectivo para prevenir la hemorrag ia postparto. Los cuidados a realizar

Ginecología y obstetricia incluyen el pinzamiento temprano del cordón umbilical, tracción contralada del cordón con palpación uterina, inspección de la placenta y cana l del parto y admin istración de fármacos uterotónicos de manera profiláctica.

Oxitocina: la adm inistración de 1O Ultras el nacimiento del hombro anterior ha demostrado reduc ir el riesgo de hemorragia postparto hasta en un 60o/o.

Ergonovina: la administración de una ampolla en dosis única posterior al nacimiento del hombro anterior también resulta útil. Sintometrina (oxitocina más ergonovi na): es más efectivo pero está asociado con más efectos adversos. No se recomienda la utilización de ergonovina o sintometrina en pacientes con hipertensión.

Carbetocina (análogo de la oxitocina) : no ha demostrado ser superior al uso de la oxitocina pero sus ventajas radican en la adm inistración en dosis ún ica y que su efecto se mantiene por va ri as horas, si bien todavía no se recomienda su uso sistemático.

Tratamiento El manejo consiste en una actuación rápida, pero sistemática. Entre las medidas generales a instaurar, se encuentran la canalización de una adecuada vía venosa para reposición de líquidos, el control de la diuresis, la oxigenoterapia, la analítica urgente

y la exploración física. El trata-

miento es etiológico, por lo que es vita l ll egar con rap idez al diagnóstico de la patolog ía responsable de la hemorrag ia posparto. El manejo de la atonía uterina consiste en: masaje uterino, fármacos uterotónicos,

31

1

31.3. 1nfeeción pos parto y puerperal oc

Se considera fieb re puerperal a la temperatu ra superior a 38 en dos tomas sepa radas, entre los días segu ndo y décimo, y es el signo guía de la infección puerpera l. La infección posparto supone 1/3 de la morta lidad materna de origen obstétrico. Las infecciones puerperales son poli microbianas.

Endometritis: suele aparecer durante el segundo o el tercer día posparto. El factor más importante es la cesárea, donde el riesgo de endometritis es 30 veces mayor que en el parto vaginal. Otros factores que favorecen la endometritis son: rotura de membranas prolongada, corioamnionitis, tactos vaginales, parto prolongado, preeclampsia, parto instrumental, anem ia, desnutrición, obesidad. Hay fiebre, leucocitosis y útero subinvo lucionado y doloroso a la pa lpación, con loquios maol ientes, que p ueden llegar a ser puru lentos. En los casos graves, hay ma lestar, hi potensión, íleo y shock. Debe realizarse análisis de orina, pues la infección urinaria puede dar un cuadro semejante. El tratamiento se basa en la antibioterapia intravenosa en dosis altas, cubriendo los gérmenes presentes en el tracto genital, que son los más frecuentemente implicados. El régimen de clindamicina y gentamicina presenta una elevada eficacia. En las pacientes con factores de riesgo se debe instaurar profilaxis intraparto, siendo el fármaco de elección ampicilina o amoxicilina + clavulánico. Mast itis: aparece más en primigrávidas, casi exclusivamente en las lactantes, hacia el segundo o tercer día de puerperio, por fisuras del aureus del neonato. Las mamas están tensas,

pezón e infección por S.

taponamiento uterino, tratamiento quirú rgico (ligadura vasos uterinos, ligadura arteria ilíaca interna o histerectomía obstétrica como última

eritematosas, dolorosas, congestivas, y puede haber febrícula y adenopatías axi lares. En ocasiones es focal y evoluciona al absceso. El

medida a realizar).

tratamiento incluye calor local, vaciamiento mamario tras las tomas y antibioterapia específica (cloxacilina, amoxicilina-clavulán ico). Si hay

Los fármacos uterotónicos disponibles son los siguientes: Oxitocina: es el más fisiológico. Produce contracción intermitente de la fibra uterina. Puede utilizarse vía intramuscular, intramiometrial

absceso, además del tratam iento antibiótico es preciso drenaje quirúrgico, desbridamiento y colocación de drenaje.

o intravenosa. Tiene efectos secundarios como náuseas, vómitos e intoxicación acuosa por acción antidiurética de la oxitocina.

31.4. 1nhibieió n de 1a 1aeta neia

Ergonovina: es un poderoso uterotónico. Se puede admin istrar por vía intramiometrial, intramuscula r o intravenosa. Su empleo no excluye el de la oxitocina y con frecuenc ia se usan para potencia r su efecto. Está contraind icado en casos de patolog ía hipertensiva. Sus

La lactancia está contraind icada en caso de infección materna por tuberculosis o por infección por VIH (esta contraindicación es relativa en

efectos secundarios son: hipertensión arterial, hipotensión, náuseas y vómitos y espasmo arterial coronario.

países en vías de desarrollo). En caso de mujeres con infección por herpes activo, pueden dar lactancia si no hay lesiones en las mamas. La hepatitis B crónica no es contra indicación para la lactancia si se hace

Carboprost: derivado de las prostaglandinas que potencia la contractilidad uterina y producen vasoconstricción. Está contraindicado en pacientes con enfermedad cardíaca, pulmonar, renal o hepática

correctamente la profilaxis al recién nacido con gammaglobulinas y la vacuna.

activas y los efectos secundarios más importantes son: vómitos, diarrea, náuseas, broncoespasmo y desaturación.

31.2. Inversión uterina Se define como el prolapso del fondo uterino a través del cérvix y se debe, en general, a una excesiva tracción del cordón umbil ical antes del alumbramiento completo. Se llama incompleta cuando el fondo no protruye por el cérvix. El riesgo fundamental es la hemorragia asociada, aunque a veces aparece endomiometritis tras la reposición uterina manual, que es la actitud de elección. El diagnóstico es evidente: dolor, hemorragia y masa vaginal!endocervical blanda rojo-azulada en ausencia del globo uterino.

La transmisión de CMV a través de la leche no produce enfermedad al recién nacido, por ello no es una contraindicación. La drogadicción (cocaína, heroína) también contraindica la lactancia materna, así como la toma de determinados fármacos: ciclofosfamida, ciclosporina, ergotamina, litio, o metotrexato. También puede estar indicada la inhibición de la lactancia por motivos sociales maternos o psicosis puerperal. Hay contraindicaciones para la lactancia materna que dependen del recién nacido: anomalías de la boca y de la vía respiratoria, alteración en la succión/deglución, metabolopatías (galactosemia, feni lcetonuria), etcétera . El fármaco de elección para inhibir la lactancia es la cabergolina. Se acompaña de hielo local y de sujetador compresivo. Durante la lactancia natural, se administran las tomas cada tres horas, aunque es más importante seguir la demanda del recién nacido.

Manual CTO de Medicina y Cirugía, 2. 8 edición

31.5. Otros problemas del puerperio

Las convulsiones puerperales sugieren eclampsia, especialmente en la primera semana . Hay que evalua r otras posibilidades (foco orgán ico, eléctrico o hemorrágico central). Las alteraciones mentales posparto son frecuentes y pueden manifestarse de tres formas clínicas: t risteza o melancolía puerperal, depresión puerperal y psicosis puerperal. El cuadro más leve y frecuente de los trastorn os del ánimo es la tris-

Los entuertos son contracciones uterin as dolorosas que pueden aparecer dura nte el puerperi o, más frecuentes en multípa ras o cuando se ha empleado oxitocina. Suelen estar en relación con la li beración de oxitocina por el estímulo del pezón. La tiroiditis posparto es de origen inmunológico, y cursa como crisis leve de hipertiroidi smo, segu ida de hipotiroidi smo. Suelen nor-

teza o melancolía pu erperal que afecta casi al 80% de las mujeres. No suele necesitar tratami ento psiquiátrico y responde bien al apoyo psicológico, com prensivo, tra nqui liza nte e informativo. Está en discusión una posible influencia de los ca mbios hormonales en su et iología. En ocasiones se in sta ura un estado man ifiestamente psicótico, llamado psicosis puerpera l, que se caracteriza por un cuad ro confuso-o nírico. Aparece más en pacientes con antecedentes maniacodepresivos y en primípa ras. Muerte materna: suele ser menor de 10/100.000 nacidos vivos, aunque au menta con la edad materna. La ca usa más frecuente es la hemorragia (30%) : por abruptio, atonía uterina, CID, etc. La segunda

ma liza rse entre los seis y los nueve meses. El síndrome hemolítico urémico posparto (fracaso renal ag udo, anem ia hemolíti ca microangiopática y trombocitopenia) es raro. La tromboflebitis o el tromboembolismo puerperal, au n hoy, con la deambulación precoz, sigue siendo un problema importa nte. Su diagnóstico y t ratamiento es el mismo que fue ra del embarazo-puerperio, y existe riesgo de rec urre ncia tanto en embarazos posteri ores como en la toma de anovulatorios. La parálisis nerviosa periférica materna suele ser producida por la compresión de la ca beza fetal o du ra nte la aplicación de un fó rceps y afecta con mayor frecuencia al obturador, el femora l y el peroneo.

ca usa más habitu al es el embolismo pulmonar (23%, aunq ue en Estados Un idos es la pri mera ca usa). La tercera ca usa más frecuente es la HTA (18%).

Cura espontáneamente en unas semanas.

Ideas clave "

La causa más frecuente de hemorragia posparto es la atonía uterina. Se trata de cuadros de hemorragia temprana. El miometrio no se contrae, por lo que no deja de sangrar.

"

Los factores predisponentes son sobredistensión uterin a, multiparidad, uso prolongado de oxitocina, útero miomatoso, manipulación excesiva, infección amn iótica y uso de relajantes uterinos.

"

El tratamiento consiste en masaje uterino, fármacos uterotónicos (oxitocina, ergonovina, carboprost), tapon amiento uterino y tratamiento qu irúrgico.

"

Las les iones del canal del parto son la segunda caus a más fre cuente y también son hemorragias temprana s. Se re lacionan con partos precipitados o instrumentales y con la macrosomía fetal. Su tratamiento cons istirá en localizar la lesión y suturarla .

"

mos, placentas succenturiatas). Se trata mediante la realizac ión de un legrado puerperal.

~ "

La endometritis puerperal es un cuadro frecuente que se caracteriza por la presencia de picos febriles a las 24-48 horas del parto asociados a do lor abdominal, sobre todo a la exploración uterina, subinvolución uterina y loquios abundantes. El tratamiento es antibiótico, con pautas de amplio espectro para cubrir bacilos gramnegativos, que son los que frecuentemente están imp licados (gentamicina + clindamicina).

"

Los factores de riesgo de endometritis son la cesárea, los partos instrumenta les, la rotura de membranas prolongada, la corioamnionitis, los partos prolongados, la anemia .. . En estos casos, está indicada la profilaxis con ampicilina o con amoxicilina-clavulánico.

"

La lactancia está contraindicada en casos de infecciones (tubercu losis, VIH, hepatitis agudas), drogad icción o por la toma de fármacos que puedan pasar a la leche (ciclofosfamida, ciclosporin a, litio, metotrexato). El fármaco de elección es cabergolina, junto a med idas locales.

La retención de restos placentarios suele producir hemorragias tardías. Está predispuesta por alteraciones placentaria s (acretis-

Casos clínicos Una paciente sometida a cesárea por inducción fallida tras amniorrexis prematura presenta, en su tercer día de puerperio, malestar general, fiebre de 38,5 oc, loquios fétidos, útero subinvolucionado y doloroso a la palpación. ¿Cuál es el diagnóstico más probable? 1) 2) 3) 4)

Endometrit is pu erperal. Dehiscencia de la histerectomía . Retenci ón de restos placentarios. Necros is isq uémica de mioma uterin o.

RC: 1

31 · Posparto y puerperio

Primigesta que, durante el final del periodo expulsivo, presenta un sangrado de sangre roja muy escaso que se hace francamente hemorrágico inmediatamente después de expulsarse el feto. La placenta es alumbrada 1O minutos después; el útero se contrae correctamente. La hemorragia por vía vaginal continúa. De los siguientes diagnósticos, el más probable es: 1) 2) 3) 4)

Atonía uterina. Reten ción de un cotil edó n place ntari o. Desga rro de cuell o uterino. Coag ul opatía.

RC: 3

Ginecología y obstetricia

Case Study A 42-year-old, gravida 8, para 7, woman was admitted to our hospital at 39 weeks gestation for a normal vaginal delivery. lnitial fetal heart rate was 130 beats/min and showed good variability. Two hours of regular contractions led to labor outcome as a normal vaginal delivery but presented a massive bleeding dueto uterine atonía in the post-placenta! pe-

1

31

riod. Which option is less correct in the management of this complication? 1) 2) 3) 4)

Oxytocin. Postpartum curettage. B-Lynch suture technique. Postpartum hysterectomy.

Correct answer: 2

.--._G_...._i~ne__c_o_Lo_gía y obstetricia_

Estados hipertensivos del embarazo

OPIE~JTACIÓN

ENARM

Tema al que hay que dedicarle tiempo.

32.1. Epidemiología y etiología

32.2. Fisiopatología y manifestaciones clínicas La preeclampsia (Figura 76) es una enfermedad mu lt isistémica y mu ltifactori al ca racterizada por la exist encia de daño endotelial que precede al d iagnósti co clínico. La placenta juega un papel importante en la patogénesis de la preeclampsia, existiendo una placentación anorm al

Su frecuenc ia oscila entre 1-5% de embarazos. Su etio log ía sigue siendo un misterio sin resolver, aunque parece ser multifactorial. Estos factores

y una d ism inución de la perfusión placentaria. La isquemia placentaria conduce a una activación y a una disfunción del endotelio vascu lar ma-

etiológicos se dividen en dos grandes grupos:

terno, resultando en un aumento de la producción de endotelina y trom-

Factores placentarios: la preeclampsia es un síndrome que apa re-

ce excl usivamente en el embarazo y necesita que haya placenta para produci rse. Está relacionado con un defecto en la placentación y un fa llo en la reorganización de las arterias espirales (las arteri as espirales irrigan la superficie endometria l y se reorganizan durante la gestación, facil itando la invasión del trofoblasto para facilitar el interca mbio uteroplacentario). Este fa llo en la placentació n puede t ener un origen inmunológ ico: dado que la unidad placentaria funciona como un aloinjerto, al fracasar los mecanismos normales de inmunotolerancia entre

boxano, un aumento de la sensibilidad vascular a la angiotensina 11 y una disminución en la form ación de agentes vasod ilatadores (óxido nítrico y prostaciclinas). Tod as estas alteraciones provoca n un aumento de las resistencias vasculares, mayor agregabilidad plaquetaria, activación del sistema de la coag ulación y disfunción endotelial, que se traducen en los síntomas y signos de la enfermed ad. El endotelio g lo merular es muy sensible a la lesión de la preecl ampsia y presenta una lesió n caract erística, la endoteliosis glomeru lar, q ue produce proteinuria. Esta proteinuria suele desaparecer poco ti empo después del parto.

trofoblasto y tejido materno, se iniciaría una reacción inmunitaria anorma l entre antígenos paternos y maternos. Presentan más riesgo las mujeres que han tenido poco contacto previo con esos espermatozoi-

El filtrado g lomerular suele ser norma l y los niveles de ren ina y de angiotensina, bajos. La lesión vascula r y la hipoproteinemia conducen al ede-

des, como ocurre en las nulíparas, o en las mu ltíparas cuando cambian de pareja. Por este motivo, también poseen mayor riesgo las mujeres que usa n métodos anticoncep-

en el posparto. Se produce una retención de sod io y d isminuye el acla-

ma extracelular. El edema agudo de pulmón puede ocu rrir, sobre todo,

t ivos de barrera y en emba razos con donación de ovocitos. Otra posible ca usa no inmunitaria es

Preeclampsia

el excesivo tamaño de la placenta

Patología placentaria Liberación de factores tóxicos circulantes

(gestación gemelar y enfermedad t rofoblástica gestacional). Factores maternos: son factores de riesgo de preeclampsia: la nu-

t

t Disfunción endotelial Vasoconstricción e hipercoagulabilidad

liparidad, la obesidad, los antecedentes familiares de preeclampsia-eclampsia, la preeclampsia en gestación previa, la hipertensión crónica, la enfermedad renal crónica, la d ia betes mell it us preges-

HELLP

tacional, la gestación múltiple, la presencia de trombofilia s.

Figura 76. Enfermedades cutáneas localizadas en la mucosa oral

Ginecología y obstetricia ramiento de ácido úrico, provocándose hiperuricemia. La creatinina se

1

32

Repercusión f etal

eleva sólo en los casos más graves por d ism inución de su aclaram iento. La rotu ra hepática o el infa rto hepático masivo son raros, pero fatales.

La mortalidad perinatal oscila entre un siete y un 60% debida a aborto ta rdío, muerte intrauterina, sufrimiento intra uterino, prema tu ridad (es-

En el ámbito digestivo, la enfermedad produce dolor epigástrico, vóm i-

pontá nea y iatrógena) y CIR.

tos y elevación de transam inasas, secundarios a necrosis hepatocelular. El síndrome HELLP es la asociación de: Hemól isis, Elevación de enzimas hepáticas Liver en inglés, y Plaquetopenia, (Low Platelets) Cerebralmente,

32.5. Tratamiento

el vasospasmo se manifiesta como alteraciones occipitales (cefalea frontal, fotops ias, escotomas y ceg uera cortical transitoria), y no como una verdadera encefa lopatía hipertensiva, sin que el grado de vasospasmo se

La raíz del problema está en la pl acenta, por lo que el único tratamiento

correlacione directamente con la graveda d (hasta un 20% de eclampsias

definitivo es quitarla; es decir, terminar la gestación . Los antih ipertensivos

debuta n co n TA normal o límite).

sólo suponen un tratamiento sintomático. La mu erte de la paciente (024%) puede producirse por: rotura o infarto hepático masivo (muy infrecuente en gestaciones b ien controladas), lesiones cerebrales, CID, abrup-

32.3. Clasificación

tia, fallo renal agudo o edema pul mona r. Medidas generales: dieta normosód ica ri ca en proteínas, reposo relativo, control de tensión y diuresis. Hay que descartar anemia, trom-

La clasificación de la hipertensión en el em barazo es la sigu ient e:

bopenia y alteración hepá tica asociadas.

Hipertensión gestacional: es la aparició n de hipertensión durante

Hipotensores: la indicación de trata miento es la persistencia de TA

el embarazo o en las primeras 24 horas del puerperio sin otros signos

d i astó li ca~

de preeclampsia.

100 mmHg o de TA sistólica ~ 150 mm Hg.

a-metildopa: es un fármaco que actúa inhibiendo la producción de

Preeclampsia: co nsiste en encontrar después de la 20• semana de

noradrenalina en las terminaciones nerviosas simpáticas, ejerciendo

gestación hipertensión más proteinuria (con o sin edemas). Es pre-

como un fa lso neurotransmisor. Se consigue un buen control de la

coz si aparece antes de la 20• semana y se asocia a los casos de en-

TA sin grandes fluctuaciones y sin producir hipotensión postura!. Es

fermedad trofoblástica, embarazo gemela r o hidrops fetal.

de acción lenta por lo que se usa en casos leves, de manera ambula-

Eclampsia: es la aparición de convu lsiones en una paciente con

toria. El efecto secundario más frecuente es la somnolencia.

preeclampsia, cua ndo no pueden ser atribuidas a otra causa.

Hidralacina: prod uce hipotensión por su acción directa sobre el

Hipertensión crónica: es una HTA detectada previamente a la ges-

músculo liso de la pared de las arteriolas, provocando vasod ila-

tación o antes de la 20.• semana de embarazo y que persiste tras el

tación y disminución de las resistencias periféricas. Ocasiona un

embarazo.

aumento del gasto y de la frecuencia ca rd íaca. Los efectos secun-

Hipertensión crónica con preeclampsia sobreañadida: consiste

darios más habituales son exantemas similares al lupus, que des-

en una hipertensión crón ica que a lo largo de la gestación desarrolla

apa recen t ras suspender el tratamiento, palpitaciones, cefaleas,

proteinu ria significativa y los síntomas y sig nos de una preeclampsia.

vómitos y diarrea. Puede disminuir el flujo uteroplacentario.

Labetalol: es un antagonista competitivo de los receptores a y ~ -adrenérg i cos.

32.4. Definiciones

Actúa disminuyendo la resiste ncia vascular pe-

riféri ca med iante el bloqueo de los receptores a-adrenérgicos de las arterio las periféricas, pero además bloquea los recepto res ~-adre n érg i cos, con lo que protege al co razón del reflejo sim-

Hipertensión: aumentode 30mm Hg en lasistólicaode 15 m m Hgen la

pático t aquicard izante que produce la vasodi lat ació n periférica.

diastólica sobre los valores normales, o t ensiones superiores a 140 o 90

Tiene pocos efect os secu ndarios

mmHg, medidas dos veces con un intervalo de al menos cuatro horas.

incrementa el flujo útero placent ario por lo que es un fá rm aco

y presenta la ventaja de que

Proteinuria: se define como la existencia de 300 mg o más de pro-

muy utilizado. Se puede administrar por vía ora l o endovenosa.

teínas en orina de 24 h o 30 mg/dl en muestras aisladas.

Nifedipina (calcioantagon ista): es un vasodilatador periférico que además tiene acció n tocolítica. Se debe tener cuidado

Criterios de gravedad

cuando se asocia al sulfato de mag nesio ya que potencia su efecto, teniendo mayor riesgo de pa ra da ca rdiorrespiratoria.

TA sistó li ca~ 160 mmHg. TA diastólica ~ 11 O mm Hg.

Los siguientes fármacos están contraind icados:

Proteinuria de~ 2 g/24 h.

Están"prohibidos"los lECA en el embarazo ya que son teratogénicos.

Creatinina > 1,2 mg/dl, por disminución del aclaramiento de creatinina.

Los diuréticos están contraindicados en el embarazo porque

Oliguria :,; 500 m i en 24 horas.

dismin uyen el volumen p lasmático y el flujo útero-placentario.

Plaquetas< 100.000 o anem ia hemolítica microangiopática.

Diazóxido: aunq ue es un potente hipotensor, no se recom ien-

Elevación de enzimas hepáticas (GOT > 70 U/1 y LDH > 600 U/1), dolor

da en el embarazo por sus efectos secundarios en la madre y

epigástrico o vóm itos.

por sus efectos te ratógenos.

Cefa lea o altera ciones visua les.

Atenolol (bloquea nte de los receptores ~-adre n érg i cos): aunque

Hemorragia retiniana, exudado en fo ndo de ojo o papiledema.

es una de los prin cipales fármacos frente a la hipertensión, su uso

Edema pulmonar.

durante el embarazo presenta unos efectos indeseables co mo su

Sínd rome HELLP: hemóli sis, elevación de las enzimas hepáticas, pla-

acción estimulante del músculo uterino y que puede comprome-

quetopenia.

ter el intercambio materno-fetal, favoreciendo la aparición deCIR.

Manual CTO de Medicina y Cirugía, 2 .a edición

Anticonvulsivantes: el tratamiento de elección en la profilaxis y el tratamiento de las convulsiones es el sulfato de magnesio. Es un fármaco

tores tales como la presentac ión, las cond iciones cervicales, la edad gestacional, etcétera . Se puede inducir el parto, así como emplear

que disminuye la excitabilidad del sistema nervioso central y es un buen

prost ag land inas locales para la mad urac ión ce rvica l. La anestesia re-

anticonvulsivante. Además tiene una ligera acción hipotensora. Su ac-

g iona l co nsigue un buen control de la hipertensión y mejora el flujo

ción se produce por disminución de la secreción de acetilcoli na a nivel

útero-placentario, por lo que puede co nsiderarse la técn ica de elec-

de la unión neuromuscu lar y al mismo tiempo, reduce la sensibilidad de

ción en estas pacientes siemp re que no exista n contraind icac iones

la placa motora a la acetilcol ina. Su vía de ad ministración es endoveno-

para la mi sma .

sa. Su principal problema es el estrecho margen tóxico-terapéutico por lo que se deberán controlar los signos de intoxicación materna; para

Se practica rá cesá rea sólo si no prog resa adecuadamente la dilatación,

ello, se rea lizarán controles del reflejo rotu liano (su abolición es un signo precoz de intoxicación), de frecuencia respiratoria (menos de 14 respira-

ante sospecha de sufrimiento feta l, em peora miento del est ado materno o mal control de la gesta nte.

ciones por minuto), de diuresis horaria y de tensión arterial. También se monitorizarán los niveles de magnesio en sangre que deben estar entre

Se term in ará la gestación en todas aquel las pacientes a término con

4,8 y 9,6 mg/dl. En caso de toxicidad ag uda, su antídoto es el gluconato

preec lampsia. En las gestaciones pretérmino, se decidirá en func ión de

de calcio. En el tratamiento de las convulsiones agudas, también es po-

la gravedad del cuadro clínico que presente la embarazada y del estado

sible emplear benzodiacepinas. En las pacientes con factores de riesgo de preeclampsia, aunque

fetal.

no existe acuerd o general, se puede ad m inistrar ácido acetil sa licílico a bajas dosis (1 00 mg/ día), desde la sema na 12 de gestación hasta el fin al de la misma, para red ucir la incidencia de la preeclampsia y las comp li cac iones de ésta.

No se debe suspender el tratamiento después del parto porque pueden reaparecer los síntomas.

32.6. Parto

Pronóstico materno

En genera l, se acepta que la vía vag inal es preferible a la cesárea . Para la elecc ió n de la vía de parto, habrá que tene r en cuenta fac-

La enfermedad no siempre recidiva en embarazos posteriores, pero produce predisposición pa ra la HTA crón ica. Es, actualmente, la tercera causa de mortalidad materna.

Ideas clave "

"

~

La etiología de la preeclampsia es desconocida. En las mujeres con factores predisponentes, se produce una alteración inmunitaria que produce isquemia y liberación de factores vasoconstrictores como el tromboxa no, provocando una lesión endotelial diseminada que da lugar a los diferentes síntomas y signos de la enfermedad. Estos factores producen una vasoconstricción generalizada, ocasionando una elevación de la tensión arterial. La alteración a nivel glomerular típica es la endoteliosis glomerular, que produce proteinuria. Estos son los dos requisitos para realizar el diagnóstico de preeclampsia.

Casos clínicos Gestante de 26 semanas de amenorrea que presenta, desde hace cinco días, malestar general, astenia, náuseas, cefalea, edemas y ligero dolor en hipocondrio derecho. En la analítica practicada presenta: Hb 8 g/dL, bilirrubina 1,4 mg/dL, LDH 670 UI!L, AST 182 UI/L, plaquetas 80.000/mm 3 • Lo más probable es que se trate de un embarazo complicado por: 1) Muerte fetal con paso de sustancias tromboplásticas a la circulación materna. 2) Desprendimiento prematuro de placenta superior al 50%.

32 · Estados hipertensivos del embarazo

"

El síndrome de HELLP es la asociación de hemólisis, elevación de las enzimas hepáticas y plaquetopenia, que puede aparecer en mujeres con preeclampsia y supone una complicación grave.

"

Existen unos criterios de gravedad, tanto clínicos como analíticos, que es necesario estudiar, que permiten clasificar la preeclampsia en leve o grave, y que ayudarán a decidir la conducta obstétrica en cada caso.

"

El tratamiento de elección es la finalización de la gestación, y será la conducta a seguir en todas las gestantes a término. Sin embargo, en gestaciones pretérmino, será preciso el tratamiento médico antihipertensivo para lograr controlar la enfermedad y prolongar el embarazo hasta al menos haber alcanzado la madurez pulmonar. Esto será posible siempre que no se esté frente a un cuadro de preeclampsia grave.

3) lsoinmunización eritrocitaria con transfusión feto-materna. 4) Preeclampsia severa. RC: 4 Se hospitaliza a una gestant e de 35 semanas por presentar presión arterial de 165/100 mmHg en repetidas tomas, edemas progresivos y proteinuria significativa. Se instaura tratamiento con hidralacina y sulfato de magnesio. Seis horas después, la paciente refiere cefalea intensa, epigastralgia y fotofobia. Se objetiva oliguria, presión arterial de 180/120 mmHg, aumento de los edemas, feto en presentación cefálica con cérvix formado y cerrado. La monitorización fetal no estresante presenta una línea de base

Ginecología y obstetricia

1

32

de 140 latidos/minuto, ritmo silente, ausencia de aceleraciones de la frecuencia cardíaca fetal y desaceleraciones tardías con cada contracción . ¿Cuál es la conducta correcta?

3) Determinar la madurez pulmonar fetal mediante estudio del líquido amniótico. 4) Finalizar la gestación mediante cesárea.

1) Realizar un perfil biofísico fetal. 2) Aumentar las dosis de antihipertensivos y anticonvulsivantes hasta controlar el cuadro y realizar una inducción del parto.

RC: 4

Case Study A 29-year-old woman, gravida 1, para O, presented at 31 weeks gestation with a two-day history of headache and epigastric pain. On examination, maternal blood pressure was 180/120 mmHg and pulse was normal. Full blood count had shown decreased hemoglobin (1 0.1 g/dl) and hematocrit (32%), as well as thrombocytopenia (95,000/J.IL); her coagulation screen was normal. The most likely treatment is:

1) 2) 3) 4)

Alpha-methyldopa. Labetalol. Nifedipine. Magnesium sulphate.

Correct answer: 2

Diabetes gestacio nal

Es un tema a medías con la asignatura de Pediatría. Se debe aprovechar para estudiar, sobre todo, el diagnóstico de la diabetes durante el embarazo.

Cambios en la gluconeogénesis: el feto emplea ciertos aminoácidos, privando a la mad re de los principales sustratos para la gluconeogénesis. Así, en la gestación, se distinguen las siguientes etapas: l . Periodo precoz, anabólico: Hay un aumento de los depósitos grasos. Hipog lucemia materna (la glucosa se está "guardando" y por eso se retira de la circulación).

33.1. Diabetes y gestación

Disminución de la insu li na y de sus necesidades en las pacientes con DM pregestaciona l (a l haber hipoglucemia en este periodo, se reduce la necesidad de insu li na en estas mujeres).

Uno de los cambios metabólicos más importantes en la mujer gestante es el relacionado con los hidratos de carbono, de forma que el 1-3% de

2.

Segunda mitad de la gestación, catabolismo:

todas las gestantes muestran algún grado de intolerancia a la glucosa.

Lipólisis: aumentan los ácidos grasos libres metabolizados a cuerpos cetónicos.

La mayoría de estas pacientes son mujeres con predisposición genética o metabólica a la diabetes, incapaces de compensar de forma adecuada los efectos diabetógenos del embarazo. Se denom ina diabetes gestacional

Resistencia a la insulina: hiperglucemia, incremento de las moléculas de insulina y de sus necesidades en la DM pregestaciona l (a l ser este periodo hiperg lucém ico, se precisa más insuli na compensatoria).

a aquel la d iabetes que se diagnostica por primera vez durante la gestación, independ ientemente de la gravedad, del requerim iento insu lín ico o de la persistencia posparto.

3.

Posparto: d ismin ució n de las necesidades de in su lina, ya que se norma liza la hiperg lucemia.

El grupo de pacientes que estaban diagnosticadas de diabetes mellitus (DM) previamente a la gestación se clasifican como diabetes pregestacional.

33.3. Morbilidad materna 33.2. Efecto diabetogénico del embarazo Resistencia a la insulina: re lacionada fu ndame nta lmente con el lactógeno placen tari o (HPL) de la segunda m itad de la gestación.

La diabetes mell itus durante el embarazo es un factor de riesgo para el desarrollo de compl icaciones en la madre y en el feto. La morbilidad materna viene definida por el aumento de los abortos espontáneos, hidramnios (25%), hemorrag ias posparto, preeclampsia (5- 15%), infecc io nes (ITU, candid iasis, corioamnionit is y endometritis posparto), cesáreas, progresión de

Este disminuye la uti lización periférica de la glucosa, provocando una hiperglucemia y un hiperinsulinismo compensatorio. Además, aumenta la producción de cortisol, estriol, progesterona y de otras

la retinopatía, nefropatía y miocard iopatía en pacientes diabéticas.

hormonas contrainsulares y se produce un aumento de la degradación placentaria y renal de la insulina. Aumento de la lipólisis: la madre usa la grasa para cubrir sus necesidades calóricas y guardar así la glucosa pa ra cubrir las del feto. La lipólisis produce ácidos grasos li bres que no atraviesan la placenta,

33.4. Mortalidad perinatal

aunque sí lo hacen el glicerol y los cuerpos cetón icos.

Ocurre en aproximadamente el 4% de los partos, fundamentalmente está relacionada con malformaciones congénitas (la causa más frecuente), prematuridad, insuficiencia respiratoria, infeccio nes y t raumatismos obstétricos.

Ginecología y obstetricia

33.5. Morbilidad fetal

(Figura 77l

1

33

formación más frecuente en los hijos de madre diabética), transposición de los grandes vasos, CIV, CIA, DAP. etcétera . Genitourinario: agenesia renal, dupl icación ureteral, etcétera. Gastrointestinales: hipoplasia de colon izquierdo, atresia ana l.

Distrés respiratorio

33.6. Diagnóstico (Figuras 78 y 79) Cardiomegalia

Para la detección de la diabetes gestacional se debe realizar una sobreca rga de 50 gramos de g lu cosa a todas la s gestantes entre la semana 24 y 28 de emba razo. Se considerará positiva con valores por encima de 130 mg/d l y d iagnóstica con un valor mayor de 180 mg/ dl (170 mg/d l en mujeres mayores de 30 años).

Co lon izquierdo hipoplás ico

Trombosis de la vena re nal

Las gestantes de alto riesgo de diabetes gestacional (obesidad severa, antecedentes de diabetes en familia res de primer grado, diabetes gestaciona l o intolera ncia a la glucosa en embarazo previo, antecedentes de

?1:----7"~"---- Síndrome

de regresión caudal

macrosomía o presentar g lucosuria en el momento actual) realizarán la prueba de detección después de las 12-14 semanas. La cu rva de tolerancia a la glucosa se puede realizar con 100 g (valores de referenc ia: 95, 180, 155 y 140 mg/d l) o con 75 g (valores de referencia: 95, 180, 155 mg/dl). Se realizará a las gestantes co n una sobreca rga de 50 g positiva, a las pacientes con diabetes gestacional en un embarazo previo y en gestantes de riesgo (tener 3 o más factores de riesgo). Se

Hipoca lcemia hipoglucemia

Figura 77. Morbilidad fetal en diabetes gestacional Durante el periodo embrionario, aparecen complicaciones como las ma lformaciones o los abort os. En el perio-

> 130 mg/ dl (-)

do fetal, puede altera rse la madurez pulmonar, lo que puede producir

> 130 mg/ dl

insuficiencia resp iratoria en el recién nacido. También aumenta la inciden-

24-28Sem

cia de muerte fetal intrauterina y las alteraciones en el crecimiento, tanto macrosomía como el caso contrario, CIR. Los fetos macrosóm icos tienen riesgo de distoc ia en el parto y, en consecuencia, de sufrir tra umatismos obstétricos. Durante el parto, crece la incidencia de rotura prematura de membranas, el parto pretérmino y el prolapso de cordón. En el neonato apa recen alteraciones metabólicas: hipoglucem ia (es la ma nifest ación

:2:2 valor alterado

> 180 mg/dl

Diabetes gestacional

Figura 78. Pruebas diagnósticas de diabetes gestaciona l

- 105 mg/ dl

patológ ica más frecuente en el hijo de madre diabética). También puede presentar hipoca lcem ia, hiperbilirru-

Test deO'Sullivan

!

Nuevo control

t -+---

- - - - - - N o r m al

SNC: anencefalia, holoprosencefalia, encefalocele, etcétera. hipertrofia Cardiovasculares: del tabiqu e interventricular (ma l-

-

-

145 mg/ dl

165 mg/dl

190 mg/ dl

< 140 mg/ dl

binemia o policitemia. Las ma lformaciones congénitas que pueden aparecer son las sig uientes: Esqueléticas: la más ca racterística es el síndrome de regresión caudal.

Intolerancia a los hidratos de carbono

1 valor alterado

Figura 79. Diagnóstico de diabetes gestacional

:2: 140 mg/ dl

~

mg/ dl 105 190 165 145

Manual CTO de Medicina y Cirugía, 2.a edición

rea liza el diagnóstico de d iabetes gestaciona l cuando dos o más de las determinaciones son patológicos. En caso de tener un solo va lor alterado

de 0,3- 1,5 Ul/kg y se ajusta dependiendo de los niveles g lucém icos, el peso de la paciente, las semanas de embarazo o las enfermedades

se diagnostica into lerancia a los hidratos de carbono.

que puedan modificar la glucemia (emesis). La dosis calculada se divide en 2/3 en ayunas y 1/3 antes de la cena.

En resumen, existen cuatro formas de rea lizar el diagnóstico de diabetes

La Hb A 1e informa del metabolismo en las últimas 8-12 semanas. Ti ene va lor pronósti co para las ma lformaciones.

gestaciona l: Glucem ia en ayuno mayor o igua l a 126 mg/dl en dos ocasio nes. Glucem ia casua l mayor de 200 mg/d l. Sobrecarga de 50 g con re sultado mayor o igual a 180 mg/dl. Curva de tole rancia a la glucosa con 100 g o 75 g con dos o más va lores alterados.

33.7. Control durante la gestación El objetivo terapéutico es mantener la g lucemia en ayuno menor o igua l a 95 mg/dl y menor de 120 mg/ dL dos horas postprand ri ales. Si el peso fetal es mayor de la percentil 90 el objetivo es una g lucemia menor a 80

Finalización del embarazo: con un control metabólico correcto y vigilancia adecuada del bienestar fetal se debe dejar evolucionar la gestación hasta el inicio espontá neo del parto. Si en la semana 40 el parto no se ha iniciado, se va lora rá la fina lización del embarazo. En principio, se intenta que el pa rto sea vag inal. Las indicaciones de cesá rea son las mismas que en la gesta nte no d iabética. En las pacientes con retinopatía proliferativa grave está indicado abreviar el periodo expulsivo, mediante cesá rea o mediante parto instrumental, para evitar un desprendimiento de retina. Esta conveniencia se aplica también en el caso de miopía magna y de otras patologías oculares con riesgo de desprendimiento de retina, independientemente de que la madre sea diabética

mg/d l en ayuno y 11 O mg/d l dos horas tras las com idas.

o no.

El control durante la gestación se realiza de las sigu ientes formas: Perfiles glucémicos: se instruye a la paciente en el autocontrol g lucémico pre y posprandial.

Control metabólico intraparto: se realizan controles glucémicos horarios que deben mantenerse entre 80 y 120 mg/d l, comenzando la

Dieta restrictiva: evitar el azúcar refi nado (pa steles, ca ramelos, refrescos, et c.) y rea lizar ej ercicio físico. Insulina: está indicada cuando no se cons igue un adecuado control glucémico a pesar de la d ieta y del ejercicio. También se usa en caso

infusi ó n de insul ina y dextrosa intravenosa cuando no se consiga. Puerperio: se mantiene el suero glucosado al 10% hasta reanudar la alimentación y se suspende la insul ina en las diabéticas gestacionales que re-

de macrosomía o polihidramn ios fetal, siendo discutido su empleo de manera preventiva. La pauta insu lín ica co nsiste en un mínimo de dos aplicaciones al día, combinando una insulina de acció n rápida e

quirieron menos de 20 Ul al día. Se deberá reclasificar a todas las pacientes que cursaron con diabetes gestacional o intolerancia a la g lucosa a partir de las 6 semanas postparto con una pru eba de to lerancia a la glucosa con 75 gramos. Si el resu ltado es normal se repetirá a los 3 años y si indica into-

intermedia media ho ra antes del desayuno y ce na. La dosis inicial es

lera ncia a los hidratos de carbono se repetirá al año.

Vigilancia en la diabética gestacional Primera visita

Historia clínica.

28 semanas

Control glucémico Pregestacional: hemoglobina glucosilada, función renal y va loración oftalmológica, perfil tiroideo si diabetes tipo 1 Semana 7-9

Ecografia: vita lidad y edad gestacional

Semana 11-14

Ecografía 11-14 SDG: marcadores cromosomopatías

Semana 16-20

Va loración oftalmológica, si retinopatía pre-existente

Ecografía obstétrica Va loración oftalmológica

32 semanas

RCTGE. Ecografía obstétrica Va lorar: perfi l biofísico y/o Doppler

36 semanas

RCTGE. Ecografía obstétrica Va lorar: perfi l biofísico y/o Doppler

Marcadores bioquímicos (PAPP- A, PHGC)

Orientar sobre tipo de parto 30-40 semanas

Marcadores bioquímicos (AFP, uE3, inhibina A, hGC)

Pruebas de bienestar fetal Inducción del parto o cesá rea según las cond iciones obstétricas

Ecografía morfológica sema na 20 Tabla 34. Vigilancia en la diabética gestaciona l

ciadas. Cuando esté por debajo del 7%, la gestante diabética tiene el mi smo riesgo de malformaciones que si no lo fuera .

Ideas clave Jii5 "

Las complicaciones que se asocian con la diabetes en el periodo embrionario so n una mayor tasa de abortos y de malformaciones.

"

"

"

La malformación más frecuente asociada a la diabetes es la hipertrofia del tabique card íaco, mientras que la más característica es el sínd rome de regresión caudal.

Las compl icaciones que se asocian durante el segundo y el tercer trimestre se re lacionan con las alteraciones del crecimiento (macrosomía fetal), alteraciones de la madurez pulmonar (mayor incidencia de enfermedad de membrana hialina) y con problema s en el momento del parto por el riesgo de distocia por la macrosomía.

"

Se debe realizar una sobrecarga de 50 g a todas las gestantes entre la 24-28 semana.

La determinación de hemoglobina glicosilada en el primer trimestre nos puede predecir el riesgo de malformaciones aso-

"

Si resulta> 130 mg / dl se realizará sobrecarga de 100 g o 75 g.

33 · Diabetes gestacional

Ginecología y obstetricia 11 Si resulta > 180 mg/d l se diagnosticará diabetes gestacional. Tambi én cuando se obtengan 2 valores patológicos en las sobrecargas de 100 g o 50 g. Si sólo se obtiene 1 valor alterado se trata de una intolerancia a los hidratos de carbono.

Case Study . A 24-year-old woman, gravida 2, para 1, at 25 weeks gestation with the following glucose test: 87/175/159/145. The most likely diagnosis is:

1

33

11 El tratamiento de la diabetes durante el embarazo es con dieta, ejercicio físico e insulina. Los antidiab éticos orales están contraindicados.

1) 2) 3) 4)

Glucose test is normal. Glucose intolerance. Gestational diabetes. Pregestational diabetes.

Correct answer: 3



Ginecología y obstetricia

Complicaciones infecciosas

O R 1 E r~ T A C 1 Ó N

ENARM

Es un tema poco importante. Hay que estudiarse las Ideas Clave.

34.1. Vacunaciones Las vacunas que están contraindicadas durante la gestación son las siguientes: pa rotiditis, rubéola, sarampión, fiebre amarilla (son virus vivos atenuados). No se recomienda la vacunación sistemática contra: gripe, poliomielitis, hepatitis B, aunque son vacunas permitidas durante el embarazo, si fueran necesarias. La vacunación frente a la fiebre tifoidea está perm itida, siempre que esté absolutamente ind icada. La de la rabia, la d ifte ria y el tétanos sí deben administrarse cuando estén ind icadas (son toxoides), al ig ual que la tos ferina (células muertas).

Diagnóstico El diagnóstico se rea liza med iant e serolog ía en la primera visita prenatal. Son ind icativos de infección activa una lgG creciente o una lgM positiva. El diagnóstico de la infección fetal se rea liza a partir de líquido amniótico o de sangre fetal, determinando lgM, PCR o cultivos. La presencia de hidrocefalia o de calcificaciones cerebrales dispersas son datos ecográficos que sugieren la presencia de infección fetal.

Profilaxis Las medidas primarias que se recom iendan para evitar la infección en las gestantes susceptibles son las siguientes: evitar tocar ojos y boca sin lavarse las manos después de haber manipulado carne cruda, evitar la ingesta de carne poco cocida, el contacto con animales reservorio (gatos, conejos, gallinas, pa lomas y perros), asegurarse de lavar bien frutas y verduras antes de consum irlas, ponerse guantes si hay que rea lizar trabajos de jard inería, etc.

Tratamiento

34.2. To X o p1a S m o Si S Se trata de una infección inocua en el adu lto, que con frecuencia pasa asintomática. Puede aparecer infección fetal por vía t ransplacentaria como consecuencia de primoinfección padecida por la madre durante el embarazo. El riesgo del feto está en relación con el momento en que ocurre la infección materna. Así, se ca lcula que el riesgo de infección fetal

Las gestantes con toxoplasmosis durante el embarazo recib irán tratamiento con espiramic ina y si existe infección fetal, se añad irán pirimetam ina y su lfad iazina en ciclos de tres semanas, alterna ndo con espiramicina hasta el fina l de la gestación. Se recomienda la suplementación con ácido folínico para prevenir la depresión medular producida por la pirimetamina.

34.3. Rubéola

alcanza el15, 30 y 60% según tenga lugar en el primero, segundo y tercer trimestre de la gestación respectivamente. Sin embargo, la gravedad de la enfermedad, de transmitirse, es mucho mayor si ocurre en las primeras 12 semanas, pues se produce un alto porcentaje de abortos o embriopatías graves. La infección en la embarazada se caracteriza por la aparición de astenia y de anorexia, acompañadas de adenopatías cervica les y cuadro faringoamigda lar, q ue puede simu lar una mononucleos is infecciosa . La cl ínica en el feto se caracteriza por coriorretin itis (más frecuente), hidrocefalia, calcificaciones cerebrales y convulsiones (tétrada de Sabin), aunque la clínica es muy variable y es posible que aparezcan formas sin toda la sintomatología.

La infección congénita por rubéo la ha d ism inu ido drásticamente por la vacunación genera lizada -mediante la triple vírica- de la población femenina en la pubertad. La rubéola puede provocar graves alteraciones fetales, dependiendo de la época de la gestación en la que tenga lugar el contagio. Cuando la mad re se contag ia dent ro de las pri meras ocho semanas, la infección fetal supera el 80%, siendo la sordera neurosensorial el defecto más común encontrado en los niños que se infectaron entre la novena a la decimotercera semana. Después de esta fecha, las malformaciones graves son menos frecuentes.

Ginecología y obstetric ia Diagnóstico

1

34

para el diagnóstico específico, ya que el embarazo es la primera causa de fa lso positivo.

Las pruebas de laboratorio para el diagnóstico de la rubéola son determinación de anticuerpos lgM

e lgG, ais lamiento del

virus en fosas nasales,

sangre, garganta, orina o líquido cefalorraquídeo o detección del virus

FTA-Abs o MHA-TP: son pruebas treponém icas. Se realizarán en caso de positivo en las no treponém icas. Proporcionan un diagnóst ico específico.

por RT-PCR.

Tratamiento Las pruebas diagnósticas en el feto pa ra conocer su afectación o no se basarán en el aislamiento del virus y/o RT-PCR a partir de muestras de

El antibiótico de elección es la pen icilina . El tratam iento antes de las 16

ve llosidades coriónicas obtenidas por biopsia entre la semana 1Oy 12 de

semanas evita la sífilis congénita. Después de esta fecha, cura la infección

gestación, líquido amniótico obtenido a partir de amniocentesis entre la

pero no evita los estigmas congénitos. En casos de alerg ia, se puede utili-

semana 14 y 16 de embarazo o sangre, la sangre se obtiene por cordo-

zar la eritromicina, si bien con este fármaco se tienen unas tasas de cura-

centesis entre la sema na 18 y 20 de gestación .

ción mucho menores por lo que es aconsejab le insistir en el tratamiento con pen icilina (puede ser preciso rea lizar desensibil ización, preferible incluso a emplear el antibiótico de segunda elección) .

34.4. Citomegalovirus 34.6. Tuberculosis y embarazo El citomegalovirus (CMV) es una de las virias is suscepti bles de ser contraídas por la madre durante la gestación y de afectar al feto. En la madre, la infección suele ser asintomática. Aunque un importante porcentaje de las

Aunque la tuberculosis en la embarazada es poco habitual, su incidencia

embarazadas son susceptibles a la infección al iniciar la gestación, sólo un

está aumentando. Los dos factores que t ienen mayor influencia en este

3% contraen la primoinfección, y de estas ún icamente en un 50% de los ca-

incremento son la asociación de la enfermedad al VIH y la aparic ión de

sos se produce la infección fetal. Si la primoi nfección t iene lugar durante el

cepas resistentes, a veces a mú ltiples fármacos. Cursa como tuberculosis

primer trimestre, los órganos afectos pueden ser: SNC corazón, ojos y oído.

en pacientes con inmunodepresión, por lo que su pronóstico es peor. El tratamiento recomendado actualmente es: isoniacida junto a piridoxina

La infección durante el segundo trimestre puede provocar microcefalia,

+ rifampic ina + etambutol o espiramicina, evitando siempre la estrepto-

hepatoesplenomega lia, CID o ictericia .

micina, que podría producir sordera congénita .

Diagnóstico

La profi laxis antituberculosa con isoniacida puede realizarse en aquellos casos en los que esté indicada .

Ante la sospecha de enfermedad materna, debe sol icitarse serología de CMV. Un va lor elevado de lgM es d iagnóstico de infección activa primaria o recurrente. Para el diagnóstico de infección fetal, es mucho más fiable

34.7. Varicela

la demostración del agente en el líquido amniótico que el estudio de la sangre fetal. La infección por varicela durante el emba razo tiene una baj a inciden-

Tratamiento

cia. No hay constancia de que el cuadro clíni co sea más grave en la

No existen medidas profilácticas ni terapéuticas específicas.

varicelosa .

34.5. Sífilis

tadora de inmun idad residual y se le informa de la ausencia de riesgos

mujer gestante que en la no gestante, excepto si apa rece neumonía

Ante un contacto, si la gestante presenta lgG en ausencia de lg M, es porpara el feto. Si la varicela se produce antes de las 20 semanas, el ri esgo para el feto de defectos congén itos por el virus está alrededor del 2%. El La sífilis congénita aunque tiene una preva lencia baja, con una tendencia

virus es eminentemente dermotropo y neurotropo. Si aparece después

claramente descendente en las últ imas décadas, sigue d iagnosticándose en

de las 20 semanas, el riesgo de defectos congénitos es prácticamente

la actualidad. Están especia lmente expuestas al contag io las gestantes jóve-

inexistente.

nes drogadictas, de bajo nivel socioeconómico y con promiscuidad sexual.

Tratamiento Las mujeres con más probabilidad de tener hijos afectos son las no tratadas con sífi lis primaria, secundaria o latencia precoz. Ya q ue en muchos casos las mujeres afectadas no presentan manifestaciones visibles, es

El tratamiento con aciclovi r (800 mg, cinco veces al día, durante cinco o siete días) está indicado si: hay desarrollo fulminante, varice la hemorrági-

preceptivo realizar una prueba serológ ica (no treponémica) a todas las

ca, fiebre alta, neumonía varicelosa o herpes zóster.

gestantes. La administración de gammag lobu lina dentro de los cuatro días que si-

Diagnóstico

g uen a un posible contag io es de dudosa eficacia. La única indicación actual de administración es si el cuadro aparece en las tres semanas pre-

VDRL o RPR: son pruebas no treponémicas, inespecíficas, válidas para el cribado (q ue es obligatorio en cua lquier gestante), pero no

vias al parto para evitar la va ricela neonatal, que presenta una elevada mortalidad.

Manual CTO de Medicina y Cirugía, 2.a edición

34.8. Hepatitis B y hepatitis C La infección materna afecta al hijo en caso de que la madre sea portadora crónica, tenga infección activa durante la gestación o hepatitis crón ica



Factores de riesgo

Administrar PAi *

· Hijo previo con enfermedad por EGB · Bacteriuria por EGB ·Parto antes de las 37 semanas

activa . En madres que sólo son HBsAg posit ivas, el porcentaje de tra nsmisión placentaria es bajo, pero cuan do son positivos HBsAg y HBeAg, el riesgo de transm isión se hace rea lmente alto, siendo del 90%, si la mad re es HBeAg positi va. El riesgo de cronificac ión es muy elevado si se adquiere en el periodo perinatal.

l

Cultivo rectovaginal en 35-37 SG

Se precisa una especia l vigilancia del crecimiento fetal. La contra indicación de la lactancia es controvert ida. Se ll evará a cabo inmunoprofi laxis grave.

Factores de riesgo

La hepatitis C tiene el ri esgo de que un 50% de las pacientes acabe pre-

PAI

No ,.,li,.do, '""'"'"' d"mooddo;

activa y pasiva, pero la gestación no aumenta el riesgo de curso clínico

sen tando una hepatopatía crón ica. Ni la hepatit is aguda por el VHC altera

EGB+

· RPM de 18 h o más · T.' intraparto > 38 oc

----Sí

~--------------~ PAI

el curso del emba razo, ni este influye negativa mente sobre la enfermedad; no obstante, existe un riesgo alto de tran sm isión verti ca l en los casos de hepatitis agu da, hepatitis crónica y si existe coinfección por VIH. NoPAl

* PAI: profi laxis antibiótica intraparto

34.9. Estreptococo B Figura 80. Profi laxis de estreptococo B Forma pa rte de la flora digestiva y coloniza la vag ina del 20-30% de las mujeres, siendo la ca usa más frecuente de infecció n bacteriana neonatal,

34.1 O. Virus del papiloma humano

con una mo rtalidad del1 0-20%. Se prod uce transm isión vertical si co ncurren factores de riesgo como los

La trans misión verti ca l parece limitarse a papilomatosis la ríngea y, rara-

sig uientes:

mente, co ndilomatosis orogenital o co njuntivo. Se pueden destruir las

CIR.

lesio nes co ndilomatosas maternas en el tercer trimestre (a unqu e no se

Prematurid ad.

debe usa r podofi lotoxina ni 5-FU).

Rotura prematura de membranas de m ás de 18 horas antes de l parto.

No se recomienda cesá rea electiva, salvo que los condilomas supongan

Corioamnioniti s.

un obstácu lo mecánico importante o alto ri esgo de sangrado.

Fiebre materna mayor de 38

oc durante el pa rto.

lnmunodeficiencia. Bacteriuria por estreptococo B.

34.11. VI H y embarazo

Asfixia perinatal. Inducción prolongada. Historia de pa rto previo de un lactante infectado por estreptococo B.

Las gestantes con infección por VIH no presentan mayor incidencia de

Numerosos tactos vag inales.

comp licacio nes obst étri cas. Los ma los resultados obstétricos observados son secunda rios a la drogadicción, al tabaquismo, etcétera. Sólo gestan-

La afectación precoz del recién nacido suele da r sepsis g rave y la tardía,

tes en estadios avanzados de la infección parecen tener peores resulta-

meni ng itis purulenta. El cribado de madres portadoras implica un cult ivo

dos. No se han observado malformaciones congé nitas asociadas a la in-

de exudado vag inal y anorrecta l a todas las mujeres embarazadas en la

fección por el VI H.

semana 35 -37. Las gesta ntes infectadas no presenta n una evolució n más rápida de la La prevención de la sepsis neonata l consiste en adm inistrar intraparto

infecc ión durante el embarazo. Los niveles bajos de CD4 son pred ictivos

(desde el com ienzo del parto, hasta el final del expu lsivo) antibioterapia

de la presentación de comp licacio nes infecciosas al igual que fuera del

intravenosa:

emba razo.

Penicili na G 5 millones de Ul, más 2,5 mi llones cada cuatro horas. Amp icilina 2 g i.v. más 1 g/4 horas. En alérg icas: eritromicina 500 mg/6 h o clindam icina 900 mg/8 h.

Se aconseja que las gestantes infectadas reciban tratam iento anti rretroviral durante el embarazo. El tratam iento estándar consiste en la combinación de al menos tres antirretrovirales. Se cons ideran seg uros la zidovudi-

La profi laxis (Figura 80) se hace en pacientes co n cultivo positivo o en las

na, la lam ivudina y la nevirapina. El empleo de indinavir y za lcitabina debe

que, teniendo cultivo negativo o desconocido, cuentan con los factores

restringi rse en lo posible y el efavirenz está en principio proh ibido ya que

de riesgo descritos.

es teratogénico en anima les.

34 · Complicaciones infecciosas

Ginecología y obstetricia Prevención de la transmisión vertical La ta sa de t ransmisión vertica l previa se sitúa en torn o al 20o/o, la mayoría de los casos ocurrirán en las última s semanas de gestación y en el parto. La gestante debe recibir una pauta de AZT o zi dovudina endovenosa y/o 3TC o lamivudina int raparto y al neonato a menos que esté contraindica-

1

34

artificial y rea liza r cesárea si se prevé un parto prolongado. Evitar la realización de microtomas, la colocación de electrodos y los pa rtos instrumentados y contraindica r la lactancia materna en todos los casos.

34.12. Herpes simple

do por toxicidad previa demostrada. Se ha demostrado que la cesárea elect iva (2o/o) d ism inuye de forma significativa el riesgo de transmisión vertica l respecto al pa rto vag inal (1O,So/o). La cesárea debe aconseja rse sin discusión cuando exista deterioro inmunológico (CD4 < 200), carga viral detectable, si no ha recibido tratamiento médico dura nte la gestación, prematuridad y en gestantes con bolsa rota mayor de cuatro horas. Dura nte el pa rto, se debe evitar la amniorrexis

Ideas clave

~

"

Las únicas vacunas que se permiten durante la gestación son los toxoides (tétanos, rabia, difteria), mientras que aquellas de virus vivos atenuados están totalmente contraindicadas (parotiditis, rubéola, sarampión, fiebre amarilla) .

"

La primoinfección por toxoplasma durante el embarazo puede provocar una infección fetal por vía transplacentaria.

"

El tratamiento de la toxoplasmosis es espiramicina y si existe afectación fetal se añadirá su lfadiacina y pirimetamina.

"

La infección congénita por rubéola ha disminuido por la vacunación sistemática. El diagnóstico se realiza mediante la determinación de anticuerpos. La presencia de lgM implica infección reciente. No existe un tratamiento efectivo para la rubéola congénita.

"

La sero logía para sífi lis es obligatoria en el primer trimestre. Se realizarán pruebas no treponém icas y si, estas resultan positivas, se harán pruebas t reponém icas. Si son posit ivas, se procederá con el tratamiento con penici li na lo más precoz posible, ya que si se realiza antes de la semana 16, evita la sífilis congénita.

"

La profi laxis con isoniacida puede rea lizarse en aque llos casos en los que esté indicada.

Casos clínicos Gestante de 24 semanas que acude a la consulta porque a su hijo de 4 años le diagnosticaron hace S días la varicela. la paciente no recuerda si padeció la enfermedad en la infancia, pero sí sabe que no fue vacunada, y está muy preocupada por la posible afectación fetal. ¿Qué actuaci ó n sería la correcta? 1) Solicita r cuantificación de lg G, y si fu era negativo, ad min istrar la gam maglobul ina específica . 2) Tranq uil izar a la paciente, informándole de la ausencia de riesgos fetales. 3) Admini strar aciclovir oral en dosis de 800 mg, S veces al día, d urante 5-7 días. 4) Admini strar gammaglobulina es pecífica y tra nq ui liza r a la madre, informán dole de la ausencia de riesg os fetales.

La transmisión de la infección al feto y al recién nacido ocurre por contacto directo con material infectado y tiene lugar en el momento del parto. La presencia de lesiones activas o el antecedente de infección activa en el tercer trimestre es indicación de cesá rea electiva para reducir el riesgo de infección neonata l.

"

En un caso de varicela durante la gestación, se admin istrará aciclovir si hay: desarrollo fu lminante, varicela hemorrágica, fiebre alta, neumonía varicelosa o herpes zóster. La adm inistración de gammaglobulina no está indicada, salvo que el cuadro aparezca en las dos o tres semanas previas al parto, y se utiliza para disminuir la gravedad de la varicela neonatal.

"

En las pacientes con VIH, es necesario el tratamiento durante el embarazo para disminuir la transmisión vertical. Se utilizará pauta de triple terap ia como en pacientes no gestantes, evitando los fármacos que ti enen demostrada teratoge nia (efavirenz y delarvidina).

"

Estará indicada la extracción fetal med iante cesárea si existe: deterioro inmunológico (CD4 < 200), carga viral detectable, no tratamiento antirretroviral durante el embarazo, prematuridad y rotura prematura > 4 horas.

"

Se recomienda real izar a todas las gestantes al final del embarazo (35-36 semanas) un cultivo vagina l y otro anorrectal para detectar a las mujeres portadoras del estreptococo B y pautar profilaxis antibiótica intraparto con penicilina o ampicilina. En las mujeres con cultivo negativo en las que existen otros factores de riesgo de seps is precoz (hijo previo con enfermedad por SGB, bacteriuria por SGB, prematuridad, rotura de membranas > 18 h, fiebre intraparto), también está indicado el tratamiento antibiótico.

Una gestante de 1O semanas tiene contacto con un niño que, 6 días más tarde, desarrolla un cuadro de exantema y síndrome general infeccioso sugerente de infección por virus de rubéola. En el primer control serológico gestacional, se detectó la negatividad de la lgG específica. ¿Cuál de las siguientes afirmaciones es la correcta? 1) La apa rición de lgM matern a positiva co nstituye indicaci ón para la determ inaci ón de la lg M fetal. 2) No existe pos ibilidad de contagio, dado q ue el ni ño ya no se ha llaba en fase de elim inació n viral. 3) En este periodo de la gestaci ón, el riesgo de la infecci ón y de afectación em brionaria es m íni mo. 4) La adm ini st rac ión inmed iata de ga mmaglobulina hiperi nmune específica reduce considerablemente el riesgo de infecci ón embrio-fetal.

RC: 1 RC: 2

Manual CTO de Medicina y Cirugía, 2. a edición

Case Study A 32-year-old woman, gravida 2, para 1, was admitted to hospital at 32 weeks gestation for fetal growth restriction. Ultrasound demonstrated fetal growth restriction, splenomegaly, microcephaly, lissencephaly and the presence of periventricular microcalcifications. The most likely diagnosis is:

34 · Complicaciones infecciosas

1) 2) 3) 4)

Congenital Congenital Congenital Congenital

toxoplasmosis. rubella. varicella. cytomegalovirus.

Correct answer: 4

Ginecología y obstetricia



Fármacos y embarazo

Categoría A: ausencia de riesgo fetal demostrada en humanos (por

ORIENTACIÓN

ENARM

Tema poco relevante.

ej emplo, ácido fó lico). Categoría B: bien to lerados en estudios anima les, pero no hay estu-

d ios en seres hu manos (po r ejemplo, pen icilina). Categoría C: ca rencia de estud ios adecuados tanto en humanos

El embarazo supone la aparición de modificaciones farmacoc inéticas que

como en an imales (por ejemplo, paroxetina).

pueden alterar la respuesta a los fármacos. Además, se introduce un nue-

Categoría D: demostrado el riesgo fetal. En determinadas ocasiones

vo factor, la presencia del feto, que puede verse afectada por los fármacos

se pueden usar si la relación riesgo/beneficio lo aconseja (por ejem-

que toma la mad re. Las modificaciones farmacocinéticas en el embarazo

plo, fenitoína).

se resumen en la Tabla 35 .

Categoría X: los riesgos son tan claramente superiores a los benefi-

cios por lo que no deben utilizarse durante la gestación (por ejemplo, estradiol).

35.1. Efectos de los fármacos sobre el feto

Algunos eJemplos de efectos teratogénicos típicos son los siguientes: Talidomida, tolbutamida y clorpropamida: producen malforma-

ciones en extremidades, pabellón auricular, ojo y vísceras. Según la fase de la gestación en que se tomen los medicamen tos

Estreptomicina: provoca lesión del VIII par, m icrom ielia, anomalías

teratogén icos, los efectos serán difere ntes. Así, en la fase de game-

esqueléticas.

togénesis, se producirá esteril idad; en la de segmentación, aborto;

Tetraciclinas: inhibición del crec imiento óseo, manchas dentarias,

en la organogénesis (embrión), ma lformaciones; y en la fase de feto,

m icromielia, sindactilia .

alteraciones func iona les tras el naci mie nto, como por ejemp lo, h ipo-

Yoduros: bocio congénit o, hipotiroidismo, retraso mental.

g lu cem ia con antidiabét icos o rales, hi pot on ía y depresión respi rato-

Cloroquina: lesiones retinianas, lesiones d el VIII par.

ria con benzodiacep in as o comp licac iones hemorrágicas con AAS

Metotrexato: anomalías congénitas mú ltiples. Incluso las dosis bajas que

y Al NE.

toman algunas pacientes con artritis reumatoide pueden provocarlas. Anfetaminas: transposición de grandes vasos, fisura palatina .

El riesgo de toxic idad fetal de los fármacos se clasifica en las siguientes

Dietilestilbestrol: adenocarcinoma de células claras en ovario y cér-

categorías:

vix, anomalías del tracto genital por persistencia mülleriana.

Proceso cinético

Absorción

Distribución

Cambios fisiológicos dependientes de la edad gestacional

,J,. vaciado gástri co

Retraso de absorción

,J,. motilidad gastrointestinal

,J,.cmáx

t flujo sanguíneo en piel, mucosas y músculo

t biodisponibil idad tópica

Eliminación renal

e intramuscular

tagua corpora l total

t vo lumen de distribució n de fármacos lipofíl icos

,J,. grasa subcutánea

t volume n de distribución y de la fracción de fármaco no un ido a proteínas

,J,. co ncentración de proteínas Metabolismo

Cambios farmacocinéticos secundarios

t o ,J,. capacidad enzimática

Poca afectación en los antimicrobianos

Colestasis

,J,. eliminación de rifampic ina

t filt ración glomeru lar y flujo sanguíneo

t depuración renal y ,J,. vida med ia de el iminación

=función tubu lar Tabla 35. Modificaciones farmacocinéticas en el embarazo

Manual CTO de Medicina y Cirugía, 2.a edición

Etanol: síndrome alcohó lico del recién nacido.

inadvertidamente estar embarazada. Hay que recordar que daña el

Dicumarínicos y warfa rina: anoma lías esqueléticas y faciales, retra-

cartílago en crecimiento.

so mental.

Antitu berculosos: la isoniacida es el fá rmaco más seguro para el

Podofilino: teratogénico, incluso por vía tópica.

feto, aunque implica un riesgo de hepatitis materna. Se emplea

Aminoglucósidos: lesiones del VIII par.

como profi laxis y como agente terapéutico, junto con el etambutol

Litio: bocio, anomalías oculares, fisura pa lati na.

(del que no hay constancia de toxicidad o teratogenia). La rifampici-

Antitiroideos: bocio, hipotiroidismo, retraso mental.

na es teratogénica en teoría, aunque está indicada en su tratamiento;

Cotrimoxazol (sulfamida): se asocia a ictericia neonatal,

la estreptom icina produciría daños en el VIII par fetal. La pirazinamida puede alterar la coagu lación.

Tiazidas: teratogénicas. Además, los diuréticos están contra indica-

ln hibidores de la ECA (captopril, etc.): malformaciones y muerte fe-

dos en el embarazo.

tal. No deben administrarse durante el embarazo.

AAS: en dosis mín imas de 100 mg/d, se ha usado para la prevención de la preeclampsia y deCIR. No debe ser uti lizado como analgésico.

cer las contracciones.

Misoprostol (prostaglandina): puede producir aborto por favore-

Cloranfenicol: síndrome gris del recién nacido. Nortloxacino, pipemídico: el fa llo más común en la actua lidad es

Difen ilhidantoínas: alteraciones de la coagulación, hasta el 30% de

emplearlos en la infección de orina de la paciente joven, que puede

ciales y digitales, además de un menor cociente intelectual.

Ideas clave "

ker-

nicterus.

~

Entre los fárm acos de uso frecu ente co ntra ind icados en el embarazo, destacan: anticoag ulantes o rales, ta lidomida, retinoi-

35 · Fármacos y embarazo

los fetos expuestos a fenitoína presentan ma lformaciones craneofa-

des, t etraciclina s, ket oconazol, q uinolonas, dietil estil bestrol (DEB) y t iacid as.

Ginecología y obstetricia



Otras patologías de la gestante

Tema poco importante, salvo el apartado de la isoinmunización Rh, que sí hay que estudiar en detalle.

Clínica Consiste en la apa rición de: Prurito generalizado: es el síntoma más precoz de la colestasis intra-

hepática gestacional. Aparece en la segunda m itad de la gestación y aumenta conforme avanza la m isma. Es debido a la acción del exceso de ácidos biliares en la piel y se presenta con predom inio nocturno. Coluria (60% de los casos).

36.1. Hiperemesis gravídica

Ictericia: en los casos más graves (10-20%), en ausencia de otros sín-

tomas hepáticos o gastrointestinales. Consiste en la apa rición de vómitos contin uos e intensos que impiden la alimentación de la embarazada ocasionando su deshidratació n y un

Se comprueba elevación de la isoforma hepática de la fosfata sa alcalina (que podría distingu irse de la placentaria por técnicas isoenzimáticas) y

trastorno metabólico más o menos grave.

del colestero l, incremento de la bilirrubina (hasta 6 mg/dl), leve aumento de los ácidos bil iares séricos y transamina sas normales o re lativamente

Tratamiento

elevadas, con GGT normal o alta (Tabla 36) .

El tratamiento consiste en:

Cua ndo la alteración es leve, se llama prurito del embarazo.

Ingreso hospitalario y reposo: es co nveniente el aislamiento y la

tranqu ilidad de la paciente. Reposición hidroelectrolítica: se rea liza por vía intravenosa, pasán-

dola a vía oral una vez que sea tolerada. Piridoxina (vitamina 8 6 ) : es efectiva en la reducción de la gravedad

Colestasis intrahepática gestacional o ictericia recurrente del embarazo

Hígado graso agudo del embarazo o esteatosis hepática aguda gravídica

de las náuseas.

En la segunda mitad del embarazo

A partir de la semana 35

Doxilamina: es un fármaco antihistam ínico que red uce la frecuencia de las náuseas en el embarazo t em prano aunque producen som-

Prurito

Síntomas de hepatitis aguda

Aumento de las enzimas de colestasis (FA, bil irrub ina) y colesterol

Aumen to de tran sam inasas

Buen pronóstico

Alta mortalidad (de madre y feto)

Recurrente en otros embarazos

No recurrente

Tratamiento méd ico (colestiramina y ursodesoxicólico)

Finalizar la gestación

nolencia. Metoclopramida: se reserva para casos refractari os con vómitos

persistentes.

36.2. Ictericia recurrente del embarazo o colestasis intrahepática gestacional

Tabla 36. Diagnóstico diferencial entre colestasis intrahepática gestacional e hígado graso agudo del embarazo

Tratamiento

Es una enfermedad ocasio nada por una exageraci ón del defecto en la excreción de ácidos biliares fis iológico q ue se observa en alg unas gestantes

El tratamiento co nsiste en administrar ácido ursodesoxicólico y colestiram ina para aliviar el prurito, supl ementar las vitaminas li posolubles A,

y en las tomado ra s de anticonceptivos ora les, probablemente debido a una susceptibilidad aumentada a los estrógenos y progestágenos, aun-

D, K, E si el tratamiento es prolongado y dar vitamina K, si hay prolongación del tiempo de protrombina. También son útiles los antihistamínicos y la dexametasona. Debe mantenerse control fetal periódico (el dato que

que el nivel de estos no suele estar elevado.

Manual CTO de Medicina y Cirugía, 2.a edición

más se asocia con la morta lidad feta l es la cifra de ácidos biliares en sangre materna) e inducir el parto cuando haya madurez feta l. El pronóstico materno es favorab le, pero el feta l puede esta r comprometido, co n morta lidad cercana al So/o. Desapa rece tras el parto, pero

Dermatosis sin riesgo maternofetal: Erupción polimorfa del embarazo: consiste en una erupción cutánea intensamente pruriginosa que se presentan al final del embarazo y no suele recidivar en gestaciones posteriores. Los corti coides, ora les o tópicos, mej oran el pru rito.

Prúrigo gestacional: apa recen lesiones pequeñas y pru riginosas que aumentan a lo la rgo del embarazo, ceden con el parto y no suelen recurrir. No hay cambios en las pruebas de laborato-

recurre en ca da gestación . No evo lu ciona a hígado graso agudo de l embarazo.

rio. El prurito mejora con corticoides tópicos o antihistamínicos

36.3. Hígado graso agudo del embarazo o esteatosis hepática aguda gravídica

ora les.

Hiperpigmentación cutánea: la distensión de la piel en el abdomen y en las mamas puede producir estrías, que suelen perdurar t ras el parto. La línea med ia abdomina l es posible que se hiperpigmente, así como la cara (cloasma) que desaparece tras el parto.

Es una comp licación poco frecuente que presenta riesgo de morta lidad tanto materna como fetal.

Clínica Suele aparecer tardíamente (mayor de 35 semanas) y el riesgo de recurrencia es muy bajo. Los sínto m as in ic iales son ná useas y vómitos, dolor abdomina l, anorex ia e ictericia. El pru rito es raro. La mitad de los casos tienen fiebre, y cas i la m itad, h ipertens ión, prote inuria y edema.

Diagnóstico Se evidencia elevación de las cifras de transaminasas, de bilirrubina y, en los casos graves, disminución del tiempo de protrombina y de la cifra de fibrinógeno.

Tratamiento La esteatosis desaparece ráp idamente tras fina lizar la gestación, pero no se cura nunca antes del pa rto, por lo que la terminación de la gestación es hoy la única actitud vál ida que ha mejorado rad icalmente el pronósti-

Figura 81 . Herpes gestacional

co actua l de la enfermedad.

36.5. Nefropatía gravídica

36.4. Dermatosis del embarazo

Véase Sección de Nefrología.

La dermatosis del emba razo engloba a un grupo muy heterogéneo de procesos dermat ológ icos asociados específi ca mente con la gestación, algu nos de los cua les ti enen alto riesgo feta l.

Dermatosis con alto riesgo maternofetal: Dermatitis autoinmunitaria por progesterona: en todas las gestaciones de la paciente se presentan lesiones cutáneas pustulosas, agrupadas. Hay eosinofil ia y sensibilidad a las pruebas cutáneas de progesterona. Dermatitis papulosa del embarazo: en todas las gestaciones aparecen diariamente pápu \as eritematosas, pruriginosas y diseminadas, y las preexistentes se oscurecen y curan en siete o diez días, dejando hiperpigmentación residua l. Todo cesa tras el alumbramiento. Se cree que la HCG está muy elevada. La prednisona, 50 mg/d, mejora el pronóstico. Herpes gestacional: placas eritematosas pruriginosas con formación de ampol las, que no siempre desaparecen tras el parto (Figura 81).

36 · Otras patologías de la gestante

36.6. Cardiopatías y embarazo Entre e\1-2% de las gestantes padece alg una card iopatía, siendo las más frecuentes las reumáticas, y de ellas, la estenosis mitra\, la más habitual de todas. Dentro de las congén itas, las que tienen mayor incidencia son la comunicación interauricu \ar y la persist encia del conducto arterial. La mortalidad materna está alrededor del 1o/o. El pronóstico fetal también empeora, ten iendo peor pronóstico en caso de cardiopatías cianóticas maternas. La actitud debe se r esperar un pa rto vagi nal, aunque se han de evitar pe ri odos de d il atac ión y expul sivos pro longados. La cesá rea aumenta la morta li dad en pacientes ca rd iópatas y ha de recomenda rse evitar el embarazo en caso de card iopatías graves o mal compensadas, como aneurismas aórticos, siendo un buen método la anticoncepción de barrera .

Dermatología

36.7. Epilepsia y embarazo

36

1

mana de gestación y, posteriorm ente, si el feto es Rh(+), una nueva dosis dentro de las primeras 72 horas pos parto (Figura 82). Estará contraindicada la profi lax is en m uj eres con detección de anticuerpos

La epilepsia no contraindica la gestación. Se han descrito de dos a tres veces

positiva. También se debe administra r una dos is de lgG anti-D tras

mayor riesgo de muerte feta l, prematuridad y malfo rmaciones congénitas

aborto, ya sea espo ntáneo o por interrupción voluntaria, em ba razo

(labio leporino, pa lada r hendido y malformaciones cardíacas). Las crisis au-

ectóp ico, biopsia cori al o amn ioce ntes is, fu niculocentesis, ve rsión cefá lica externa.

mentan en un 15% de los casos, disminuyen en un 24% y no cambian en un

25%. Se aconseja pautar ácido fól ico a dosis de 5 mg/día y vitamina K, 20 mg/ día, dos semanas antes de la fecha prevista del parto, y 1 mg i.m. al neonato al nacimiento. No está justifica do el cambio de fármaco antiepiléptico, porque

Mujer

ninguno es total mente inocuo. Se recomienda monoterapia y la menor dosis posible, teniendo en cuenta que la gestación varía la dosis de fármaco libre. Rh(-)

Rh(+)

La carbamazepi na y el ácido va lproico aumentan el riesgo de defectos del tubo neural (1o/o) y la difen ilhidantoína produce el síndrome de la hiCoombs indirecto a la mujer

dantoína fetal (véase Ca pítulo 34. Fármacos y embarazo). De los nuevos

Nada

fármacos antiepi lépt icos, la lamotrigina es del que se t iene experien cia clínica más amplia y en el que no se han observado mayor número de anomalías congén itas que en la población normal.

36.8. Gestante Rh negativo

Mujer sensibilizada, la profilaxis no está indicada

Gammaglobulina humana anti-D a las 28-32 semanas de gestación

Cuando la madre es Rh(-) y el padre Rh(+) (aproximadamente el 12% de pa rejas), el problema ocurre si el feto es Rh(+) Si hay isoi nmunización Rh,

BebéRh(-)

~

Parto

~

BebéRh(+)

la mad re desa rroll a anticuerpos anti-Rh en respuesta al antígeno presente en el feto, fundam entalmente el antígeno D. En sucesivos embarazos, Nada más

los anticuerpos lgG, sintetizados ya en mayor cantidad y con mayor afinidad por el antígeno, atraviesan la placenta y prod ucen hemólisis fetal.

2.' dosis de gammaglobu lina antes de las 72 h

Figura 82. Profilaxis

En un 25% de los casos, se presentará la fo rm a más grave (hidrops fetalis).

Tratamiento Se debe realizar a toda embarazada la determinaci ón del grupo ABO, del factor Rh y el test Coombs indirecto (que detecta anticuerpos circu lantes) en la primera visita.

Los tratam ientos disponibles pa ra su utilización durante el em barazo son: fun icu locentesis con transfusión intraute rin a o plasmaféresis y administración de altas dosis de inmunog lobul inas. Su indicación se realizará en

Profilaxis

función de la edad gestacional, de la t asa de anticuerpos circu lantes y de

Se debe admin ist rar lg anti-D en gestantes Rh(-) con test de Coo m bs

consistirá en la fototerapia únicamente o combinada con la exang uino-

indirecto negativo si la pa reja es Rh(+) o desconocido en la 28• se-

transfusión.

los hal lazgos ecográficos de hidrops. En el recién nacido, el tratam iento

importante que se manifiesta por una gran elevación de las transam inasas y alteración de la s pruebas de coagulación. Es necesaria la finalización del embarazo, por la elevada mortalidad materna y feta l que implica.

Ideas clave Jf!S "

"

La colestas is intrahepática se caracteriza por la aparic ión de prurito como síntoma más precoz. En algunos casos puede aparecer también ictericia y coluria. En la analítica se encuentra una elevación de la fosfatasa alcalina y de la b ili rrubina con transaminasas normales. Se trata con ácido ursodesoxicólico y colestiramina para aliviar los síntomas, y se suplementan las vitaminas liposolubl es. El hígado graso agudo del embarazo es un cuadro grave con una clínica inespecífica en la que hay una lesión hepática

"

A todas las embarazadas hay que realizarles grupo, Rh y test de Coombs (indirecto). En las mujeres con test negativo, se realizará profila xis con anti-0 en dos momentos: semana 28 y en el parto. Además, habrá que administrar una dos is extra siempre que la paciente se someta a una prueba invasiva, como por ejemplo, amniocentesis. En los casos de aborto y embarazo ectópico, si la paciente es Rh negativa, también hay que administrar profila xis anti-0.

Manual CTO de Medicina y Cirugía, 2.8 edición

Case Study A 36-year-old healthy woman, gravida 3, para 2, presented to hospital at 35 weeks gestation with a three-day history of malaise, nausea, vomiting and abdominal pains. Her pregnancy had been uncomplicated. On admission, her vital signs appeared to be stable, but her blood pressure was 140/90 mmHg with no proteinuria. The results of the complete blood count was as follows: hemoglobin 11.2 g/dl and platelets 127,000/ml. The biochemical markers were as follows: AST 318.9 UI/L, ALT 439.8 UI/L

36 · Otras patologías de la gestante

and creatinine 1.5 mg/dl. The coagulation profile was as follows: prothrombin time (PT) 13 sec; activated partial thromboplastin time (APTT) 41 sec. The most likely diagnosis is: 1) 2) 3) 4)

Pre -eclampsia. Acute fatty liver of pregnancy. lntra hepatic cho lestasis of preg nancy. Herpes gestationis.

Correct answer: 2

Ginecología y obstetricia

1

Recommended reading 1

In the face of a cardiotochographic recording where we find wh at is shown in the attached image, indicate which ofthe following answers would be the most likely to justify it [Figure 1a]: l.

Hypoxaemia and acidosis.

-f- FHR bpm

FHR bpm

=f=~.o

200

- 1- 180

180

+ f-

160

160

' ·-

Failure of the placenta to adequately perform gas exchange. Foetal head compression.

h-:71

4.

Decreased oxygen supply to the foetus.

-1- 120

120

S.

Uterine hypertonia.

- f-

100

100

- f-

80

80

- 1-

60

60

2. 3.

Since this is a di rect question, the word ing does not add any data to guide us. We are com pletely dependent on our capacity to interpret the image. However, we can start from the possible answers in arder to find the solution.

1

1 1\

100

J

Answers nos. 1, 2 and 4 describe situations that would produce foetal

1 1

hypoxia. Therefore, in t he th ree situations, t he expected alteration would be type 11 DIPs. Even if yo u are not capable of recogn ising them, yo u can

\

1 1 .;.!1

could be produced by at least these three answers, and, evidently, all three cannot be simu ltaneously correct. Consequently, what we see in

""-

this image cannot be type 11 DIPs. As rega rds answer no. S, uterine hy-

Figure 1a.

1

t- 10 -

60

. \.1 _\20

f-- 12

l

1 1

8

·~\ ... ..A

80 1

1'1

s-,-o

l \

6 -

UA U.

100

1\

-\ 4

=H -

Y-i

( 1 1 \ 1

& 1/ l _L

80

1\ 40

imagine that we are not observing t hi s type of decelerations, since they

1

(

""' .......

\

40

1-

20

~

pertonia could hinder gas exchange, due to the compression that the myometrium would produce on the vessels, since it is hyperton ic. There-

pression, and are due to vagus nerve stimulation, for wh ich reason they

fore, th is answer has som ething in common with nos. 1, 2 and 4, and, as a

subside with atropine. Type 11DIPs do show such a t ime gap, greater than

resu lt, we shou ld not t ru st it. However, option no. 3 stays "hang ing". Even

20 seconds between the decrease in heart rate and t he uterine co ntrac-

without great knowledge about th e subject, it could be identifi ed as the

tion. They are ca u sed by foeta l acidosis and, therefore, ha ve a worse prog-

correct answer by elimination.

nosis than type 1DIPs. In fact, the appeara nce of type 11 DI Ps is an indication to study the foeta l pH by means of microtomy.

What the image shows are type 1DIPs. As you can see, the decreases in heart rate are completely synch ronous w ith the uterine contractions,

Below, you can compare type 1DIPs and type 11 DIPs; it is easy to distin-

without any t ime gap. They are usually associated with foetal head com-

guish t hem in the case of a typical image.

Ginecología y obstetricia



Recommended reading 1

1- 1- FHR bpm

-¡- FHK bpm -¡-- 200

--

180

+- 160 ~ [; ~~

'

¡- t ,HK bpm r- 200

180

1-- 180

180

160

r-

160

dp

pe ll;

:f=

f-

t--

-1-- 120

. f,HR bpm

200

200

160

-

~

r- r-j-;1r-

120

120

-

1--,...-- 100

--

80

-1--

60

100

80-

1-- 100 -

60

-

60

100

-

1 1\

100

l

80

1 ¡_ 1/ 1

\

60 \ 40

. 12o

{ 1/'ul\. -~~~

r1

UA D.

r1

~~- '- 12 1 1 1

1 1

1 - 10 -

1

8

60 40

1

1

1-\ 2 'i>~

80

-

1

..,

20

60

1 1 1

1 40

1

80 f-

l-10

1 1 1

1_ 1

8

1 1 1

1-6 4

\

20

1\

1\A\ 1\\.1'1&1

100

1( - r- 1-12

100

1

1

4

60

80

'1

_\.,_ -

80

100

1

6 -

1\

1¿

'\

60

40 f20

1:- 2

1"\. 1 ~ ".A ¡,.... rV'I ../

l ~~o/

'

Figure 1c. DIP 11.

Figure 1 b. DIP l.

Good prognosis

lntermediate prognosis

Bad prognosis

120 to 160

> 160

Variability

10 to 2S

Sto 1O